Download as pdf or txt
Download as pdf or txt
You are on page 1of 686

Topic Tag

PHASE I + II
PM JANAM ESI – Government Schemes

Fast Track Special Court Scheme ESI – Government Schemes

PHASE I
VR Lalithambika GA – Persons in News
Arshia Sattar GA – Persons in News
Henry Kissinger GA – Persons in News
World Aids Day GA – Days in News
RELEVANT FOR
PHASE 1 & PHASE 2
• Total outlay – Rs.24,104 crore (Central
Share:Rs.15,336 crore and State Share: Rs.8,768
crore)
• Focus on 11 critical interventions through 9 line
Ministries including Ministry of Minority Affairs
Following are the 11 critical interventions:
▪ Provision of pucca houses - 4.90 lakh
▪ Connecting roads - 8000 KM
▪ Piped Water Supply - All PVTG habitations
including 4.90 lakhs HHs to be constructed under
the mission
▪ Community water supply - 2500 Villages/
habitations with population of less than 20 HHs
▪ Mobile Medical Units with medicine cost - 1000
(10/district)
▪ Construction of hostels - 500
▪ Vocational education & skilling - 60 Aspirational
PVTG blocks
▪ Construction of Anganwadi Centers - 2500
▪ Construction of Multipurpose Centers
(MPC) - 1000
▪ Energization of HHs (Last mile connectivity)
- 57000 HHs
▪ Provision of 0.3 KW solar off-grid system -
100000 HHs
▪ Solar lighting in streets & MPCs - 1500 units
▪ Setting up of VDVKs - 500
▪ Installation of mobile towers - 3000 villages
Other Ministerial Intervention:
▪ Ministry of Ayush will set up Ayush
Wellness Centre as per existing
norms.
o Ayush facilities will be extended • According to Census 2011, India has ST
population of 10.45 crore.
to PVTG habitations through
• Out of which 75 communities located in 18
Mobile Medical Units. States and the Union Territory of
▪ Ministry of Skill Development and Andaman and Nicobar Islands have been
Entrepreneurship will facilitate skill categorized as Particularly Vulnerable
Tribal Groups (PVTGs).
and vocational training in PVTG
habitations, Multipurpose centres
and hostels as per the suitable
skills of these communities.
CT 360 GA+ESI – 16th November 2023
• Launch Year: 2019
• Ministry: Ministry of Law and Justice
• Implemented by: Department of Justice
• Centrally Sponsored Scheme
• Aim: Effective implementation of Criminal Law
Amendment Act, 2018 by ensuring targeted disposal of
pending rape & POCSO Act cases by setting up Fast Track
Special Courts (FTSC).
• Financial implication: Rs. 1952.23 crore (Rs. 1207.24 crore
as Central Share and Rs. 744.99 crore as State share).
• Key Features
• It mandates the establishment of exclusive POCSO Courts
for districts with over 100 POCSO Act cases.
• Central share is to be met from Nirbhaya Fund
• Setting up Fast Track Courts (FTCs) and its functioning
comes within the domain of the State Governments who set
up such courts in consultation with respective High Courts
As on August 2023, 758
• Scheme does not intend to create any permanent FTSCs including 411
infrastructure. exclusive POCSO Courts
are functional in 29 States
/ UTs which have disposed
of more than 1,88,000
pending cases.

At the launch of the scheme a total of 1023 FTSCs has to be set up under
the scheme out of which 389 FTSCs were exclusively meant to handle
POCSO Act cases and the remaining 634 FTSCs has to deal with either
rape cases or both rape and POCSO Act cases depending on the pendency
and requirement
RELEVANT FOR
PHASE 1
• Award Name: Légion d'honneur
• Given for: significant contributions to fostering
space cooperation between France and India.
• Presented by: Thierry Mathou, Ambassador of
France to India.

• About Legion D’honneur


• Established in 1802 by Napoleon Bonaparte. the Legion of Honour
stands as the highest civilian award bestowed by the French
Republic.
• It is granted in recognition of exceptional service to France,
without regard to the nationality of the individuals receiving the
honour.
• Award: Chevalier dans l’Ordre des Arts et des Lettres
(Knight of the Order of Arts and Letters).
• Given for: ‘outstanding achievements in the field of
literature’
• About Arshia Sattar
• She co-founded Sangam House in 2008 along with
David William Gibson with the vision of creating a
supportive space for writers expressing themselves in
regional languages.

Her book Mahabharata for Children was


awarded the Sahitya Akademi Prize for
Children’s Literature in 2022.
• Won Nobel Prize for Peace in 1973 along with Le
Duc Tho.
• Given for jointly having negotiated a cease fire in
Vietnam in 1973.
About Henry Kissinger
• He served as US Secretary of State and National
Security Advisor under former President Richard
Nixon and Gerald Ford.
Theme - “Let
communities lead”.
Let’s Practice
ESI Government Schemes

Q1. Which statement is correct regarding the PM JANAM scheme?


1. It has a provision of providing pucca houses.
2. It has a provision of providing 0.3 KW solar off-grid system to 10000
households.
3. It is a centrally sponsored scheme.
A. 1 and 2
B. Only 3
C. 1 and 3
D. 2 and 3
E. Only 2

Answer: Option C
GA Persons in News

Q2. Recently ISRO scientist V.R. Lalithambika has been conferred with
Légion d'honneur award for her significant contributions to fostering
space cooperation between France and India. The award was stated in
which of the following year?
A. 1881
B. 1805
C. 1802
D. 1757
E. 1702

Answer: Option C
GA Persons in News

Q3. Recently (November 2023), Arshia Sattar has been awarded Knight of the
Order of Arts and Letters by French government. Before it she has also won
the Sahitya Akademi Award for her children book depicting which of the
following famous epic saga?
A. Ramayan
B. Bhagvad Gita
C. Mahabharat
D. Shiv Puran
E. None of the Above

Answer: Option C
ESI Homework Question Government Schemes

Q4. Which of the following statements is/are correct with respect to the Fast
Track Special Court scheme?
1. It is being implemented by Department of Justice.
2. The financing of the Scheme will be on the pattern of Centrally Sponsored
Scheme.
3. Setting up Fast Track Courts (FTCs) and its functioning comes within the
domain of the State Governments who set up such courts in consultation
with respective High Courts.
A. 1 and 2
B. 2 and 3
C. 1 and 3
D. 1, 2 and 3
E. None of the above
Home Work Question from
previous lecture
ESI Homework Question Social Justice

Q4. __X__ is the only film festival in South Asia that is accredited by
International Federation of Film Producers’ Associations (FIAPF) in Competitive
Feature Films Category and was started in 1952.
54th edition of __X__ has taken place and which of the following film has won
the Best Film award.
A. Endless Borders
B. Kantara
C. Blaga’s Lessons
D. Party of Fools
E. When The Seedlings Grow

Answer: Option A
ESI Homework Question Government Schemes

Q5. Which statement is/are incorrect regarding the scheme for Providing
Drone to the Women Self Help Group?
1. Central Financial Assistance @ 80% of the cost of drone and
accessories/ancillary charges up to a maximum of Rs. Eight Lakh will be
provided to the women SHGs for purchase of drones.
2. One of the members of the women SHGs who is well qualified, 18 and
above years of age will be elected for 10 day training
3. Funds are allocated for the year 2023-24 only.
A. Only 1
B. 2 and 3
C. Only 3
D. 1 and 2 Answer: Option B
E. All of the above
Topic Tag
PHASE I + II
Codex Alimentarius Commission ESI – Social Sectors - Healyj

International Maritime Organization ESI – Regional Economic Co-operations

PHASE I
WISE Prize for Education GA – Awards
International Day of Persons with Disabilities GA – Days in News
Aarogya Maitri Aid Cube GA – First in News
Cyclone Michaung GA – Environment
RELEVANT FOR
PHASE 1 & PHASE 2
• It is significant as it positions India in a pivotal role in the international
standard-setting process for food products.
Key Highlights
• India's proposal for global standards for millets has been accepted by
the Executive Committee.
About Executive Committee under Codex Alimentarius Commission
• It comprises of the Chair, three vice Chairs, six regional coordinators
and seven elected representatives from the various geographical
regions of Codex.
About Codex Alimentarius Commission
• Founded in 1963.
• It is a collection of internationally adopted food
standards and related texts presented in a
uniform manner.
• Aim: To develop and endorse food standards,
guidelines, and codes of practice contributing
to the safety, quality, and fairness of the
international food trade.
• Tenure - 2024–25 biennium.
• India falls under the Category of 10 states with "the
largest interest in international seaborne trade",
• Alongside Australia, Brazil, Canada, France, Germany, the
The JPO Program is an established
Netherlands, Spain, Sweden, and the United Arab Emirates program within the United Nations with
(UAE). the main objective of providing young
professionals an opportunity to gain
• India has proposed to nominate at least 2 qualified hands-on experience in international
candidates for the Junior Professional Officer (JPO) cooperation under the supervision of
Program at the IMO. specialists and to contribute to the
advancement of their nation’s mandate.
Key initiatives focus on strengthening our
global maritime presence as part of Amrit
Kaal Vision 2047 Action Plan includes
dedicated IMO cell in India, appointment of
a permanent representative at IMO
headquarters, London, implementation of
BIMSTEC Master Plan, creating a robust
BIMSTEC institutional structure to ensure
implementation of regional projects in a
coordinated & timely manner etc.
RELEVANT FOR
PHASE 1
• Award Name: WISE Prize for Education
• Given for: for her significant contributions to enhancing equity and
access to education.
About Safeena Hussain
• Founder of NGO Educate Girls (founded in 2007)
• Other Prominent Awards: USAID Millennium Alliance Award, MIT
Solve’s Learning for Girls & Women Challenge, Skoll Award for
Social Entrepreneurship, the WISE Award, and the World Bank India
Development Marketplace Award, for her work in girls’ education

'Pratham' co-founder Madhav


Chavan was the first Indian
About World Innovation Summit for Education
recipient of the WISE prize in 2012
• Established in 2011 for providing quality education to
• It is an initiative spearheaded by Her Highness of Qatar Sheikha Moza bint Nasser. millions of underserved children in
• 11th edition of WISE Summit held in Doha with the theme “
• Organised by the Qatar Foundation in association with Education Above All.
India.
• theme ‘Creative Fluency: Human Flourishing in the Age of AI.
• The day is dedicated to spreading awareness and
mobilizing support for the rights and well-being of
people with disabilities globally.
• Theme: ““United in action to rescue and achieve the
SDGs for, with, and by persons with disabilities.”
About Aarogya Mitra Aid Cube
• Designed indigenously under the Project 'BHISHM’.
• It is a modular trauma management and aid system for
emergency response and humanitarian efforts.
• It can handle bullet injuries, burns, head, spinal and
chest injuries, minor surgeries, fractures and major
bleeding.
• It can treat as many as 200 patients.

Project BHISHM stands for Bharat Health Initiative for Sahyog


Hita and Maitri.
It relies on the Rubik’s Cube concept and each of the cubes is
designed to weigh less then 20 kilos to make it easy to carry
up manually.
• Name proposed by: Myanmar.
• Next Cyclone will be named as Remal given by Oman.

RBI Grade B 2023 – Phase I


Let’s Practice
GA Government Initiative

Q1. Recently (November 2023), world’s first portable hospital “Aarogya Maitri
Aid Cube” has been unveiled at Gurugram under which of the following
project?
A. ARJUN
B. BHISHM
C. MAITRI
D. BRAHMOS
E. AAROGYA

Answer: Option B
GA Environment

Q2. According to the 'List of North Indian Ocean Tropical Cyclone


Names', what is the name of the next cyclone after cyclone ‘Michaung'
and the name has been recommended by which country?
A. Tej, India
B. Asan, Pakistan
C. Marasur, India
D. Remal, Oman
E. Remal, Yemen

Answer: Option D
ESI Social Sectors - Health

Q3. Recently (November 2023), India has joined the Codex Alimentarius
Commission which is a significant move as it positions India in a pivotal role in
the international standard-setting process for food products. The commission
was founded in which of the following year?
A. 1964
B. 1965
C. 1950
D. 1963
E. 1970

Answer: Option D
GA Homework Question First in News

Q4. Recently (November 2023), Safeena Hussain, founder of 'Educate


Girls,' has become the first Indian woman to bag the WISE Prize. She is
the second Indian to won the awards after Madhav Chavan who is the
co-founder of which of the following organization?
A. Sarthi
B. Saras
C. Pratham
D. Parimal
E. Satyarthi
ESI Homework Question Regional Economic Co-Operation

Q5. Which among the following statements given is/are incorrect with
respect to the International Maritime Organization?
1. India has been re-elected as the member of the IMO council till
2024-25.
2. Mr. Kitack Lim is the secretary general of IMO.
3. First Meet of IMO took place in 1948.
A. 1 and 2
B. 2 and 3
C. Only 1
D. Only 2
E. Only 3
Home Work Question from
previous lecture
ESI Homework Question Government Schemes

Q4. Which of the following statements is/are correct with respect to the Fast
Track Special Court scheme?
1. It is being implemented by Department of Justice.
2. The financing of the Scheme will be on the pattern of Centrally Sponsored
Scheme.
3. Setting up Fast Track Courts (FTCs) and its functioning comes within the
domain of the State Governments who set up such courts in consultation
with respective High Courts.
A. 1 and 2
B. 2 and 3
C. 1 and 3
D. 1, 2 and 3 Answer: Option D
E. None of the above
Topic Tag
PHASE I + II
PLI Scheme for Textile ESI – Government Schemes

National Circular Economy Roadmap for ESI – Sustainable Development and


reduction of Plastic waste in India Environmental Issues
Build for Bharat ESI – Employment Generation in India

India Ageing Report 2023 ESI – Reports and Indices

PHASE I
Gram Manchitra GA – Government Initiatives
Suganthy Sundararaj GA – Persons in News
Garba GA – Miscellaneous
RELEVANT FOR
PHASE 1 & PHASE 2
• Production Linked Incentive (PLI) Scheme for Textiles
• Launch: 2021
• Nodal Ministry: Ministry of Textiles
• Aim: To promote production of high value Man-Made Fibre (MMF)
fabrics, garments and technical textiles
• Financial Allocation: ₹10,683 crore for production over a span of 5
years from FY 2025-26 to FY 2029-30
• on incremental turnover achieved during FY 2024-25 to FY 2028-
29.
• Priority to be given to investment in Aspirational Districts, Tier 3,
Tier 4 towns and rural areas
Under the scheme
• Gestation period: 2022-23 and 2023-24
• Performance years: 2024-25 to 2028-29
Key Features
• Scheme Part- 1: Any person including Company/Firm/LLP/Trust
willing to create a separate manufacturing company and invest
minimum ₹300 Crore (excluding land and administrative building cost)
• Eligible to get incentive when they achieve a minimum of ₹600 Crore
turnover by first Performance Year
• Scheme Part- 2: Any person including Company/Firm/LLP/Trust
willing to create a separate manufacturing company and invest
minimum ₹100 Crore (excluding land and administrative building cost)
• Eligible to get incentive when they achieve a minimum of ₹200 Crore
turnover by first Performance Year

Incentives for Companies investing


• Over ₹300 crore in plant, machinery, equipment and civil works will
get incentive of 15% of their turnover, which needs to be ₹600 crore
in 3rd year
• Between ₹100 crore and ₹300 crore are eligible to receive duty
refunds and incentives of 11% of their turnover after achieving double
turnover than invested by 3rd year
National Circular Economy Roadmap for reduction of Plastic
waste in India
• Research began in July 2020.
• Collaborative effort by Indian and Australian research
institutions by involving TERI, CSIR-NEERI, and Australian
universities.
• Aim/Objective: Establish a circular economy in India's plastics
sector and reduce plastic waste.
• It is a part of the Australia-India Comprehensive Strategic
Partnership, supporting the UN Global Plastics Treaty.
Features:
• Landfill reduction by 30% and increase in
recycling rates to 67% by 2035.
• Complete phasing out of single-use plastics by
2035.
• Over 80% of waste streams to be digitally
tracked and managed by 2035.
• A cleaner environment with 20-50% less
greenhouse emissions, improved air quality,
and reduced microplastics in the food chain
by 2035.
• Creation of secondary markets for used
plastics and primary markets for eco-friendly
products.
• Serving as a model for global transformative
change.

Seven Pillars of Circular Economy for Plastic


About Build for Bharat' initiative.
• Launched by: Open Network for Digital Commerce
(ONDC), Google Cloud India, Antler in India, Paytm,
Protean, and Startup India.
• Aim/Objective: To address challenges in digital commerce
and foster innovation and practical solutions across
various sectors.
Key Facts and Initiatives:
• Nationwide program with over 200,000 participants
including startups, enterprises, and educational
institutions.
• Implemented in more than 50 cities with the participation
of leaders, specialists, venture capitalists, and
incubators.
Three key categories within the initiative:
• 'NextGen Ventures' for Venture Creation: Focused on accelerating
venture creation on ONDC, offering capital raising pathways,
launch and scale support, and business incubation for aspiring
founders and early-stage teams.
❑ Winners receive opportunities from Antler in India, mentorship,
and equity-free grants up to 5 crore rupees.
• 'Scalable Solutions' for Addressing Friction Points of Network
Participants: Invites participation from organizations and
individuals to develop solutions for challenges faced by Network
Participants.
❑ Winners receive Cloud credits from Google Cloud India for
further project development.
• 'Foundation Solutions' for College Students: Aimed at students over
18 years of age to identify proof of concept for friction points faced
by NPs.
❑ Winners benefit from Cloud credits from Google Cloud India.
Released by: International Institute for Population
Sciences and the United Nations Population Fund
About the Report
• Previous edition was released in 2017
• Theme: Caring for Our Elders Institutional Responses
Stresses 3 key challenges
• Women living longer than men
• A high proportion of rural population among the
elderly
• Ageing of the aged
Key Findings
Global Findings
• People aged 60 years or above: 1.1 billion (13.9%
of the Global population)
• Number of older persons in the world is likely to
double to 2.1 billion by 2050 (22% of the total
population)
• Elderly population will grow from 11.5% in 2022
to 20% in 2050

India Findings
• Elderly population growth rate in India is 41%
(40.6%)
• Elderly in India’s population will double by 2050
• By century-end, elderly to surpass 36% of India's
population
• During 2000 and 2022: Total population of India
grew by 34%, Elderly grew by 103%
• During 2022-2050: Total population will increase
by 18%, Older population will grow at 134%
• Population of persons aged 80+ years will grow
279%
• India has 65% of inhabitants under 35 (One of the
Highest in the world)
• Women have higher life expactancy at the age of 60
and at the 80, when compared to men
• By 2031: 951 men for 1,078 women
• Life expectancy: Men aged 60(18.3 years) Women United Nations had declared 2020-30 as the
Decade of Healthy Ageing
aged 60 (19 years) United Nations General Assembly (UNGA)
• 18.7% of elderly people did not have any income has designated October 1 as the International
• Over 40% elderly are in poorest quintile Day of Older Persons
RELEVANT FOR
PHASE 1
• Launched by the Ministry of Panchayati Raj.
• Aim/Objectice:
• To encourage Spatial Planning by the Gram Panchayat.
• Facilitates Gram Panchayats in planning at their level
using geo-spatial technology.
• Key Facts and Initiatives:
• Offers a unified Geo Spatial platform for visualizing
developmental works across different sectors.
• Provides a decision support system for Gram Panchayat
Development Plan (GPDP).
• Repository of information on works and assets related
to various sectors like water harvesting, agriculture, mActionSoft, a mobile solution,
and irrigation. launched for capturing photos with
Geo-Tags for works resulting in assets.
• Tools assist in site identification for projects, asset
• Geo-tagging of assets occurs
tracking, cost estimation, and project impact before, during, and after the
assessment. completion of work.
• Key Facts:
• Given By: Public Relations Society of India (PRSI)
• Purpose of Receiving the Award:
• Recognizing her dedication to the
communications and public relations industry,
particularly in the healthcare domain.
• Acknowledging her role in shaping strategic
management of the public relations function in
healthcare.
Garba is the 15th cultural item from India to be added to the
UNESCO list, following Kolkata's Durga Puja in 2021.
Other Inscriptions added:
• Rickshaws and Rickshaw painting in Dhaka from Bangladesh,
• Songkran, the traditional Thai New Year festival from Thailand;
• Hiragasy, a performing art of the Central Highlands of
Madagascar;
• Junkanoo from the Bahamas, and the Procession and
celebrations of Prophet Mohammed’s birthday in Sudan.

The list of the Intangible Cultural Heritage of Humanity of the Ramlila, Tradition of Vedic Chanting
2003 UNESCO Convention for the Safeguarding of the and Kuttiyam, Sanskrit Theater was
Intangible Cultural Heritage currently has some 704 elements the first Indian sites to be included in
corresponding to 5 regions and 143 countries. list in 2008 .
Let’s Practice
GA Miscellaneous

Q1. Recently (November 2023), Garba is the 15th cultural item from India to
be added to the UNESCO list, following Kolkata's Durga Puja in 2021. Ramlila,
Tradition of Vedic Chanting and Kuttiyam, Sanskrit Theater was the first Indian
sites to be included in list in _____.
A. 2003
B. 2008
C. 2004
D. 2005
E. 2006

Answer: Option B
GA Government Initiative

Q2. Which among the following statements is/are correct with respect
to the Gram Manchitra initiative?
1. It is an initiative of Ministry of Rural Development.
2. It’s aim is to encourage Spatial Planning by the Gram Panchayat.
3. It offers tools that assist in site identification for projects, asset
tracking, cost estimation, and project impact assessment.
A. 1 and 2
B. Only 2
C. 1 and 3
D. 2 and 3
Answer: Option D
E. Only 1
ESI Social Sectors - Health

Q3. Which among the following statement(s) is/are correct with respect to
the PLI Scheme for Textile?
1. Priority will be given to investment in Aspirational Districts, Tier 1, Tier 2
towns and urban areas.
2. Any person including Company/Firm/LLP/Trust willing to create a separate
manufacturing company and invest minimum ₹100 Crore (excluding land and
administrative building cost) is eligible in part 2 of the scheme.
3. In case of fast paced investment when threshold investment and threshold
turnover is achieved by FY 2023-24, incentive cannot be given.
A. Only 1
B. Only 2
C. Only 3 Answer: Option B
D. 1 and 2
E. 2 and 3
ESI Reports and Indices

Q4. Consider the following statements regarding the India Ageing Report
2023:
1. 1.1 billion (13.9% of the Global population) people aged 60 years or above
2. The number of older persons in the world is likely to double to 2.1 billion by
2030 (22% of the total population)
3. In the Developed world the share of older persons will increase from 26%
in 2022 to 50% in 2050
Which of the following statements is/are correct?
A. Only 1
B. Only 2
C. Only 3 Answer: Option A
D. Both 1 and 2
E. Both 2 and 3
Sustainable Development and
ESI Homework Question
Environmental Issues

Q5. Which among the following statement is/are correct with respect
to the targets of National Circular Economy Roadmap for reduction of
Plastic waste in India to be achieved by 2035?
1. Landfill reduction by 30% and increase in recycling rates to 67%.
2. Complete phasing out of single-use plastics.
3. Over 90% of waste streams to be digitally tracked and managed.
A. Only 1
B. Only 2
C. Only 3
D. 1 and 2
E. 2 and 3
ESI Homework Question Employment Generation in India

Q6. Which among the following statement is/are incorrect with respect to the
targets of three key categories within the Build for Bharat' initiative?
1. 'NextGen Ventures' for Venture Creation winners will receive
opportunities from Antler in India, mentorship, and equity-free grants up
to 5 crore rupees.
2. 'Scalable Solutions' for Addressing Friction Points of Network Participants
aimed at students over 18 years of age to identify proof of concept for
friction points faced by NPs
3. 'Foundation Solutions' for College Students invites participation from
organizations and individuals to develop solutions for challenges faced by
Network Participants.
A. Only 1
B. 2 and 3
C. Only 3
D. 1 and 2
E. Only 2
Home Work Question from
previous lecture
GA Homework Question First in News

Q4. Recently (November 2023), Safeena Hussain, founder of 'Educate


Girls,' has become the first Indian woman to bag the WISE Prize. She is
the second Indian to won the awards after Madhav Chavan who is the
co-founder of which of the following organization?
A. Sarthi
B. Saras
C. Pratham
D. Parimal
E. Satyarthi

Answer: Option C
ESI Homework Question Regional Economic Co-Operation

Q5. Which among the following statements given is/are incorrect with
respect to the International Maritime Organization?
1. India has been re-elected as the member of the IMO council till
2024-25.
2. Mr. Kitack Lim is the secretary general of IMO.
3. First Meet of IMO took place in 1948.
A. 1 and 2
B. 2 and 3
C. Only 1
Answer: Option E
D. Only 2
E. Only 3
Topic Tag
PHASE I + II
Global Partnership on Artificial Intelligence ESI – Human Development

Global River Cities Alliance ESI – Sustainable Development and


Environmental Issues
World Malaria Report ESI – Reports and Indices

PHASE I
First Bullet Train Terminal GA – First in News
Naye Bharat ka Samaveda GA – Books and Authors
Integrated Urban Flood Management GA – First in News
Activities for Chennai Basin Project
Taylor Swift GA – Persons in News
International Lunar Research Station GA – Science and Technology
RELEVANT FOR
PHASE 1 & PHASE 2
• Venue: Bharat Mandapam, New Delhi
• India is the lead chair of GPAI for 2024.
About GPAI
• Launch Year: June 2020.
• It has a Council and a Steering Committee, supported by a Secretariat
hosted by the OECD, and two Centres of Expertise in Montreal and Paris.
• Member Countries: 29
• GPAI’s founding members include Australia, Canada, France, Germany,
India, Italy, Japan, Mexico, New Zealand, the Republic of Korea, Singapore,
Slovenia, the United Kingdom, the United States of America, and the
European Union.
• Objective: Bridge the gap between AI theory and practice, promote
responsible AI development.
Features:
• Collaborative working groups on responsible AI, data governance, future
of work, and innovation.
• Focus on comprehensive, objective, open, and transparent AI
assessments​.
YUVA Ai Youth for Unnati and Vikas with AI’
• A collaborative initiative of National e-Governance Division (NeGD),
Ministry of Electronics & Information Technology (MeitY) and Intel India
• Implementing Agency: National e-Governance Division (NeGD), Intel India
• Aim/Objective: to foster a deeper understanding of AI, to enable school
students from class 8 to 12 across the nation with AI skills and empower
them to become human-centric designers and users of AI..
Progressing in three phases:
• First Cohort: More than 8,500 students registered, post which, they
attended online orientation sessions to learn fundamental concepts of AI.
• Phase 2, Top 200 AI-based ideas were shortlisted and shortlisted
students attended online deep dive AI training and mentorship sessions
with certified Intel AI coaches and experts – helping students enhance
their solutions.
• Students then submitted their AI projects to be evaluated for Phase 3.
• In Phase 3, Top 50 students were shortlisted and they were invited to
attend a four-day face-to-face rapid modeling workshop – receiving one-
on-one mentorship, apprenticeship and guidance from industry experts to
fine-tune their projects and develop them into prototypes.
• An on-spot project presentation was conducted by a multiple jury panel
to shortlist Top 10 students.
About Global River Cities Alliance
• Launched at COP28.
• Ministry Involved: Ministry of Jal Shakti, Government of India
• Implementing Agency: National Mission for Clean Ganga
(NMCG)
• Aim/Objective: Promote river conservation and sustainable
urban water management.
• Members
• Countries - India, Egypt, Netherlands, Denmark, Ghana,
Australia, Bhutan, Cambodia, Japan
• Cities like The Hague, Adelaide, and Szolnok.
• International funding agencies –World Bank, ADB, AIIB, and
knowledge institutions like KPMG are partners.
Features:
• It is the first of its kind globally, signifying a commitment to
collaborative action for the well-being of rivers and their
ecosystems.
• Sustainable Urban Water Management: Focuses on
developing sustainable practices for managing urban water
resources.
• Comprehensive Approach: Integrates urban planning with
water management to enhance the health and sustainability
of river cities.
• Knowledge Exchange and Advocacy: Encourages sharing of
best practices, capacity development, and high-level
advocacy among member cities..
About World Malaria Report 2023
• Released by: WHO
Global Findings
• Number of malaria cases in 2022 is 249 million
❑ It has seen an increase of 5 million as compared to the
previous year.
❑ Of the 5 million additional cases, Pakistan had the
highest 2.1 million followed by Ethiopia, Nigeria, Uganda
and Papua New Guinea.
• Number of malaria deaths: Remained higher than the
pre-pandemic levels.
❑ There were 608,000 deaths reported in 2022 as
compared to 576,000 cases in 2019
Region Worst hit by Malaria
• Africa
❑ 94 per cent of all malaria cases (233 million cases) and 95 per cent global malaria deaths (580,000
deaths) in 2022.
❑ About 78 per cent of all malaria deaths in the Region were among children under the age of five.
• South East Asian Region
❑ Accounted for only 2 per cent of the global malaria burden and managed to contain the disease in
the last two decades.
❑ India, Bangladesh, Bhutan, the Democratic People’s Republic of Korea, Nepal, Thailand and Timor-
Leste has managed to reduce their malaria incidence by 55 per cent or more since 2015
❑ Bhutan (for the first time), Timor-Leste (for the second time) and Sri Lanka reported zero malaria
cases in 2022.
❑ Nepal, for the first time, didn’t record a single indigenous malaria death.
Target
• WHO had identified graded targets of reducing case incidence and mortality rates by 75 per cent in
2025 and 90 per cent in 2030.
❑ The world is 55 per cent off track to reach its 2025 target of reducing malaria case incidence and 53
per cent off track to achieving the 2025 target of reducing malaria fatality rate
India Specific Findings
• India accounted for 66 per cent of the 5.2 million malaria cases
recorded in 2022
❑ Highest for any country in the South East Asia region
❑ Almost 46% of all cases in the region were due to Plasmodium
vivax.
• India saw a decline of 30 per cent in malaria incidence and 34
per cent in mortality in 2022.
• India accounted for 1.4% of total malaria cases in the world.
• India, along with Indonesia, also accounted for 94 per cent of all
the deaths that occurred due to malaria in 2022.

Plasmodium vivax is a protozoal parasite and a human


pathogen, which is the most frequent and widely distributed
cause of recurring malaria
RELEVANT FOR
PHASE 1
• Place: Sabarmati Multimodal Transport Hub.

Mumbai-Ahmedabad High-Speed Rail


• Launched in 2017.
• Entities Involved in Launching the Initiative: Indian
Government and Japan (executed with technical and
financial assistance from the Government of Japan).
Project Details:
• Speed and Distance: Train to operate at a speed of 320
km/hr, covering 508 km with 12 stations.
• Project Cost: Estimated at Rs 1,08,000 Crore, with 81%
funded through a Japanese soft loan.
• Duration: Expected completion by 2028.
• Key Facts:
• Book Name: Naye Bharat ka Samaveda
• It highlights the essence of India's Constitution and
its relevance in contemporary society.
• It focuses on constitutional values, India's ethos, and
PM Modi's vision for 'Ek Bharat Shreshtha Bharat'
(One India Magnificent India).
• Name of the Initiative: 'Integrated Urban Flood
Management Activities for Chennai Basin Project’
• Launched under: National Disaster Mitigation Fund (NDMF)
• Aim: To enhance Chennai's resilience to flooding and
improve urban flood management by establishing a
framework.
• Budget: ₹561.29 crore from the National Disaster
Mitigation Fund (NDMF), including ₹500 crore of Central
assistance.
• Name of the Award: TIME Person of the Year 2023
• Given To: Taylor Swift
• She is the first woman to appear twice on a Person of the
Year cover since the franchise began in 1927
• Purpose of Receiving the Award: Swift and the first person
to be selected for achievements in the arts.
• Previous Recipient - 2022: Volodymyr Zelensky, Ukraine’s
president, and the “spirit of Ukraine.”, 2021: Elon Musk, for
his influence on the global stock market.
• Recent Female Recipients: Angela Merkel in 2015 and
Greta Thunberg in 2019.

• Before 1999, the award was titled “Man of the Year” and mostly awarded to male global leaders and CEOs.
• Only three women received the title individually before 1999.
• Wallis Simpson (1936): An American socialite whose intended marriage to British King Edward VIII led to a constitutional crisis and
his abdication.
• Queen Elizabeth II (1952): Recognized following her ascension to the throne of the United Kingdom.
• Corazon Aquino (1986): Honored for her role in the peaceful revolution in the Philippines that led to the departure of President
Ferdinand Marcos and her subsequent presidency.
Egypt joins as the first Arab country
and the second African nation after
About International Lunar Research Station (ILRS) South Africa to join ILRS.

• Led by - China National Space Administration (CNSA)


• Vision: To construct a permanent lunar base in the 2030s,
with precursor missions planned for the 2020s.
• Participating Countries: Egypt, South Africa, Venezuela,
Pakistan, and Azerbaijan.
Let’s Practice
GA Science and Technology

Q1. Recently (December 2023), Egypt joins as the first Arab country and the
second African nation after _______ to join International Lunar Research Station
(ILRS) with a vision: To construct a permanent lunar base in the 2030s
A. Argentina
B. Chile
C. Cuba
D. South Africa
E. None of the Above

Answer: Option D
GA Persons in News

Q2. Recently (December 2023), Taylor Swift has become the first woman to
appear twice on a Person of the Year cover since the franchise began in 1927.
Which among the following women have received the TIMES Magazine
Person of the year award:
1. Wallis Simpson
2. Queen Elizabeth II
3. Corazon Aquino
4. Greta Thunberg
5. Giorgia Meloni
A. 1, 2 and 3
B. 2, 3, 4 and 5
C. 1, 2, 3 and 5 Answer: Option D
D. 1, 2, 3 and 4
E. 1, 2, 3, 4 and 5
GA First in News

Q3. Recently (December 2023), India has launched its first project to tackle
urban floods as 'Integrated Urban Flood Management Activities for Chennai
Basin Project”.
It has a funding of ₹561.29 crore from the National Disaster Mitigation Fund
(NDMF), including ₹_____ crore of Central assistance.
A. 200
B. 300
C. 400
D. 500
E. 550

Answer: Option D
ESI Human Development

Q4. Which among the following statement(s) is/are correct with


respect to the Global Partnership on Artificial Intelligence (GPAI)?
1. India is the lead chair for 2024.
2. India is one of the founding members of the GPAI.
3. It’s objective is to foster a deeper understanding of AI, to enable
school students from class 8 to 12 across the globe.
A. Only 1
B. Only 2
C. Only 3
D. 1 and 2 Answer: Option D
E. 2 and 3
ESI Human Development

Q5. Consider the following statements with respect to the World


Malaria Report for the year 2023:
1. It is based on the theme “Time to deliver zero malaria: invest,
innovate, implement”.
2. India accounted for 1.4% of total malaria cases in the world.
3. Nepal, for the first time, didn’t record a single indigenous malaria
death.
Choose the correct one:
A. Only 1
B. Only 2
Answer: Option E
C. Only 3
D. 1 and 2
E. 2 and 3
ESI Homework Question Reports and Indices

Q6. WHO had identified graded targets of reducing case incidence and
mortality rates by ___ per cent in 2025 and 90 per cent in ______.
A. 50, 2050
B. 75, 2050
C. 50, 2030
D. 75, 2030
E. 65, 2030
Sustainable Development and
ESI Homework Question Environmental Issues

Q7. Which among the following statement with respect to Global River
Alliance is is/are incorrect?
1. India is one of the founding nine member countries.
2. It is implemented by Namami Gange Initiative of Government of India.
3. It Focuses on developing sustainable practices for managing urban water
resources.
A. Only 1
B. Only 2
C. 1 and 3
D. All of the above
E. None of the Above
Topic Tag
PHASE I + II
Schemes for unorganised labour ESI – Social justice

Asian Development outlook ESI – Reports and Indices

Logistics Costs in India Report ESI – Reports and Indices

PHASE I
Nyholm Prize GA – Awards in News
New PM of Poland GA – Person in news
INS Tarmugli Commissioned GA – Defence News
National Energy Conservation Day GA – Days in News
RELEVANT FOR
PHASE 1 & PHASE 2
The details of the social security schemes available for unorganised sector
workers are as follow:
• Life and Disability cover is provided through Pradhan Mantri Jeevan Jyoti
Bima Yojana (PMJJBY) and Pradhan Mantri Suraksha Bima Yojana (PMSBY).
❑ Risk coverage under PMJJBY scheme is for Rs. 2.00 Lakh in case of death of
insured, due to any reason, at annual premium of Rs. 436/-.
❑ The risk coverage under PMSBY is Rs. 2.00 Lakh in case of accidental death
or total permanent disability and Rs. 1.00 lakh for partial permanent disability
due to accident at a premium of Rs. 20/- per annum.
• The health and maternity benefits are insured through Ayushman Bharat-
Pradhan Mantri Jan Arogya Yojana (AB-PMJAY) under deprivation and
occupation criteria.
❑ It provides health insurance coverage upto Rs. 5.00 lakhs per family for
secondary and tertiary care related hospitalization.
• To provide old age protection to unorganised sector workers, the
Government of India had launched a pension scheme in 2019 namely
Pradhan Mantri Shram Yogi Maan-dhan Yojana (PM-SYM).
• It provides a monthly pension of Rs. 3000/- after attaining the age of 60
years to unorganized workers.
• The Asian Development Bank (ADB) has raised its FY24
growth forecast for India to 6.7% in the latest Asian
Development Outlook.
❑ It has raised the estimate from 6.3% it made in September
2023.
• ADB’s growth forecast for FY25 is unchanged at 6.7 per
cent.
• It kept its earlier forecasts on Indian inflation unchanged,
5.5 per cent in FY24.

• It revised the region’s growth projection to 4.9% for calendar year 2023 per cent from
4.7 per cent earlier, citing robust domestic demand.
• It maintained the forecast for 2024, at 4.8 per cent.

The Reserve Bank of India (RBI) also revised its growth


estimate last week, raising it to 7% from the earlier
projection of 6.5% for FY24.
• Prepared by: National Council of Applied Economic
Research (NCAER) with guidance of the Asian
Development Bank
• Launched by: Logistics Division, Department for Promotion
of Industry and Internal Trade (DPIIT), Ministry of
Commerce and Industry
• This report presents-
(a) a baseline aggregated logistics cost estimate
(b) a framework for long-term logistics cost calculation.

India’s National Logistics Policy, which was launched on


17th September 2022, aims to reduce logistics costs in
India.
Recommendation-
• It recommends a hybrid approach using primary (covering all trade flows, product types,
industry trends, OD pairs, etc.) and secondary survey data, as well as real-time Big Data to
provide an estimate of logistics cost.
RELEVANT FOR
PHASE 1
• She has been awarded for her advocacy of the
significance of chemical education.

The Excellence in Education Prizes celebrate


inspirational, innovative, and dedicated people working
in primary, secondary, further education and higher
education – including teachers, technicians and more.
• He will replace former Prime Minister Mateusz
Morawiecki.
Additional Information-
• Poland:
❑ Capital: Warsaw
❑ Currency: Polish złoty
• Indian Navy commissioned INS Tarmugli, a Fast Attack
Craft
• Commissioned at: Naval Dockyard Visakhapatnam

The ship is a Trinkat Class FAC which was gifted to the


Maldivian Naval Defence Forces (MNDF) in 2006 by the
Govt of India.
• Name of the Award: National Energy Conservation
Awards 2023
• National Energy Conservation Day: Celebrated on
December 14

• National Energy Conservation Day and the Awards


• Organized by: Bureau of Energy Efficiency (BEE), under Ministry of
Power, Government of India
• Prime Minister lauded the Guiness World Record of the largest
reading activity on 14 Dec 2023 at SP College, Pune
• Total of 3066 parents read to their children to promote
reading culture in the society through storytelling.
• This event was organised by: National Book Trust, India in
collaboration with Pune Municipal Corporation
Let’s Practice
GA

Q1. Recently (December 2023), who among the following has been awarded
with the Nyholm prize by the Royal Society of Chemistry?
A. Savita Ladage
B. Carolyn Bertozzi
C. John B. Goodenough
D. Eric Archer
E. None of the Above

Answer: Option A
ESI

Q2. Consider the following statements regarding the schemes launched for
unorganised labours and identify the incorrect statements:
1. Risk coverage under PMJJBY scheme is for Rs. 2.00 Lakh in case of death of
insured, due to any reason, at annual premium of Rs. 436/-
2. The risk coverage under PMSBY is Rs. 2.00 Lakh in case of accidental death or
total permanent disability and Rs. 1.00 lakh for partial permanent disability due to
accident at a premium of Rs. 10/- per annum
3. AB-PMJAY provides health insurance coverage upto Rs. 5.00 lakhs per family.
4. Pradhan Mantri Shram Yogi Maan-dhan Yojana (PM-SYM) to provide a monthly
pension of Rs. 3000/- after attaining the age of 60 years to unorganized workers.
A. 1, 2 and 3
B. 2, 3, and 4
C. 1, 2, 3 and 4 Answer: Option D
D. Only 2
E. None of the above
GA

Q3. Recently (December 2023), the Asian Development Bank (ADB) has
revised India's growth forecast for the current financial year to _____, citing
stronger-than-expected second-quarter performance. This adjustment marks
an increase from the previous projection of 6.3%, as revealed in the Asian
Development Outlook December 2023 report released.
A. 5.7%
B. 6.5%
C. 6.7%
D. 6.3%
E. 7.0%

Answer: Option C
ESI Homework Question Reports and Indices

Q4. National Energy Conservation Day is being celebrated every year on


_______.
A. December 11
B. December 12
C. December 13
D. December 14
E. December 15
ESI Homework Question

Q5. Which among the following statement with respect to Logistics Costs in India:
Assessment and Long-Term Framework is/are incorrect?
1. Logistics Division, Department for Promotion of Industry and Internal Trade (DPIIT),
Ministry of Commerce and Industry (MoCI) launched the report.
2. It is prepared by the National Council of Applied Economic Research (NCAER), in a
consultative manner, with guidance of the New Development Bank (NDB) experts.
3. It contribute towards achieving target of reducing Logistics Costs in India.
A. Only 1
B. Only 2
C. 1 and 3
D. All of the above
E. None of the Above
ESI Homework Question previous session Reports and Indices

Q6. WHO had identified graded targets of reducing case incidence and
mortality rates by ___ per cent in 2025 and 90 per cent in ______.
A. 50, 2050
B. 75, 2050
C. 50, 2030
D. 75, 2030
E. 65, 2030

Answer: Option D
Homework Question Sustainable Development and
ESI previous session Environmental Issues

Q7. Which among the following statement with respect to Global River
Alliance is is/are incorrect?
1. India is one of the founding nine member countries.
2. It is implemented by Namami Gange Initiative of Government of India.
3. It Focuses on developing sustainable practices for managing urban water
resources.
A. Only 1
B. Only 2
Answer: Option B
C. 1 and 3
D. All of the above
E. None of the Above
Topic Tag
PHASE I + II
SATHEE Portal ESI – Social Sectors - Education

India achieves 2 NDC Targets ESI – Sustainable Development and


Environmental Issues
World Bank’s Migration and Development ESI – Reports and Indices
Brief
PHASE I
Vijay Hazare Trophy GA – First in News
Exercise Astrashakti GA – Defence in News
Aditya L1 GA – Science and Technology
Khelo India Para Games GA – First in News
Surat Diamond Bourse GA – Places in News
RELEVANT FOR
PHASE 1 & PHASE 2
About SATHEE (Self-Assessment, Test and Help for Entrance Examination) portal
• Initiative of: Department of Higher Education, Ministry of Education in collaboration with IIT Kanpur.
• Aim: To bridge the gap for students who cannot afford costly entrance exam guidance and coaching.
• Purpose: To provide quality education and self-paced assessment for students preparing for
competitive exams like JEE, NEET, and other state-level engineering and various examinations
What are the targets
• India submitted its First National Communication in 2015 with 8 targets including 3 quantitative
targets:

o To reduce the emissions intensity of its GDP by 33 to 35 percent by 2030 from 2005 level.

o To achieve about 40 percent cumulative electric power installed capacity from non-fossil fuel-
based energy resources by 2030.

o Creation of additional carbon sink of 2.5 to 3 billion tonnes of CO2 equivalent through
additional forest and tree cover.

Targets Achieved

• As on 31st October, 2023: Cumulative electric power installed capacity from non-fossil fuel-based
energy resources is 186.46 MW(43.81%) of the total cumulative electric power installed capacity.

• As in December 2023: Emission intensity of its GDP has been reduced by 33 percent between
2005 and 2019.
Updated NDC Targets in August 2022:

o Target to reduce emissions intensity of its GDP has been enhanced to


45 percent by 2030 from 2005 level.

o And the target on cumulative electric power installed capacity from


About Nationally Determined
non-fossil fuel-based energy resources has been enhanced to 50% by Contributions
2030. • NDCs are at the heart of the Paris
Agreement.
o Creation of additional carbon sink of 2.5 to 3 billion tonnes of CO2
• It depicrs any country's national
equivalent through additional forest and tree cover.
action plans to limit the earth's
Additional Information average temperature rise to well
below two degrees Celsius and
• India created an additional carbon sink of 1.97 billion tonnes of CO2 preferably to 1.5 degrees Celsius as
equivalent between 2005 and 2019. compared to the pre-industrial
(19850-1900) levels.
• According to the 3rd national communication, the energy sector accounted • NDCs are submitted every five
for the maximum share of anthropogenic emissions (75.81 per cent), years to the UNFCCC secretariat.
followed by agriculture (13.44 per cent), Industrial Process and Product Use
(8.41 per cent), and waste (2.34 per cent).
• Title of the Report: "Migration and Development Brief"
• Released by: The World Bank
• Key Highlights
• Focus - Global remittance flows to low- and middle-income
countries (LMICs), with a detailed analysis of regional trends,
the impact of global economic conditions, and leveraging
remittances for development finance.
Top 5 recipient countries in 2023:
o India (USD 125 billion),
o Mexico (USD 67 billion),
o China (USD 50 billion),
o Philippines (USD 40 billion)
o Egypt (USD 24 billion).
Key Findings
• Growth rate of remittances to LMICs: Estimated 3.8%
increase in 2023, reaching USD 669 billion.
• Remittance flows by region:
o Growth in Latin America and the Caribbean (8%),
South Asia (7.2%), East Asia and the Pacific (3%), and
Sub-Saharan Africa (1.9%).
o Declines in the Middle East and North Africa (5.3%,
mainly due to Egypt) and Europe and Central Asia
(1.4%).
• The United States as the largest source of
remittances.
• Remittance costs: Averaging 6.2% to send USD 200;
banks as the costliest channel at 12.1%.
India Specific Ranking:
• India is the largest recipient of remittances:
USD 125 billion in 2023
• Increase from 2022's USD 111 billion.
• Expectation for 2024 - USD 135 billion
RELEVANT FOR
PHASE 1
• Winner: Haryana
• Runner Up: Rajasthan
• Player of the Match and Series: Sumit Kumar (Haryana
Team).
Key Facts
• It marks Haryana's first senior national trophy win in 32
years since the Ranji Trophy in 1991.

About Vijay Hazare Trophy


It began in the 1993–94 season but was played at zonal level only until
2002–03 when it was expanded to become a national competition.
In 2007, it was renamed in honour of Vijay Hazare.
Most Successful Team – Tamil Nadu
• Organizde by: Indian Air Force
• India becomes the first country to demonstrate this
capability.
• Key Facts
• India has demonstrated the firepower of its Akash
surface-to-air (SAM) weapon system during the Air
Force Exercise Astrashakti 2023.
• About Akash
• Indigenously designed and developed by the DRDO.
• It is a Short Range Surface to Air Missile System to
protect vulnerable areas and points from air attacks.
• Range of Operation: 4.5 km to 25 km
• A single Akash missile system engaged and
destroyed four unmanned aerial targets
simultaneously at around 30 km.
About SAMAR (Surface to Air Missile for Assured
Retaliation) Air Defence System
• Can engage aerial threats at speeds of 2 to 2.5 Mach.
• Features a twin-turret launch platform capable of
launching two missiles in single go, depending on the
threat scenario.
Aditya-L1 Solar Mission:
• Launched/Developed by: Indian Space Research
Organisation (ISRO).
• India's first solar mission, set to reach Lagrange Point 1
(L1) in the first week of January 2024.
• Objective: To observe the Solar corona and conduct in-situ
observations of the Solar wind at L1.
• The satellite will stay in Earth's orbit for sixteen days
before a 125-day journey to the Sun.

Recently, Solar Ultraviolet Imaging


Telescope (SUIT) instrument on the
Aditya-L1 spacecraft has successfully
L1 is about 1.5 million kilometres from the Earth between the planet and captured the first full-disk images of the
the Sun. Sun.
Top 3 States Pranav Soorma of Haryana
• Haryana bettered the Asian record in
• Uttar Pradesh the club throw event
• Tamil Nadu

About Khelo India Para Games


• Organized by: Ministry of Youth Affairs and Sports
• Mascot: 'Ujjwala'
• Participants: from 32 states and union territories, including the
Services Sports Control Board.
• Disciplines: Para athletics, para shooting, para archery, para
football, para badminton, para table tennis and para weight
lifting.
About Surat Diamond Bourse
• It is the world's largest and most modern centre for
international diamond and jewellery business.
• Aim: To shift the diamond trading business from Mumbai to
Surat, a hub for diamond cutting and polishing.
• Built inside the Diamond Research and Mercantile (DREAM)
City.
Key Features:
• Comprises a state-of-the-art ‘Customs Clearance House’ for
Import-Export activities.
• Recognized by Guinness World Records as the largest office
building in the world.
Additional Point
• Platinum ranking from the Indian Green Building Council
(IGBC).
Let’s Practice
GA Places in News

Q1. Which among the following statement is/are correct with respect to Surat
Diamond Bourse?
1. It is the world's largest and most modern centre for international diamond
and jewellery business.
2. It has received Platinum ranking from the Indian Green Building Council
(IGBC).
3. It aims to shift the diamond trading business from Mumbai to Surat, a hub
for diamond cutting and polishing.
A. Only 1
B. Only 2
C. 1 and 3
Answer: Option D
D. All of the above
E. None of the Above
GA First In News

Q2. Recently (December 2023), First edition of Khelo India Para games has
concluded in New Delhi. Which among the following event was part of the
event:
1. Para shooting
2. Para archery
3. Para football
4. Para Lawn Tennis
5. Para athletics
A. 2, 3, 4 and 5
B. 1, 2, 3 and 5
C. 1, 2, 3 and 4
Answer: Option E
D. 1, 2, 3, 4 and 5
E. 1, 2, 3 and 5
GA Defence

Q3. Recently (December 2023), India has demonstrated the firepower of its
Akash surface-to-air (SAM) weapon system during the Air Force Exercise
Astrashakti 2023. Which among the following statements is/are incorrect with
respect to the Akash System
A. It is indigenously designed and developed by the DRDO.
B. It is a Short Range Surface to Air Missile System to protect vulnerable
areas and points from air attacks.
C. It has a range of operation from 5 km to 25 km
D. A single Akash missile system engaged and destroyed four unmanned
aerial targets simultaneously at around 30 km.
E. All of the above are correct

Answer: Option C
Sustainable Development and
ESI Homework Question Environmental Issues

Q4. Consider the following statements about Nationally determined


contributions:
1. NDCs are at the heart of the Paris Agreement.
2. India has achieved the target to reduce the emissions intensity of its GDP
by 33 to 35 percent by 2030 from 2005 level.
3. India created an additional carbon sink of 1.97 billion tonnes of CO2
equivalent between 2005 and 2019.
A. Only 1
B. Only 2
C. 1 and 3
D. All of the above
E. None of the Above
ESI Homework Question Reports and Indices

Q5. Which among the following is not among the Top 5 recipient countries in
2023 as per the World Bank’s Migration and Development Brief?
1. Canada
2. India
3. Mexica
4. USA
5. China
A. Only 4
B. Only 1
C. 2, 3 and 5
D. 1 and 4
E. 1, 4 and 5
Topic Tag
PHASE I + II
New Swarnima Loan Scheme ESI – Government Schemes

RAMP Scheme ESI – Government Schemes

National Vehicle Scrappage Policy ESI – Sustainable Development and


Environmental Issues
PHASE I
National Sport Award 2023 GA – Awards
Noma GA – Science and Technology
Gaja Capital Business Book Prize GA – Awards
FIH Hockey Star Awards GA – Awards
One Nation One Corporate Card GA – Banking and Finance
RELEVANT FOR
PHASE 1 & PHASE 2
• Ministry – Ministry of Social Justice

• Implementing Agency: National Backward Classes Finance and


Development Corporation (NBCFDC).

• Objective: To foster self-dependence among women of Backward Classes


through Term Loans.

Eligibility:

• Women belonging to Backward Classes as notified by Central/State


Governments.

• Annual family income should be up to Rs. 3 Lakh.


Key Features:

• Maximum Loan Amount: Rs. 2 Lakh per beneficiary.

• Lower interest rates compared to general loan schemes.

Pattern of Financing:

• NBCFDC Loan: 95%.

• Channel Partner Contribution: 5%.

Interest Rate:

• NBCFDC to Channel Partner: 2% p.a.

• Channel Partner to Beneficiary: 5% p.a.

• Tenure: Loans to be repaid in quarterly installments within a


maximum of 8 years, including a six-month moratorium on
principal recovery.
What are the three sub schemes
Implemented by SIDBI

• MSME GIFT Scheme: It intends to help MSMEs adopt green technology with interest
subvention and credit guarantee support.

• MSE SPICE Scheme: First ever scheme in the Government to support circular
economy projects which will be done through credit subsidy and will lead to realising
the dream of MSME sector towards zero emissions by 2070.

• Implemented by - National Informatics Centre Services Inc.

• MSE Scheme on Online Dispute Resolution for Delayed Payments: First of its kind
scheme to synergise legal support with modern IT tools and Artificial Intelligence to
address the incidences of delayed payments for Micro and Small Enterprises.
About Raising and Accelerating MSME Performance (RAMP) Scheme:

o Launch: 2022

o Nodal Ministry: Ministry of Micro, Small and Medium Enterprises


(MoMSME)

o Monitoring: National MSME Council - headed by Minister for MSME

Aim:

o To Improve access to market and credit by Strengthening institutions


and governance at the Centre and State

o Improving Centre-State linkages and partnerships

o Addressing issues of delayed payments and greening of MSMEs

o To scale up implementation capacity and MSME coverage in States

• Tenure: 2021-22 to 2025-26

• Type of Scheme: Central Sector Scheme (assisted by World Bank)

• Financial Outlay: Rs.6,062.45 crore (USD 808 Million)

o World Bank - Rs.3750 crore (USD 500 Million)

o Government of India - Rs. 2312.45 crore (USD 308 Million)


Key Features
o In line with recommendations made by U K Sinha Committee, KV Kamath Committee and
Economic Advisory Council to Prime Minister (PMEAC)

o To support various COVID-19 Resilience and Recovery Interventions of Ministry of MSME

Policy Provider

o To enable the delivery of more effective and cost-efficient MSME interventions to improve
competitiveness and business sustainability.

Knowledge Provider

o Through bench-marking, sharing and demonstrating best practices/success stories by


leveraging international experiences

Technology Provider

o Access to high-end technology resulting in digital and technological transformation of


MSMEs

Access to Finance

• Strategic Investment Plans (SIPs) to be prepared with support of all states/UTs would act
as a roadmap for development of an improved MSME sector
About Digi ELV
• Launched/developed by: Rosmerta
Technologies
• DigiELV is a trading platform for End-of-Life
Vehicle (ELV) Certificate of Deposit.
• Recognized by: Ministry of Road Transport &
Highways.
• Objective of DigiELV: To enhance operational
efficiency, compliance, and sustainability in
vehicle scrapping.
RELEVANT FOR
PHASE 1
• Major Dhyan Chand Khel Ratna Award 2023: Chirag
Chandrashekhar Shetty and Rankireddy Satwik Sai Raj
(Badminton) Major Dhyan Chand Khel Ratna Award
given to sportsperson for spectacular
• Arjuna Award: 26 Recipients include Mohammed Shami, Aditi and outstanding sports performance
Gopichand Swami (Archery), Ojas Pravin Deotale (Archery) R over the past four years.
Vaishali (Chess), Diksha Dagar (Golf). Arjuna Award: To sportsperson for
• Dronacharya Award: R. B. Ramesh, coach of India's third consistent good performance over four
years, leadership, sportsmanship, and
female chess grandmaster.
discipline.
• Dhyan Chand Award for lifetime:Ms Manjusha Kanwa Dhyan Chand Award: For lifetime
(Badminton), Shri Vineet Kumar Sharma (Hockey), Ms Kavitha achievement in sports and continued
Selvaraj (Kabaddi) contribution post-retirement.
• Maulana Abul Kalam Azad (MAKA) Trophy Guru Nanak Dev Maulana Abul Kalam Azad (MAKA)
University Punjab) followed by Lovely Professional University Trophy: To the top-performing university
(Punjab) and Kurukshetra University (Haryana) in inter-university tournaments.
• Reason for addition
• Estimated 140,000 cases per year with a
prevalence of 770,000 cases (data from 1998).

About Naoma
• Noma, also known as cancrum oris or gangrenous stomatitis.
• It is a severe gangrenous disease of the mouth and face.
• Mortality rate of approximately 90%.
• Affected Demographic: Primarily affects children aged 2-6 years,
especially in poor communities in developing countries..
• Historical Context: Previously common in the Western world;
• Eradicated there with economic progress and improved
nutrition.
• Reported in European concentration camps during World War 2.
• Given By: Gaja Capital, a leading Indian private equity firm.
• Given To:
• 'Against All Odds: The IT Story of India’ - Kris Gopalakrishnan,
N Dayasindhu and Krishnan Narayanan
• 'Winning Middle India: The Story of India’s New-Age
Entrepreneurs’ - TN Hari and Bala Srinivasa

About Gaja Capital Business Book Prize Award


Established in 2019 to encourage Indian entrepreneurship and business
literature.
Offers the largest book prize in India with INR 15 lakh in prize money.
FIH Award for the Year 2023
• Player of the Year (Men): Hardik Singh (India)
• Player of the Year (Women): Xan de Waard (Netherlands)
• Goalkeeper of the Year (Men): Pirmin Black (Dutch)
• Goalkeeper of the Year (Women): Savita (India)
• Rising Stars (Men): Gaspard Xavier (FRA)
• Rising Stars (Women): Teresa Lima (ESP)

Hardik Singh also won the player of


the year title at the Hockey India
About FIH: Awards 2022.
Savita Punia had won the
• Establishment: The FIH was founded in 1924. recognition in 2021 and 2022 as
• Its primary role is to govern and develop the sport of hockey globally. well.
• Headquarters: Lausanne, Switzerland.
• Membership: 137 Nations
• Purpose:
• It digitizes business expenses for corporates and Small
and Medium Enterprises (SMEs).
• The card is designed to function as a National Transit
Card, facilitating travel throughout India on public
transport accepting bank-issued cards.
• The card can be used for business travel, small value
payments, branch expenses, etc., on any Merchant POS
(Point of Sale) device and online portals.
• Key Facts:
• The card is built on the National Common Mobility Card
(NCMC) specifications with offline capabilities.
• It supports the Government of India's vision of One Nation
One Card
Let’s Practice
GA Awards

Q1. Recently (December 2023), Gaja Capital Business Book Prize 2023 jointly
awarded to which among the following book?
1. Against All Odds: The IT Story of India
2. Time Shelter:India’s New Age IT Story
3. Winning Middle India: The Story of India’s New-Age Entrepreneurs
A. Only 1
B. Only 3
C. 1 and 3
D. All of the above
E. 2 and 3

Answer: Option C
GA Science and Technology

Q2. Recently (December 2023), Which among the following disease


has been added in the list of tropical neglected disease?
A. Noma
B. Dengue
C. Buruli ulcer
D. Chagas
E. Podoconiosis

Answer: Option A
GA Banking and Finance

Q3. Recently (December 2023), an innovative card, built on an


indigenously developed multi-wallet platform with Transit Issuance
and Processing Capability, serves as an Employee Tax Benefit
instrument. It also comes with a corporate expense management
platform for the employer to manage their expenses digitally. _____ a
Mumbai-based fintech entity specializing in online and offline
payments processing that has developed this card.
A. Paytm
B. Paycraft
C. Pay Pal
D. Razor Pay
E. All of the above Answer: Option B
GA Sports

Q4. Recently (December 2023), Savita Punia has won the women's
goalkeeper of the year award during the FIH Hockey Star Awards
2023.She has already won this award earlier in ______ and 2022.
A. 2017
B. 2021
C. 2020
D. 2019
E. 2018

Answer: Option B
ESI Government Schemes

Q5. Which among the following statement is/are correct about the
New Swarnima Loan Scheme?
1. It’s objective is to foster self-dependence among women of
Backward Classes through Term Loans.
2. Annual family income should be up to Rs. 3 Lakh.
3. Under the scheme interest rate from NBCFDC to beneficiary will
be 2% p.a.
A. Only 1
B. Only 2
C. Only 3 Answer: Option D
D. 1 and 2
E. 1 and 3
ESI Government Schemes

Q6. Recently (December 2023), Union Minister of MSME has


launched three sub schemes under the RAMP Scheme. Which among
the following statement is/are incorrect about the schemes?
1. MSME SPICE scheme intends to help MSMEs adopt green
technology with interest subvention and credit guarantee support.
2. MSE Scheme on Online Dispute Resolution for Delayed Payments
is implemented by SIDBI.
3. MSME Gift scheme is implemented by SIDBI.
A. 1 and 2
B. 1 and 3 Answer: Option D
C. 2 and 3
D. Only 2
E. 1, 2 and 3
ESI Homework Question Government Schemes

Q7. Which among the following statement is/are incorrect about the RAMP
Scheme?
1. It is in line with recommendations made by Urijit Patel Committee, KV
Kamath Committee and Economic Advisory Council to Prime Minister
(PMEAC)
2. It is a central sector scheme assisted by World Bank.
3. It is monitored by National and State MSME Council.
4. It enable the delivery of more effective and cost-efficient MSME
interventions to improve competitiveness and business sustainability
A. 1, 3 and 4
B. Only 1
C. 2 and 4
D. 1 and 3
E. 1, 2, 3 and 4
GA Homework Question AWARDS

Q8. Which among the following players and Awards is matched correctly as
per the National Sports Awards 2023
Player Award
1. Ojas Pravin Deotale Dhyan Chand Lifetime Achievemt Award
2. Manjusha Kanwa Arjuna Award
3. R.B. Ramesh Dhronacharya Award
4. Muhammad Shami Arjuna Award
A. 1 and 3
B. 2 and 4
C. 3 and 4
D. 1 and 2
E. 1, 3 and 4
Home Work Question from
previous lecture
Sustainable Development and
ESI Homework Question Environmental Issues

Q4. Consider the following statements about Nationally determined


contributions:
1. NDCs are at the heart of the Paris Agreement.
2. India has achieved the target to reduce the emissions intensity of its GDP
by 33 to 35 percent by 2030 from 2005 level.
3. India created an additional carbon sink of 1.97 billion tonnes of CO2
equivalent between 2005 and 2019.
A. Only 1
B. Only 2
C. 1 and 3
D. All of the above Answer: Option D
E. None of the Above
ESI Homework Question Reports and Indices

Q5. Which among the following is not among the Top 5 recipient countries in
2023 as per the World Bank’s Migration and Development Brief?
1. Canada
2. India
3. Mexica
4. USA
5. China
A. Only 4
B. Only 1
C. 2, 3 and 5
D. 1 and 4 Answer: Option D
E. 1, 4 and 5
Topic Tag
PHASE I + II
Palna Scheme ESI – Government Schemes

Mission Shakti Scheme ESI – Government Schemes

Energy Conservation Building Code (ECBC) ESI – Sustainable Development and


Environmental Issues
Public-Private Partnership (PPP) Beginner's ESI – Employment Generation In India
e-Course
PHASE I
Indian Ocean Naval Symposium GA – Defence
Khelo India Youth Games 2024 GA – Sports
RELEVANT FOR
PHASE 1 & PHASE 2
Palna National Creche Scheme:

• Objective: To address the urgent need for quality crèche care


facilities by providing a safe & secure environment for nutritional,
health, and cognitive development of the children, thereby
enabling more mothers to take up gainful employment.
• Monitoring compliance of the stipulations laid down under
Maternity Benefit Act
• Setting up of crèche facilities by the establishments
❑ To provide crèche facility to all mothers, irrespective of their
employment status
• Integrated package of the services:
o Day care facilities including Sleeping Facilities, Early Stimulation for children below 3
years and Pre-school Education for 3 to 6 years old children, Supplementary Nutrition
(to be locally sourced),
o Growth Monitoring and Health Check-up and Immunization in convergence with
Mission Poshan 2.0
• Crèche facility caters to children in the age group of 6 months to 6 years for 7.5 hours a
day.
o As the child stays for 7 and 1/2 hours in the crèche, three meals i.e. one
snack/breakfast in the morning, one meal (hot cooked) around noon and one snack in
afternoon may be provided.
• Crèches shall be open for 26 days in a month.
• Ideally number of children in a crèche facility should not be more than 25.
• It is ideal to have the crèche near the homes of children or near the place of work of the
mothers (at a walkable distance i.e. ½ - 1 km).
Mission Shakti: Approved as Umbrella Scheme in 2021-22

Nodal Ministry: Ministry of Women and Child Development

Aim: Strengthening interventions for women safety, security and empowerment

Objectives

• To provide to all women and girls in need of care and protection, with short
term and long-term services and information for their holistic development and
empowerment

• Provide assistance to women affected by violence and for those in distress

• To put in place quality mechanisms for rescue, protection and rehabilitation of


women in need of assistance and victims of crime and violence

• Create awareness among masses for inducing positive behavioural change


towards women and girls

• To prevent gender-biased sex selective elimination; to ensure survival,


protection,education and development of the girl child
• Tenure 2021-22 to 2025-26 during 15th Finance Commission period

• Financial Outlay: ₹20,989 crores (Central Share - ₹15,761 crore, State Share -
₹5228 crores)

Funding Pattern

• Sambal - 100% by Central Government

• Samarthya

❑ 60:40 between Centre and State Governments and UTs with legislature

❑ 90:10 - North East & Special Category States

❑ UTs without legislature, 100% -central government


Sub – Schemes Sambal - for safety and security of women

• One Stop Centre (OSC)

• Women Helplines (181-WHL)

• Beti Bachao Beti Padhao (BBBP)

New component of Nari Adalat - women collective

• Providing women with an alternate Grievance Redressal Mechanism

• Resolving cases of petty nature (harassment, subversion, curtailment of rights


or entitlements) faced by women at Gram Panachayat level and within families

• Formed of committed and socially respected women


Samarthya - for empowerment of women

• Shakti Sadan - Integrated Relief and Rehabilitation Home

❑ Swadhar Greh - for Women in difficult circumstances

❑ Ujjawala - for Prevention of Trafficking

❑ Unmarried girls of any age and boys up to 12 years allowed to stay in


Shakti Sadans with their mothers

❑ Women can stay upto a maximum period of 3 years (Extension can


be granted)

• Working Women Hostel (Sakhi Niwas)

• National Creche Scheme

• Pradhan Mantri Matru Vandana Yojana (PMMVY)


About Energy Conservation Building Code (ECBC)
• Launched by: Ministry of Power in 2007. • 20th Electric Power Survey Report:
• Developed by: Bureau of Energy Efficiency (BEE). o Prepared by the Central
• Implementation lies with the State Governments Electricity Authority.
• Objective: To promote energy efficiency in the commercial o It suggests that the combined
building sector. consumption of electricity in the
• Features: commercial and residential
categories is expected to reach
o ECBC sets minimum energy standards for new
around 40% of the total
commercial buildings having a connected load of 100 kW
electricity consumption by the
or contract demand of 120 kVA or more. year 2031-32.
• BEE launched a new version of ECBC in 2017 with a long-
term vision.
o One of the major updates to the code is inclusion of
incremental, voluntary energy efficiency performance
levels (Total 3: ECBC, ECBC Plus, Super ECBC).
About Public-Private Partnership (PPP) Beginner's e-Course

• Jointly Developed by: Infrastructure Finance Secretariat


(Under DEA, Ministry of Finance), and the World Bank.
• Objective: To provide foundational knowledge and insights
for individuals interested in PPPs in India.
• Duration: 7 hours and 15 mins but is designed to be self-
paced.

• Those taking up the course need not have any prior


experience in PPP to take this course
Key Features

Accessible Learning:
• Available online, the course ensures accessibility to a wide audience across the country.
• Self-paced modules accommodate diverse learning preferences and schedules.
Expert-Driven Content:
• Curated by industry experts and policymakers, the course content reflects current trends
and best practices in PPPs.
• Real-world case studies provide practical insights into successful PPP models.
Interactive Learning:
• Engaging multimedia elements, quizzes, and discussions foster interactive learning
experiences.
Certification:
• Upon completion of the e-Course, learners will receive a certificate recognizing their
proficiency in PPP fundamentals.
RELEVANT FOR
PHASE 1
• IONS was conceived by the Indian Navy in
• As a first, Flag designed by India was 2008.
selected as the IONS Flag. • Objective: An open and inclusive platform for
discussions on regionally relevant maritime
issues among Navies of the littoral states of the
Indian Ocean Region.
• Inaugural edition of IONS was held in Feb 2008
• India is also scheduled to take over as at New Delhi, with Indian Navy as the Chair for
the Chair of IONS (2025-27) during two years (2008 - 2010).
9th CoC planned to be conducted in
• With the inclusion of Korea, It has raised the
India in end 2025.
collective strength of IONS to 34 (25 Members
& 09 Observers).
• Mascot: 'Veeramangai.'
o The mascot represents legendary freedom fighter Rani Velu
Nachiyar, who is commonly referred to as 'Veera Mangai' in the
state of Tamil Nadu.
o 'Veera Mangai' stands for 'brave woman', as she was the first
Indian queen to go to war against the British-run East India
Company in India.

The first edition of KIYG, held in Delhi in 2018.


Let’s Practice
ESI Sustainable Development and Environmental Issues

Q1. Which of the following statements is true regarding the Energy


Conservation Building Code (ECBC)?
A. The ECBC was launched by the Ministry of Power in 2015 and aims to
promote energy efficiency in residential buildings only.
B. ECBC sets minimum energy standards for all types of buildings regardless
of their energy consumption or connected load.
C. The ECBC has been notified by all states and Union Territories in India as
of October 2023.
D. Under ECBC, new commercial buildings with a connected load of 100 kW
or contract demand of 120 kVA or more must meet minimum energy
standards.
E. The 20th Electric Power Survey Report, prepared by the Ministry of Power,
suggests that commercial and residential electricity consumption will
decrease by 2031-32.

Answer: Option D
GA SPORTS

Q2. The Khelo India Youth Games, a flagship initiative to promote


grassroots-level sports across the country, have previously been
hosted in Delhi, Pune, Guwahati, Panchkula, and Bhopal. The
forthcoming edition will unfold in four vibrant cities of Tamil Nadu –
Chennai, Trichy, Madurai, and Coimbatore. The mascot for the 6th
Khelo India Youth Games 2024, depicts which among the following
legendary freedom fighter?
A. Rani Durgavati
B. Kanniga Devi
C. Rani Velu Nachiyar
D. Rajakamalam
E. Parantaka

Answer: Option C
ESI Employment Generation In India

Q3. Which among the following statement is/are incorrect about the
Public-Private Partnership (PPP) Beginner's e-Course?
1. Those taking up the course need not have any prior experience in
PPP to take this course.
2. It’s objective is to provide foundational knowledge and insights for
individuals interested in PPPs around the world.
3. Jointly Developed by Infrastructure Finance Secretariat and the
World Bank
A. 1 and 2
B. 1 and 3
C. 1, 2 and 3
D. Only 3 Answer: Option A
E. Only 1
ESI Government Schemes

Q4. What is the tenure of Mission Shakti?


A. 2021-22 to 2026-27
B. 2020-21 to 2025-26
C. 2019-20 to 2024-25
D. 2022-23 to 2027-28
E. 2021-22 to 2025-26

Answer: Option E
ESI Homework Question Government Schemes

Q5. Which of the following statement(s) is/are correct with respect


to Palna – Creche facility?
1. Crèche facility caters to children in the age group of 6 months to 12
years.
2. Ideally number of children in a crèche facility should not be more
than 25.
3. It aims to provide crèche facility to all mothers, irrespective of their
employment status.
A. 1 and 2
B. 1 and 3
C. Only 2
D. Only 3
E. 2 and 3
Home Work Question from
previous lecture
ESI Homework Question Government Schemes

Q7. Which among the following statement is/are incorrect about the RAMP
Scheme?
1. It is in line with recommendations made by Urijit Patel Committee, KV
Kamath Committee and Economic Advisory Council to Prime Minister
(PMEAC)
2. It is a central sector scheme assisted by World Bank.
3. It is monitored by National and State MSME Council.
4. It enable the delivery of more effective and cost-efficient MSME
interventions to improve competitiveness and business sustainability
A. 1, 3 and 4
B. Only 1 Answer: Option C
C. 2 and 4
D. 1 and 3
E. 1, 2, 3 and 4
GA Homework Question AWARDS

Q8. Which among the following players and Awards is matched correctly as
per the National Sports Awards 2023
Player Award
1. Ojas Pravin Deotale Dhyan Chand Lifetime Achievemt Award
2. Manjusha Kanwa Arjuna Award
3. R.B. Ramesh Dhronacharya Award
4. Muhammad Shami Arjuna Award
A. 1 and 3
B. 2 and 4
C. 3 and 4 Answer: Option C
D. 1 and 2
E. 1, 3 and 4
Topic Tag
PHASE I + II
MedTech Mitra ESI – Social Sector in India Health

IEC Campaign ESI – Government Schemes

India Skills Report 2024 ESI – Reports & Indices

PHASE I
INS Imphal GA – Defence
National Consumer Rights Day GA – Important Days
RELEVANT FOR
PHASE 1 & PHASE 2
'MedTech Mitra': A Strategic Initiative to Empower MedTech
Innovators and Advance Healthcare Solutions

• MedTech Mitra is a platform that will help the young talents of the
country by holding their hands and giving them final shape to their
research, knowledge, logic etc. and help them in getting regulatory
approval.

• Facilitate indigenous development of affordable, quality MedTech


devices and diagnostics leading to considerable reduction in the
import dependence of this sector.
• MedTech Mitra will empower emerging start-ups and ensure ease
of innovation, ease of doing research and development, ease of
rendering service in building an Atmanirbhar Bharat.
CT 360 Session 15th & 16th Nov 2023
About the campaign

• Phase 1 Started initially in 100 districts

• Covering approximately 500 blocks and 15,000 PVTG habitations across 18 states and
the Union Territory of Andaman and Nicobar Islands

• In phase II, it will cover remaining districts.

• During the campaign period, Aadhar card, Community certificate and Jan Dhan
accounts will be provided
S.No. Interventions Ministry Number of Beneficiaries
Targets
Individual based interventions
1 Provision of pucca houses for beneficiaries Ministry of Rural 4.90 lakh HHs
Development (MoRD)
2&3 Energization of HHs Ministry of Power (MoP) 100000 household in
and Ministry of Non- 1500 habitations
Renewable Energy
(MoN&RE)
4 Piped water supply/Community water supply Ministry of Jal Shakti All PVTG households
(MoJS)
Community based interventions
5 Connecting roads Ministry of Rural 8000 Kms
Development
6 Mobile Medical Units with medicine cost Ministry of Health & Family 1000 (10/district)
Welfare
7 Construction of hostels Department of School 500
Education
8 Construction of Anganwadi Centres (AWCs) Ministry of Women & Child 2500
Development
9 Construction of Multipurpose Centres Ministry of Tribal Affairs 1000

10 Setting up of VDVKs Ministry of Tribal Affairs 500

11 Installation of mobile towers Department of All uncovered villages


Telecommunication
• Theme of report : "Impact of AI on the Future of Work, Skilling & Mobility"
• Two cities, Kochi and Thiruvananthapuram, in Kerala have emerged as the most preferred places to work
among the youth in India
• Employability in India has improved as 51.25% of the youth surveyed are employable with required skills.
• Kerala is the most preferred state for both men and women.
• Kochi is the first among the top 10 cities where most women prefer to work.
RELEVANT FOR
PHASE 1
• 2-destroyers of the same class INS
Visakhapatnam and INS Mormugao were
commissioned into the Navy in 2021 & 2022
• INS Imphal, a Project 15B stealth guided respectively.
missile destroyer, was commissioned into • 4th Project 15B stealth guided missile destroyer
the Indian Navy. ‘Surat’ will be commissioned in 2024
• Designed by: Indian Navy’s Warship Design
Bureau
• Constructed by: Mazagon Dock
Shipbuilders Limited (MDL), Mumbai
• National Consumer Rights Day
❑ The day is being celebrated every
year on December 24.
In 1986, the Consumer Protection Act was passed and received the assent of the President on
December 24.
Since then the day is celebrated on this day to commemorate the occasion.
The Consumer Protection Act acts to protect consumers from defective goods, negligent
services, and unfair trade practices.
The six fundamental rights of the Consumer Protection Act are:
• Right to safety, Right to choose, Right to be informed, Right to be heard, Right to seek
redressal and Right to consumer education.
• The day is celebrated on the 25th of
December every year to foster awareness
among the people of the country about
accountability in Governance and
Government.

Good Governance Day is being celebrated every year on the birth anniversary of former Prime
Minister Atal Bihari Vajpayee.
Let’s Practice
ESI Social Sector Health

Q1. Union Minister of Health and Family Welfare and Chemicals & Fertilizers
Dr. Mansukh Mandaviya virtually launches ‘___________': A Strategic Initiative to
Empower young Innovators and Advance Healthcare Solutions. It will help the
young talents of the country by holding their hands and giving them final
shape to their research, knowledge, logic and help them in getting regulatory
approval?
A. MedTech Mitra
B. MediTech Mitra
C. VedTech Mitra
D. HealthTech Mitra
E. SwasthTech Mitra

Answer: Option A
GA Important Days

Q2. National Consumer Rights Day is being celebrated every year on December
24.Which among the following is/are among six fundamental rights of the Consumer
Protection Act?
1. Right to safety
2. Right to choose
3. Right to be informed
4. Right to be heard
5. Right to seek redressal
6. Right to consumer education

A. 1, 2, 3, and 4 only
B. 1 and 3 only
C. 1, 2 and 3 only
Answer: Option E
D. All of the above except 5
E. 1, 2, 3, 4, 5, and 6
ESI Reports & Indices

Q3. Which of the following statement(s) is/are correct with respect to India Skills
report 2024?
1. It has been published by Wheebox in association with University Grants Commission
(UGC), Confederation of Indian Industry (CII), Association of Indian Universities
(AIU), Microsoft and Taggd.
2. Theme of report is "Impact of AI on the Future of Work, Skilling & Mobility".
3. Employability in India has improved as 51.25% of the youth surveyed are employable
with required skills.
4. Telangana is the state with the most employable resources among the 18-21 age
group, followed by kerala.
A. 1 and 2
B. 1 and 3
C. 1, 2 & 3
D. Only 1 & 4 Answer: Option E
E. 2, 3 and 4
ESI Homework Question Government Schemes

Q4. Consider the following statement with respect to “PM Janjati Adivasi Nyaya Maha
Abhiyan (PM-Janman)” launched recently:
1. It will work towards protection and nurturing of people living in metro politan cities
and the state capitals.
2. Government of India has allocated Rs 24000 crore for the scheme.
3. Under the mission, basic facilities such as road and telecom connectivity, electricity,
safe housing, clean drinking water and sanitation etc will be provided to the people.
Choose the correct statement?
A. Only 1
B. Only 2
C. Only 3
D. 2 and 3
E. All of the above
Home Work Question from
previous lecture
ESI Homework Question Government Schemes

Q5. Which of the following statement(s) is/are correct with respect


to Palna – Creche facility?
1. Crèche facility caters to children in the age group of 6 months to 12
years.
2. Ideally number of children in a crèche facility should not be more
than 25.
3. It aims to provide crèche facility to all mothers, irrespective of their
employment status.
A. 1 and 2
B. 1 and 3
C. Only 2
Answer: Option E
D. Only 3
E. 2 and 3
Topic Tag
PHASE I + II
PM Vishwakarma Yojna ESI – Government Schemes

Migration and Mobility Agreement ESI – Regional Economic Cooperation

PHASE I
International Day of Epidemic Preparedness GA – Important Days
‘Captain’ Vijayakanth GA – Person in News
MP's Tansen Festival GA – Fairs and Festivals
India and Malaysia MoU in Broadcasting GA – MoUs and Agreements
RELEVANT FOR
PHASE 1 & PHASE 2
Objective: to provide information about the assistance provided by
the Government of India for the overall development of traditional
artisans.

This scheme has three pillars: Sammaan, Samarthya and


Samriddhi.
Who is Vishwakarma? AIM

Providing benefits to Vishwakarmas, who are


An artisan or craftsperson working with hands and either self-employed or intend to setup their
tools and engaged in one of covered family-based own small-scale ventures
traditional trades, in unorganized sector on self-
employment basis

Covers artisans and craftspeople engaged in 18 preservation of cultural practices, generational


trades in rural and urban areas across India skills and guru-shishya parampara and providing
identity and recognition to them
Objectives

recognition of platform for


skill support for
artisans and easy access brand
upgradation better and
craftspeople to collateral promotion
to hone modern
as free credit and market
skills tools
Vishwakarma linkages

Credit Support
• Collateral free ‘Enterprise Development Loans’ of upto ₹3 lakh in 2 tranches of ₹1 lakh and ₹2 lakh
with tenures of 18 months and 30 months
❑ At concessional rate of interest fixed at 5%, with Government of India subvention to extent of 8%
❑ No prepayment penalty after 6 months of loan disbursement
Eligibility
• Minimum age: 18 years on date of registration
• Should be engaged in trade concerned on date of registration
• Should not have availed loans under similar credit-based schemes of Central
Government or State Government for self-employment/ business development e.g.
PMEGP, PM SVANidhi, MUDRA, in past 5 years

e-Shram database to be
primarily used for identification
of potential beneficiaries
National Steering
Committee - apex body
for implementation

Tenure:
Central Sector
FY 2023-24 to
Scheme
FY 2027-28

Target:
1st year - 5 lakh
families, total -
30 lakh families
over 5 years
• This Agreement will remain into force for a period of 5 years.

• Key provisions:
i. Indian students wishing to gather initial professional experience, after
completing academic/vocational training in Italy may be granted temporary
residence in Italy for up to 12 months.
ii. The Italian side has detailed provisions related to Professional Training,
Extracurricular Internships and Curricular Internships which allow Indian
students/trainees to gain experience in Italian skill/training standards.
iii. Under the Flows Decree, Italian side has offered incremental reserved
quotas for both seasonal and non-seasonal workers from 2023-2025.
RELEVANT FOR
PHASE 1
• Aim: to promote cooperation in public broadcasting in
the field of radio and television

With this, the total number of MoUs signed by Prasar Bharati with different countries has
increased to 46.
The DMDK made its electoral debut in the 2006 Tamil Nadu Legislative Assembly elections.
Let’s Practice
ESI Government Schemes

Q1. Which of the following statement is incorrect with respect eligibility under
PM Vishwakarma Scheme?
A. Minimum age should be 21 years on date of registration.
B. Person should be engaged in trade concerned on date of registration
C. Registration and benefits restricted to 1 member of family.
D. Person in government service and his/her family members is not eligible
E. None of the above

Answer: Option A
GA Regional Economic Cooperation

Q2. The Union Cabinet chaired by the Prime Minister has given its ex-post facto approval to
Migration and Mobility Agreement between the Government of the Republic of India and the
Government of the Italian Republic. Which among the following is/are the key provision under MoU?
1. Indian students wishing to gather initial professional experience, after completing
academic/vocational training in Italy may be granted temporary residence in Italy for up to 12
months.
2. Italian side has detailed provisions related to Professional Training, Extracurricular Internships
and Curricular Internships which allow Indian students/trainees to gain experience in Italian
skill/training standards.
3. Under the Flows Decree, Italian side has offered incremental reserved quotas for both
seasonal and non-seasonal workers from 2023-2025.

A. 1 and 2
B. 1 and 3
C. 2 & 3
D. Only 1 Answer: Option E
E. All of the above
ESI Important Days

Q3. The UN General Assembly invites all Member States, organizations of the United
Nations system and other global, regional and subregional organizations, the private
sector and civil society, including non-governmental organizations, academic institutions,
individuals and other relevant stakeholders to observe the International Day of Epidemic
Preparedness every year on _____________.
A. 25 December
B. 26 December
C. 27 December
D. 28 December
E. 29 December

Answer: Option C
ESI Homework Question Government Schemes

Q4. PM Vishwakarma’ aims to support traditional artisans and craftspeople. Which of


the following statement(s) is/are incorrect with respect to PM Vishwakarma scheme?
1. 2 types of skilling programmes - Basic and Advanced and a stipend of ₹500 per day
to be provided to beneficiaries while undergoing skill training.
2. Credit Support upto ₹2 lakh (1st Tranche) and ₹3 lakh (2nd Tranche) with a
concessional interest rate of 5% to be provided.
3. Total Financial Outlay of the scheme is ₹13,000 crore.

Choose the correct statement?


A. Only 1
B. Only 2
C. Only 3
D. 1 and 3
E. All of the above
Home Work Question from
previous lecture
ESI Homework Question Government Schemes

Q4. Consider the following statement with respect to “PM Janjati Adivasi Nyaya Maha
Abhiyan (PM-Janman)” launched recently:
1. It will work towards protection and nurturing of people living in metro politan cities
and the state capitals.
2. Government of India has allocated Rs 24000 crore for the scheme.
3. Under the mission, basic facilities such as road and telecom connectivity, electricity,
safe housing, clean drinking water and sanitation etc will be provided to the people.
Choose the correct statement?
A. Only 1
B. Only 2
C. Only 3 Answer: Option D
D. 2 and 3
E. All of the above
Performers
Topic Tag
PHASE I + II
16th Finance Commisiion ESI – Fiscal Policy

PHASE I
XPoSat mission GA – Science and Technology
First Sainik School for Girls GA – Defence/First in News
Maharishi Valmiki International Airport GA – Places in News
RELEVANT FOR
PHASE 1 & PHASE 2
Secretary to the Commission: Ritvik Ranjanam Pandey

Sixteenth Finance Commission:

❑ Determine the principles for grants-in-aid of state revenues from the


Consolidated Fund of India.
❑ Suggest measures to augment the Consolidated Fund to support
Panchayats and Municipalities.
❑ It will review financial arrangements for Disaster Management initiatives
under the Disaster Management Act, 2005.
• Submission of Report: 31st October 2025 covering a period of five
years commencing on 1st April 2026.
About Finance Commission
• Article 280
• Ministry: Ministry of Finance.
• Objective: To determine the method and formula for distributing tax proceeds
between the Centre and states and among the states as per constitutional
• 15th Finance
arrangements and present requirements. Commission
• Features: • Chairman: NK Singh.
• Constituted in:
❑ The Finance Commission is a constitutional body with a Chairman and four
November 2017.
members appointed by the President. • Cover the period from:
❑ It decides on tax distribution, grants-in-aid, and financial support to local bodies. April 1, 2020, to March
31, 2025.
❑ The Commission holds consultations with various stakeholders to finalize tax
distribution formulas for a five-year period.
❑ The President can refer any matter of financial importance to the Commission.
❑ Recommendations of the Finance Commission are taken into consideration by the
President under article 281 and laid before each House of Parliament, reflecting
its constitutional importance and impact on India's financial system.
RELEVANT FOR
PHASE 1
• Name of the Initiative: PSLV-C58 X-ray Polarimeter Satellite
(XPoSat) mission.
About XPoSat Satellite
• ISRO's first dedicated scientific satellite to conduct space-based
polarization measurements of X-ray emission from celestial
sources like black holes and neutron stars.
• Payloads: POLIX (developed by the Raman Research Institute)
and XSPECT (developed by the Space Astronomy Group of URSC,
Bengaluru).
• Mission Life: About 5 years

India has become second nation to launch an X-ray


polarimetry mission, following NASA’s IXPE (Imaging X-ray
Polarimetry Explorer).
• Name of the School: Samvid Gurukulam Girls Sainik School.
o Initiative of: Ministry of Defence and Uttar Pradesh
Government.
o Aim: To serve as a beacon for girls aspiring to join the Armed
Forces and serve the country
o It has approximately 120 seats.
o Training will be given by ex-servicemen.
o Institutional Capacity: To accommodate nearly 870 students.

Government’s broader initiative is to establish 100 new Sainik Schools in First Sainik School in Uttar
partnership with NGOs, private, and state government schools across all Pradesh was established in
states and union territories Lucknow in 1960
• Capacity: In the first phase, the airport can handle 10 lakh
passengers annually. After the second phase, it will cater
to 60 lakh commuters annually.
Let’s Practice
GA Science and Technology

Q1. Which of the following statements is true regarding the (XPoSat)


mission)?
A. It is India’s first dedicated scientific satellite to conduct space-based
polarization measurements of X-ray emission from celestial source.
B. India has become second nation to launch an X-ray polarimetry mission.
C. It’s Payload include Payloads: POLIX (developed by the Raman Research
Institute) and XSPECT .
D. A and B.
E. A, B and C

Answer: Option E
GA DEFENCE

Q2. Recently (December 2023), Union Minister of Defence along with


Chief Minister of Uttar Pradesh has launched the first sainik school for
girl to serve as a beacon for girls aspiring to join the Armed Forces
and serve the country.
First Sainik School in Uttar Pradesh was launched in which of the
following year?
A. 1950
B. 1970
C. 1955
D. 1960
E. 1991

Answer: Option D
ESI Fiscal Policy

Q3. Which of the following statements about the Sixteenth Finance


Commission are correct?
1. The Commission is expected to deliver its report by the 31st day of
October, 2025.
2. Dr. Arvind Panagariya has been appointed as the Chairman of the
Commission.
3. The Commission's recommendations are expected to commence from the
1st day of April, 2026.
4. The Finance Commission is a statutory body.
A. 1 and 3
B. 1, 2 and 3
C. Only 3
D. Only 1
Answer: Option B
E. 2 and 3
ESI Homework Question Fiscal Policy

Q4. Recently (December 2023), concluded 16th Finance commission


will work on which of the following measures?
1. Determine the principles for grants-in-aid of state revenues from
the Consolidated Fund of India.
2. Suggest measures to augment the Consolidated Fund to support
Panchayats, Blocks and Municipalities.
3. It will review financial arrangements for Disaster Management
initiatives under the Disaster Management Act, 2007.
A. Only 2
B. 1 and 3
C. Only 1
D. Only 3
E. 2 and 3
Topic Tag
PHASE I + II
Green Cover Index ESI – Human Development

Urban Square ESI – Government Schemes


(DAY-NULM, PM SVANidhi)
SMART 2.0 ESI – Social Sectors – Heath

PHASE I
World Braille Day GA – Days in News
Nadia Calvino GA – Appointment
Desert Cyclone GA – Defence
RELEVANT FOR
PHASE 1 & PHASE 2
• Agreement has signed for a period of three years to develop and report
a "Green Cover Index.”

About Green Cover Index:

• Ministry: Ministry of Road Transport and Highways


• Implementing Agency: NHAI in collaboration with National Remote
Sensing Centre (NRSC).
• Developed by: NRSC under ISRO.
• Aim/Objective: To create a comprehensive framework for the estimation
and continuous monitoring of green cover along National Highways.
• Green Highway Policy 2015
• Objective: To enhance the green cover along highways, reduce air pollution and dust,
and promote sustainable development with a focus on community involvement and
employment generation.
• Features:
• The policy dedicates 1% of the total project cost of all highway projects to plantation
and maintenance, amounting to about Rs. 1000 crore annually.
• Expected to generate employment for approximately five lakh rural individuals.
• Involvement of various stakeholders including farmers, private sector, and government
institutions for greening and developing eco-friendly National Highway corridors.
• Continuous monitoring and annual performance audits of executing agencies.
About Urban Square
• Ministry: Ministry of Housing & Urban Affairs.
• Aim/Objective: To foster urban empowerment and celebrate the
creativity and resilience of urban artisans and entrepreneurs.
• Features:
• Focus on marginalized urban communities.
• Integration of street vendors and urban self-help group members.
• Recreation of city chowks and public squares.
• Coverage: Nationwide, with exhibitors representing diverse regions of the
country.
Deendayal Antyodaya Yojana – National Urban Livelihoods Mission (DAY-NULM)
• Launch Year: 2013
• Nodal Ministry: Ministry of Housing and Urban Affairs
Objectives
• To reduce poverty and vulnerability of urban poor households by enabling them to access gainful self-
employment and skilled wage employment opportunities
• To provide shelters equipped with essential services to urban homeless
• To address livelihood concerns of urban street vendors by facilitating access to suitable spaces,
institutional credit, social security and skills to the urban street vendors
Funding pattern
• North Eastern and Himalyan states: 90:10, Other States: 60:40, UTs 100 : Nil
Key Features
• Replaced Swarna Jayanti Shahari Rozgar Yojana (SJSRY) and is also known as Rashtriya Shahri
Aajeevika Mission in Hindi
• Beneficiaries: Urban poor, including the urban homeless
Implementation
• All District Headquarter Towns and other cities with the population of 1,00,000 or more as per 2011
census
• States/ UTs are also allowed to implement all or any of the components of the Mission in any other
Statutory Town, based on the local capacity and requirement
• Interest subvention on bank loans is provided to beneficiaries through PAISA (Portal for
Affordable Credit and Interest Subvention Access) Portal [centralized electronic platform]
developed by Allahabad Bank in 2018
Process & Mechanism
• Employment through Skill Training and Placement with an expenditure of
❑ Rs. 15, 000 per person is allowed on training of urban poor
❑ Rs. 18,000 per person is allowed on training in North-East and J&K
• Social Mobilization and Institution Development
❑ At least 1-member from each urban poor household (preferably a woman) should be brought
under the Self-Help Group network
❑ City Livelihood Centres will be established in Mission cities
❑ Under Sub-Component – Building Community Institutions: SHGs and their Federations maximum
of Rs. 10,000/- can be spent per self-help group for its formation, handholding up to 2-years,
training of all the members, bank linkage, formation of federation and other related activities
❑ Interest Subsidy to urban poor to set up Individual micro-enterprises with loan Limit of Upto Rs
2 Lakh and Group enterprises with loan Limit: Upto Rs 10 Lakh
❑ Permanent all-weather 24 X 7 shelters for urban homeless
❑ Support to Urban Street Vendors
Pradhan Mantri Street Vendor’s Atmanirbhar Nidhi (PM-SVANIDHI)
▪ Launch – 2020
▪ Ministry of Housing and Urban Affairs
▪ Implementing Agency – Small Industries Development Bank of India (SIDBI)
▪ Objectives –
o Facilitate collateral free working capital loan up to ₹ 10,000 of 1 year tenure, with enhanced loan of
₹ 20,000 and ₹ 50,000 in 2nd and 3rd trances respectively, on repayment of earlier loans
o Incentivize regular repayment, through interest subsidy @ 7% per annum
o Reward digital transactions, by way of cash back up to ₹1,200 per year
▪ Central Sector Scheme
▪ Tenure – Till Decemeber 2024
▪ Target Beneficiaries – All street vendors in urban areas and surrounding semi-urban areas and rural
areas as on or before March 24, 2020
▪ Collateral free working capital loan up to ₹10,000, Enhanced loan of ₹20,000 & ₹50,000 in the 2nd and
3rd tranches respectively, on repayment of earlier loans
▪ No penalty on early repayment of loan
▪ Cashback of ₹ 1 per digital transactions subject to maximum of ₹ 100 in a month i.e. ₹ 1200 in a year
▪ SVANidhi Se Samriddhi – Launched in January 2021 to map the socio-economic profile of
the PM SVANidhi beneficiaries and their families
❑ Links beneficiaries’ families to existing 8 welfare schemes of Government of India,
targeting their holistic development and socio-economic upliftment
❑ These schemes are PM Jeevan Jyoti Bima Yojana, PM Suraksha Bima Yojana, PM Jan
Dhan Yojana, One Nation One Ration Card, PM Shram Yogi Maandhan Yojana,
Registration under Building and other Construction Workers (BoCW), Janani Suraksha
Yojana and PM Matru Vandana Yojana
About SMART 2.0
• Launch Body: Central Council for Research in Ayurvedic Sciences
(CCRAS) in collaboration with National Commission for Indian System of
Medicine (NCISM).
SMART 1.0' covered
• Objective: To promote robust clinical studies in Ayurveda through mutual
collaboration with academic institutions/hospitals across the country.
around 10 diseases
with active
• Research Focus: Pprioritizes research areas including Bal Kasa,
Malnutrition, Insufficient lactation, Abnormal Uterine Bleeding, participation from 38
Osteoporosis in post-menopausal women, and Diabetes Mellitus (DM) II. colleges.
• Implementing Agency: CCRAS, an apex organization for Ayurveda
research.
• Features:
o Focus on safety, tolerability, and adherence to Ayurveda
formulations.
o Collaboration with Ayurveda academic institutions/hospitals
nationwide.
o Provision for institutions to express interest in participation.
RELEVANT FOR
PHASE 1
• Theme of the Day: "Empowering Through Inclusion and
Diversity"

World Braille Day marks the birthday of Louis Braille, born in


1809 in France, the inventor of the Braille script.
• She is the the first woman and the first Spaniard to lead the institution.
• Predecessor: Werner Hoyer, who was president of the Bank since 2012
About Nadia Calvino
• She has chaired the International Monetary and Financial Committee
(IMFC) of the International Monetary Fund from 2022 to 2023.
• She has received prestigious awards including the US Foreign Policy
Association Medal, the Women and Technology Award, and the Master of
Digital Award.

• European Investment Bank was established in 1958.


• It is the lending arm of the European Union and is the biggest multilateral
financial institution in the world. It is owned by the EU Member States.
• 1st edition.

• Venue: Mahajan, Rajasthan, India.


• Key Facts:
• Type: Joint Military Exercise.
• Aim: To enhance interoperability in Sub-conventional Operations,
including Fighting in Built-Up Area (FIBUA) in desert/semi-desert
terrain under Chapter VII of the United Nations Charter on Peace
Keeping Operations.

• Chapter VII of the United Nations Charter pertains to "Action with respect to Threats to the Peace, Breaches
of the Peace, and Acts of Aggression.“
• It is a critical part of the Charter that outlines the UN Security Council's power to maintain peace and security.
Let’s Practice
ESI Human Development

Q1. Consider the following statements about the Green Highway Policy 2015 and choose the
correct combination of statements:
1. The policy aims to enhance green cover along highways and promote sustainable
development.
2. A total of 5% of the highway project cost is dedicated to plantation and maintenance
activities.
3. The policy is expected to generate employment for approximately five lakh rural
individuals.
4. Stakeholders involved in the policy include farmers, private sector, and government
institutions.
5. Continuous monitoring and annual performance audits are conducted for non-
governmental organizations only.
A. 1, 3, and 4
B. 2, 3, and 5
C. 1, 2, and 5 Answer: Option A
D. 1, 3, and 5
E. 2, 4, and 5
ESI GOVERNMENT SCHEMES

Q2. Consider the following statements about the Deendayal Antyodaya


Yojana – National Urban Livelihoods Mission (DAY-NULM) and choose the
correct combination.
1. The DAY-NULM was launched in 2013 to reduce poverty and vulnerability
of urban poor households.
2. The program is implemented in all cities with a population of 1,00,000 or
more as per the 2011 census.
3. Interest subvention on bank loans is provided to beneficiaries through the
PAISA Portal developed by the Reserve Bank of India in 2015.
A. 1 only
B. 1 and 2 only
C. 2 and 3 only
D. 1, 2, and 3
E. None of the above
Answer: Option B
ESI SOCIAL SECTOR - HEALTH

Q3. Regarding the SMART 2.0 initiative launched by the Central Council for
Research in Ayurvedic Sciences (CCRAS) in collaboration with the National
Commission for Indian System of Medicine (NCISM), identify the correct
statements?
A. The program aims to promote robust clinical studies in Homeopathy and is
implemented by an external agency.
B. SMART 2.0 focuses on research areas such as Malnutrition, Insufficient
lactation, and Diabetes Mellitus (DM) II, with CCRAS as the implementing
agency.
C. The initiative does not prioritize collaboration with Ayurveda academic
institutions/hospitals and mainly focuses on Allopathy.
D. SMART 1.0 covered around 20 diseases with active participation from over
50 colleges.
E. There is no provision for institutions to express interest in participation in
SMART 2.0.
Answer: Option B
GA APPOINTMENT

Q4. Which among the following statement is/are correct with respect to
Nadia Calviño recently (December 2023) seen in news?
A. She is the second woman and the first Spaniard to lead the institution she
has been appointed to.
B. She succeeded Werner Hoyer, who had been president of the Bank since
2018.
C. She has chaired the International Monetary and Financial Committee
(IMFC) of the International Monetary Fund from 2022 to 2023.
D. Among her accolades, she has received the European Financial
Management Association Medal and the Visionary Leadership Award.
E. She has received the US Foreign Policy Association Medal, the Women and
Technology Award, and the Master of Digital Award..

Answer: Option C
ESI Homework Question GOVERNMENT SCHEMES
Q5. Consider the following statements about the Pradhan Mantri Street Vendor’s
Atmanirbhar Nidhi (PM-SVANIDHI) and choose the correct statement:
1. The PM-SVANIDHI was launched in 2020 by the Ministry of Housing and Urban
Affairs, with SIDBI as the implementing agency.
2. It offers a collateral-free working capital loan up to ₹5,000 for the first tranche,
with enhanced loans on repayment.
3. The scheme provides an interest subsidy of 5% per annum to incentivize regular
repayment.
4. Digital transactions under the scheme are rewarded with a cashback of up to
₹1,200 per year.
5. SVANidhi Se Samriddhi was launched in January 2022 to map the socio-economic
profiles of PM-SVANIDHI beneficiaries.
[A] 1 and 4 only
[B] 2, 3, and 5 only
[C] 1, 4, and 5 only
[D] All are correct
[E] None are correct
GA Homework Question DEFENCE

Q6. Recently (December 2023), Defence exercise “Desert Cyclone”


has been concluded between India and UAE to enhance
interoperability in Sub-conventional Operations, including Fighting in
Built-Up Area (FIBUA) in desert/semi-desert terrain under Chapter VII
of the United Nations Charter on Peace Keeping Operations.
What is the primary focus of Chapter VII of the United Nations
Charter??
A. Outlining the roles and responsibilities of the UN General Assembly
B. Detailing the procedures for the admission of new UN members
C. Pertaining to "Action with respect to Threats to the Peace,
Breaches of the Peace, and Acts of Aggression"
D. Describing the financial regulations and contributions of UN
member states
E. Establishing the framework for international economic cooperation
Homework Question from the
previous session
ESI Homework Question Fiscal Policy

Q4. Recently (December 2023), concluded 16th Finance commission


will work on which of the following measures?
1. Determine the principles for grants-in-aid of state revenues from
the Consolidated Fund of India.
2. Suggest measures to augment the Consolidated Fund to support
Panchayats, Blocks and Municipalities.
3. It will review financial arrangements for Disaster Management
initiatives under the Disaster Management Act, 2007.
A. Only 2
B. 1 and 3
C. Only 1
D. Only 3 Answer: Option C
E. 2 and 3
Topic Tag
PHASE I + II
PRITHVI Scheme ESI – Government Schemes

SVAMITVA ESI – Government Schemes

PRASADAM ESI – Social Sectors – Heath

PHASE I
Pandit Hariram Dwiwedi GA – Persons in News
2023 Creative Arts Emmy Awards GA – Awards
RELEVANT FOR
PHASE 1 & PHASE 2
About PRITHvi VIgyan (PRITHVI)
• Ministry: Ministry of Earth Science
• Aim/Objective:
❑ Augmentation and sustainance of long-term observations of the
atmosphere, ocean, geosphere, cryosphere and solid earth to record the
vital signs of the Earth System and change
❑ Development of modelling systems for understanding and predicting
weather, ocean and climate hazards and understanding the science of
climate change
❑ Exploration polar and high seas regions of the Earth towards discovery of
new phenomena and resources;
❑ Development of technology for exploration and sustainable harnessing of
oceanic resources for societal applications
❑ Translation of knowledge and insights from Earth systems science into
services for societal, environmental and economic benefit.
• Tenure: 2021-26
• Budget Outlay: Rs. 4,797 crore

Five existing sub-schemes:

• ACROSS (Atmosphere & Climate Research-Modelling Observing Systems & Services) that focuses on long-term
observations and advanced modelling systems for weather and climate predictions.

• O-SMART (Ocean Services, Modelling Application, Resources and Technology) which targets sustainable
exploration and utilization of marine resources, utilizing cutting-edge technology

• PACER (Polar Science and Cryosphere Research) which aims to explore the mysteries of the Earth’s poles and
glacial regions, contributing to a deeper understanding of global climate changes.

• SAGE (Seismology and Geosciences) aiming to strengthen earthquake monitoring and research on the Earth’s
solid components

• REACHOUT (Research, Education, Training, and Outreach) which fosters talent development and ensures
effective translation of research into services directly benefiting society.
• Ministry of Panchayati Raj has been conferred the award for “Digital
Transformation Initiatives in Land Governance through SVAMITVA
Scheme.

Awards won by SVAMITVA


Gold Prize in National Awards for e-Governance 2023 in the category
“Use of Emerging Technology for Providing Citizen-Centric Services”
Gold award for the category “Innovative Use of Technology in e-
Governance for Digital Transformation” at Digitech Conclave 2023
organized at Goa
Survey of Villages and Mapping with Improvised Technology in Village Areas (SVAMITVA)
• Pilot Phase: 2020 in 9 States - Haryana, Karnataka, Madhya Pradesh, Maharashtra, Uttar Pradesh,
Uttarakhand, Punjab, Rajasthan & Andhra Pradesh
• Launch Nationwide:2021
• Nodal Ministry: Ministry of Panchayati Raj
• Technology Implementation Agency: Survey of India
• Objectives
❑ Creation of accurate land records for rural planning to reduce property related disputes and to bring
financial stability to citizens in rural India
❑ Determination of property tax
❑ Creation of survey infrastructure & GIS maps
❑ Support in preparation of Gram Panchayat Development Plan (GPDP) by use of GIS maps
• Type: Central Sector Scheme
• Target: To cover 6.62 Lakh villages of entire country from Financial Year 2020-21 to 2024-25
• Components
❑ Establishment of Continuous Operating Reference System (CORS) Network
❑ Large Scale Mapping using Drone
❑ Information, Education, Communication (IEC) Initiatives
❑ Enhancement of Spatial Planning Application “Gram Manchitra”
❑ Online Monitoring System
❑ Documentation Support/Workshops/Exposure Visits
Key Features
• Abadi area includes inhabitant land, inhabited areas contiguous to Abadi and wadis/basties
in rural areas and other Gram Panchayat and community assets like village roads, ponds,
canals, open spaces, school, Anganwadi, Health sub-centres, etc. would also be surveyed
and GIS maps would be created
• Implemented with collaborative efforts of Ministry of Panchayati Raj, State Revenue
Department, State Panchayati Raj Department and Survey of India (SoI) to provide ‘Record
of Rights (RoRs)’ to village household owners possessing houses in inhabited areas in
villages with issuance of legal ownership rights (Property cards/Title deeds)
• Preparation and distribution of Property Cards is responsibility of respective State
Government
• Memorandum of Understanding (MoU) would be signed between Survey of India and State
Government for implementation
• 4-layer Institutional Mechanisms for Implementation, Monitoring and Management:
National, State, District and Panchayat level
• SAHYOG application - mobile based application to capture ownership data of Property
owners
• SARATHI application - automates process of features/map correction, which is otherwise
done manually
About PRASADAM
• Venue: Ujjain, Madhya Pradesh
• Objective: To provide citizens with access to healthy and hygienic food.
• It offers convenient and culturally rich dining options for the 1-1.5 lakh
devotees who visit the Mahakaleshwar Temple daily.
• It is designed to provide various facilities including a kids' play area,
drinking water facility, CCTV surveillance, parking, public conveniences
and seating spaces.
Other Key Launches:
o Launch of the Mannhit app for mental health screening.
o Release of “The DART Book” by Food Safety and Standards Authority of
India (FSSAI) for checking common food adulterations at home.
o Introduction of a mobile food testing van, Food Safety on Wheels
(FSW), for awareness and adulteration testing.
RELEVANT FOR
PHASE 1
• Renowned Hindi and Bhojpuri poet, lyricist and litterateur.
• His noted poetry collections: "Anganaiya" and “Jeevandayini Ganga”..
• Held in Los Angeles (USA).
• Top Winners:
• The Last of Us: 8 awards (highest number of wins by a fiction
programme)
• Outstanding Actor in a Short Form Comedy or Drama Series: Tim
Robinson - I Think You Should Leave with Tim Robinson (Netflix)
• Outstanding Actress in a Short Form Comedy or Drama Series: Jasmine
Guy - Chronicles of Jessica Wu (Tubi)
• Outstanding Main Title Design: The Last of Us (HBO Max)
• Outstanding Motion Design: Ms. Marvel (Disney+)
• Notable Winners:
• Barack Obama won his second Emmy for best narrator for "Working:
What We Do All Day."
Let’s Practice
ESI Social Sectors - Health

Q1. Which of the following statements are correct about the 'PRASADAM'
initiative and other key launches related to health and food safety in India
recently in January 2024?
A. It was inaugurated in Indore, Madhya Pradesh.
B. The Ministry of Tourism launched 'PRASADAM' to promote traditional Indian
cuisine.
C. It aims to provide healthy and hygienic food.
D. The Mannhit app for mental health screening was launched by the Ministry
of Electronics and IT.
E. Food Safety on Wheels (FSW) is a mobile fast-food service introduced by
the Ministry of Health & Family Welfare.

Answer: Option C
GA PERSONS IN NEWS

Q2. Consider the following statements about Pandit Hariram Dwivedi and
choose the correct statement:
1. Pandit Hariram Dwivedi was known for his contributions to English and
Marathi literature.
2. His notable poetry collections include "Anganaiya" and "Jeevandayini
Ganga."
3. He was a renowned lyricist in the Tamil film industry.
4. His demise was a significant loss to the Hindi and Bhojpuri literary world.
A. 1 and 3
B. 2 and 4
C. 1, 2, and 3
D. 2, 3, and 4
E. 1, 3, and 4
Answer: Option B
GA AWARDS
Q3. Consider the following statements about the 75th Creative Arts Emmy
Awards 2023 and choose the correct combination:
1. "The Last of Us" won the highest number of awards in the non-fiction
category.
2. Barack Obama won his first Emmy for best narrator for "Working: What We
Do All Day."
3. "I Think You Should Leave with Tim Robinson" won Outstanding Motion
Design.
4. The award for Outstanding Actress in a Short Form Comedy or Drama
Series was won by Jasmine Guy for "Chronicles of Jessica Wu."
A. 1, 2, and 3
B. 1, 3, and 4
C. Only 4
D. 2, 3, and 4
E. 1 and 4 Answer: Option C
ESI GOVERNMENT SCHEME

Q4. Which of the following statements are true regarding the PRITHvi VIgyan
(PRITHVI) initiative?
1. The initiative is under the Ministry of Earth Science.
2. Its primary objective is to enhance industrial and urban development.
3. The program has a tenure from 2021 to 2026.
4. PRITHVI’s budget outlay is Rs. 4,797 crore.
5. One of its sub-schemes, O-SMART, focuses on urban technological
advancements.
A. 1, 3, and 4 only
B. 2 and 5 only
C. 1, 2, and 3 only
D. 3, 4, and 5 only Answer: Option A
E. All of the above
ESI Homework Question GOVERNMENT SCHEMES

Q5. Consider the following statements about the awards won by the
SVAMITVA scheme under the Ministry of Panchayati Raj:
1. SVAMITVA received the Silver Prize in National Awards for e-Governance
2023 for “Innovative Use of Technology in e-Governance”.
2. The scheme was awarded the Gold Prize in National Awards for e-
Governance 2023 in the category “Use of Emerging Technology for
Providing Citizen-Centric Services”.
3. It won a Gold award at Digitech Conclave 2023 in Goa for “Digital
Transformation Initiatives in Land Governance”.
A. 1 and 3 only
B. 2 and 3 only
C. 3 only
D. 2 only
E. All of the above
ESI Homework Question GOVERNMENT SCHEME

Q6. Identify the correct statements about the sub-schemes of PRITHvi


VIgyan (PRITHVI).
1. ACROSS focuses on advanced modelling systems for industrial weather
predictions.
2. PACER is dedicated to polar science and cryosphere research.
3. REACHOUT is aimed at developing healthcare services.
4. SAGE’s primary aim is to improve earthquake monitoring and geoscience
research.
[A] 2 and 4 only
[B] 1, 3, and 4 only
[C] 2, 3 and 4 only
[D] 3 and 4 only
[E] All of the above
Topic Tag
PHASE I + II
UNESCO World Heritage Committee ESI – Regional Economic Co-operation

PHASE I
Svyam Siddha GA – Government Initiatives
Ustad Rashid Khan GA – Persons in News
Golden Globes 2024 GA – Awards
Gabriell Attal GA – Appointment
RELEVANT FOR
PHASE 1 & PHASE 2
India will chair the committee for the first time in July 2024 during 46th session in New
Delhi.
• World Heritage Committee:
• Originated from the Conference of
o Meets annually. Allied Ministers of Education (CAME) in
o Responsible for: 1942 during World War II.
✓ Allocates financial assistance upon requests from States Parties • Initially focused on rebuilding
education systems post-war with a
✓ Inscriptions or deletions of properties on the World Heritage List universal approach.
and the List of World Heritage in Danger.
• Officially established in November
o Committee Membership: 1945 at the United Nations Conference
✓ Terms last six years, with most States Parties voluntarily serving (ECO/CONF) in London with 44
only four years for broader participation. countries participating
• Headquarters: Paris, France.
✓ Current 21 Member States include Argentina, Belgium, Bulgaria,
• Membership: 194 Members and 12
Greece, India, and several others.
Associate Members.
o Strategic Objectives - "Five Cs":
✓ Credibility, Conservation, Capacity-building, Communication,
Communities.
CORRIGENDUM

CT 360 – 20th and 21st December


RELEVANT FOR
PHASE 1
• About Svyam Siddha
o A specialized financial product by SBI for SHG women
entrepreneurs, offering loans up to Rs. 5 lakhs.
o Simplify loan process: Reduction in documentation, eliminating
the need for quotations and Detailed Project Reports (DPRs).
Launch of Training Toolkit Package: To
o DAY-NRLM's Role: Facilitate loan applications and oversee loan enhance capacities of State Rural Livelihoods
Missions (SRLMs) to increase access to formal
repayments through its field cadre. finance for SHG members.
Toolkit Developed with: International Finance
o Aligned with Vision of helping 2 Crore Lakhpati Didis from SHG Corporation (IFC) under World Bank-funded
women. National Rural Economic Transformation
Project (NRETP).
About Ustad Rashid Khan
• Renowned figure in Hindustani classical music.
• An exponent of the Rampur-Sahaswan gayaki, closely related to the Gwalior
gharana.
• He was known for his vilambit khayals and mastery in sargams and tarana.
• Awards: Padma Shri, Sangeet Natak Akademi Award, Padma Bhushan, Banga
Bibhushan, and Sangeet Mahasamman.

• Ustad Rashid Khan is a Great-grandson of Ustad Inayat Hussain Khan, founder of


Rampur Sehaswan gharana, and disciple of Bahadur Hussain Khan, a descendant
of Mian Tansen.
Venue: California, USA
Important Awards
o Christopher Nolan's "Oppenheimer" won:
✓ Best Motion Picture Drama
✓ Best Director (Christopher Nolan)
✓ Best Drama Actor (Cillian Murphy)
✓ Best Original Score (Ludwig Göransson)
✓ Best Supporting Actor (Robert Downey Jr.)
o Best Actress in a Dramatic Film: Lily Gladstone for "Killers of the Flower Moon".
✓ Notably, Gladstone is the first Indigenous winner in this category.
✓ She began her acceptance remarks by speaking in the Blackfeet language.
• Golden Globe Awards were held for the
first time in 80 years without the
involvement of the Hollywood Foreign
Press Association.
• Replaced Elisabeth Borne.
• Key Facts:
o Political Career and Achievements:
✓ Previously served as France's Minister of Education and National
Youth since July 2023.
✓ Bold decisions: Includes controversial ban on the wearing of the
Muslim Abaya in French public schools.
Let’s Practice
GA GOVERNMENT INITIATIVE

Q1. Which of the following statements regarding the recent MoU signed
between the Rural Development Ministry and State Bank of India (SBI) is/are
correct?
A. The MoU aims to facilitate Infrastructure Development for rural Self-Help
Group (SHG) women.
B. The financial product "Svyam Siddha" by SBI offers loans up to Rs. 10 lakhs
to SHG women entrepreneurs.
C. DAY-NRLM's role in the MoU includes facilitating loan applications and
overseeing loan repayments through its field cadre.
D. The Training Toolkit Package developed under the MoU is in collaboration
with the Asian Development Bank (ADB).
E. The vision of the MoU is to create 1 Crore Lakhpati Didis from SHG women.

Answer: Option C
GA PERSONS IN NEWS

Q2. Which of the following statements regarding Ustad Rashid Khan,


the renowned Hindustani classical musician, is/are correct?
[A] He was an exponent of the Kirana Gharana.
[B] He was awarded the Padma Shri, Sangeet Natak Akademi Award,
and Padma Bhushan for his contributions to Hindustani classical music.
[C] Ustad Rashid Khan was known for his fast-paced tempos.
[D] He is a grandson of Ustad Bismillah Khan .
[E] Ustad Rashid Khan was a direct descendant of Mian Tansen.

Answer: Option B
GA APPOINTMENT

Q3. Consider the following statements about Gabriel Attal's appointment


as the Prime Minister of France and identify the correct ones:
1. Gabriel Attal replaced Jean Castex as the Prime Minister of France.
2. He served as the Minister of Education and National Youth before his
appointment as Prime Minister.
3. One of his notable decisions as Minister was the implementation of
the controversial ban on wearing the Muslim Abaya in French public
schools.
A. Only 1
B. Only 2
C. Only 3
D. 2 and 3
E. 1 and 3 Answer: Option D
GA AWARDS

Q4. Consider the following statements about the 81st Golden Globe Awards
2024 and select the correct ones:
1. The Golden Globe Awards were held in New York, USA.
2. Oppenheimer won Best Motion Picture Drama.
3. Lily Gladstone won Best Actress in a Dramatic Film for "Killers of the
Flower Moon" and is the first Indigenous winner in this category.
4. The Hollywood Foreign Press Association played a significant role in the
81st Golden Globe Awards.
5. Cillian Murphy won Best Drama Actor for his role in "Oppenheimer".
[A] 1, 2, and 3 only
[B] 2, 3, and 5 only
[C] 2, 4, and 5 only
[D] 1, 4, and 5 only Answer: Option B
[E] 3, 4, and 5 only
ESI REGIONAL ECONOMIC COOPERATION

Q5. Which of the following statements is/are correct about UNESCO's World
Heritage Committee Session in India?
A. The 46th session of the World Heritage Committee will be held in Mumbai.
B. The World Heritage Committee is responsible for the allocation of the
World Education Fund.
C. The World Heritage Committee’s strategic objectives include Credibility,
Conservation, Capacity-building, Communication, and Communities.
D. UNESCO's headquarters is located in Geneva, Switzerland.
E. The World Heritage Committee's Committee Membership terms last for
four years.

Answer: Option C
ESI Homework Question REGIONAL ECONOMIC COOPERATION

Q6. Consider the following statements about UNESCO and the World Heritage
Committee and select the correct ones:
1. UNESCO was officially established in 1945 at the United Nations Conference in
London.
2. The World Heritage Committee meets biennially to discuss heritage conservation.
3. UNESCO's membership includes 194 Member States and 12 Associate Members.
4. India will be chairing the World Heritage Committee for the second time.
5. The main objective of UNESCO is to promote international cooperation in education,
arts, science, and culture.
[A] 1, 3, and 5 only
[B] 2, 4, and 5 only
[C] 1, 2, and 4 only
[D] 3, 4, and 5 only
[E] 1, 2, and 3 only
ESI Homework Question GOVERNMENT SCHEMES

Q6. Consider the following statements about the RAMP scheme and choose
the correct statement:
1. The RAMP scheme is a Central Sector Scheme aimed at improving access
to market and credit for MSMEs.
2. The scheme is assisted by the World Bank with a total financial outlay of
₹6,062.45 crore.
3. One of the key features of RAMP is to support various COVID-19 resilience
and recovery interventions.
4. The National MSME Council, which is headed by the Minister for MSME, is
responsible for the overall monitoring of the scheme.
[A] 1, 2, and 3 only
[B] 1, 3, and 4 only
[C] 2, 3, and 4 only
[D] 1, 2, and 4 only
[E] All of the above
Homework Question from the
previous session
ESI Homework Question GOVERNMENT SCHEMES

Q5. Consider the following statements about the awards won by the
SVAMITVA scheme under the Ministry of Panchayati Raj:
1. SVAMITVA received the Silver Prize in National Awards for e-Governance
2023 for “Innovative Use of Technology in e-Governance”.
2. The scheme was awarded the Gold Prize in National Awards for e-
Governance 2023 in the category “Use of Emerging Technology for
Providing Citizen-Centric Services”.
3. It won a Gold award at Digitech Conclave 2023 in Goa for “Digital
Transformation Initiatives in Land Governance”.
A. 1 and 3 only
B. 2 and 3 only
C. 3 only
Answer: Option B
D. 2 only
E. All of the above
ESI Homework Question GOVERNMENT SCHEME

Q6. Identify the correct statements about the sub-schemes of PRITHvi


VIgyan (PRITHVI).
1. ACROSS focuses on advanced modelling systems for industrial weather
predictions.
2. PACER is dedicated to polar science and cryosphere research.
3. REACHOUT is aimed at developing healthcare services.
4. SAGE’s primary aim is to improve earthquake monitoring and geoscience
research.
[A] 2 and 4 only
[B] 1, 3, and 4 only
[C] 2, 3 and 4 only
[D] 3 and 4 only
Answer: Option A
[E] All of the above
Topic Tag
PHASE I + II
Swachh Survekshan Awards 2023 ESI – Sustainable Development and Environment Issues

Startup Shala ESI - Employment Generation

PHASE I
Henley Passport Index 2024 Reports & Indices
MTHL Atal Setu GA – Place in News
National Youth Day GA – Days in News
Indian of the Year Award GA - Awards
RELEVANT FOR
PHASE 1 & PHASE 2
• Primary goal of Swachh Survekshan is to encourage
large scale citizen participation and create
awareness amongst all sections of society about the
importance of working together towards making
towns and cities better places to reside in.
• Conducted under the ambit of the Swachh Bharat
Mission (Urban)
• The theme for the year 2023 “Waste to Wealth”

• The theme for the year 2024 “Reduce, Reuse and Recycle”
• Swachh Bharat Mission Anthem for 2024 - Naya Sankalp Hai, Naya Prakalp Hai
Most Important Awards

Category Winner

All India Clean City Rank 1 (Population >1 Lakh) Indore (Madhya Pradesh) & Surat (Gujarat)

All India Clean City Rank 3 (Population >1 Lakh) Navi Mumbai (Maharashtra)

All India Clean City Rank 1 (Population <1 Lakh) Sasvad (Maharashtra)

All India Clean City Rank 2 (Population <1 Lakh) Patan (Chhattisgarh)

All India Clean City Rank 3 (Population <1 Lakh) Lonavala (Maharashtra)

Cleanest Cantonment Board Mhow Cantonment Board (Madhya Pradesh)

Best SafaiMitra Surakshit Sheher Chandigarh

Cleanest Ganga Town Rank 1 Varanasi (Uttar Pradesh)

Cleanest Ganga Town Rank 2 Prayagraj (Uttar Pradesh)


Evaluation criteria for the 4,477 participating cities
included factors such as-
• handling legacy dumpsites
• effective management of plastic waste
• implementation of the ‘reduce, reuse, recycle’
principles
• ensuring the well-being of Safaimitras
Startup India Innovation Week
• Organised by: Department for Promotion of Industry and Internal Trade (DPIIT),
Ministry of Commerce and Industry
• Duration: 10th-18th January 2024.
• ‘Startup Shala’ - Startup India’s flagship Accelerator Program was launched.
❑ Aim: extensive handholding of startups at the scaleup stage.
❑ The initiative is a 3-month long accelerator program for early-stage startups to provide
them access to knowledge, network, funds, or guidance required to scale up.
❑ Each program cohort will focus on a particular sector, the first one being the clean-tech
sector.

• Start Up India Initiative was launched by Government of India on 16th January


2016.
RELEVANT FOR
PHASE 1
• Henley Passport Index, which
ranks the world’s most travel-
friendly passports, has released its
list for 2024.
• Curated by Henley & Partners
• India, has improved its ranking and
moved up to the 80th position
from the previous year’s ranking of
84.
• Atal Bihari Vajpayee Sewri-Nhava Sheva Atal Setu (earlier: Mumbai
Trans Harbour Link)
o Built at cost of about Rs 17,840 crores.
o Location: Navi Mumbai (Maharashtra)
o Length: 21.8 km (16.5 km length over the sea and about 5.5 km
on the land)
o It is the longest bridge in India and also the longest sea bridge in
India.
• 12th of January 2024 marks the birth anniversary of Swami
Vivekananda, and is celebrated as National Youth Day.
• The Hon’ble Prime Minister inaugurated the 27th National Youth
Festival at Nashik Maharashtra on 12th January.
• “Indian of the Year Award" for the year 2023 in the category
‘Outstanding Achievement’ awarded to Team ISRO (Indian Space
Research Organisation).
• ISRO Chairman - S. Somnath
Let’s Practice
GA Places In News

Q1. Which of the following statements about the Atal Bihari Vajpayee Sewri-
Nhava Sheva Atal Setu is/are in-correct?

[A] It is the longest bridge in India, with a significant portion over the sea.
[B] The bridge is a 6-lane structure.
[C] The total cost of the bridge was less than Rs 10,000 crores.
[D] It is the longest sea bridge in the India.
[E] The bridge's length is about 21.8 km, with around 16.5 km over the sea and
5.5 km on the land.

Answer: Option C
ESI - Sustainable Development and
GA
Environment Issues

Consider the following statements regarding the Swachh Survekshan Awards 2023 and
select the correct ones.
1. Surat has been ranked first along with Indore in the All India Clean City Rank for cities with
a population of more than 1 lakh.
2. Maharashtra was ranked 1 in the category of “Best Performing States”.
3. The Swachh Survekshan Awards are a part of the Swachh Bharat Mission (Rural).
4. The primary goal of Swachh Survekshan is to facilitate industrial development in cities.
5. The theme for the year 2023 is "Waste to Wealth".

[A] 1 and 2 only


[B] 1, 2, and 5 only
[C] 2 and 4 only Answer: Option B
[D] 1, 2, and 3 only
[E] 3, 4, and 5 only
GA Awards

Q3. Recently, who was awarded the 'Indian of the Year Award' for the year
2023 in the category of ‘Outstanding Achievement’?

[A] The President of India


[B] Team ISRO (Indian Space Research Organisation)
[C] The Indian Cricket Team
[D] A notable Indian philanthropist
[E] An Indian Olympian athlete

Answer: Option B
GA Reports & Indices

Q4. Review the following statements about the Henley Passport Index 2024
and choose the correct ones.
1. France, Germany, Italy, Japan, Singapore, and Spain have the most
powerful passports according to the index.
2. The Henley Passport Index ranks countries based on their economic
strength.
3. India has improved its position to the 80th rank in the 2024 index.
4. The Henley Passport Index is curated by Henley & Partners.
5. Estonia was ranked last in the index.
[A] 1, 3, and 4 only
[B] 2, 3, and 5 only
[C] 1, 2, and 5 only Answer: Option A
[D] 3, 4, and 5 only
[E] All of the above
ESI Homework Question ESI - Employment Generation

Q5. Consider the following statements about the 'Startup Shala' initiative by
Startup India and identify the incorrect ones.
1. 'Startup Shala' is a one-year long incubation program for mid-stage
startups.
2. The program is designed to support startups at the scaleup stage with a
variety of resources, including access to knowledge and funds.
3. The first cohort of the program is focused on the clean-tech sector.

[A] Only 1
[B] Only 2
[C] Only 3
[D] 2 and 3
[E] 1 and 3
Homework Question ESI - Sustainable Development and
ESI
Environment Issues

Q6. Review the following statements about the Swachh Survekshan 2023
and identify the correct ones.

1.The Mhow Cantonment Board in Madhya Pradesh was declared the


Cleanest Cantonment Board.
2.Chhattisgarh was recognized as the 'Best Performing State' in Swachh
Survekshan 2023.
3.The components of Swachh Survekshan 2023 include Service Level
Progress, Certification, and Citizen Voice, contributing to the final score.

[A] Only 1
[B] Only 2
[C] Only 3
[D] 1 and 3
[E] 2 and 3
Homework Question from the
previous session
ESI Homework Question REGIONAL ECONOMIC COOPERATION

Q6. Consider the following statements about UNESCO and the World Heritage
Committee and select the correct ones:
1. UNESCO was officially established in 1945 at the United Nations Conference in
London.
2. The World Heritage Committee meets biennially to discuss heritage conservation.
3. UNESCO's membership includes 194 Member States and 12 Associate Members.
4. India will be chairing the World Heritage Committee for the second time.
5. The main objective of UNESCO is to promote international cooperation in education,
arts, science, and culture.
[A] 1, 3, and 5 only
[B] 2, 4, and 5 only
[C] 1, 2, and 4 only
Answer: Option A
[D] 3, 4, and 5 only
[E] 1, 2, and 3 only
ESI Homework Question GOVERNMENT SCHEMES

Q6. Consider the following statements about the RAMP scheme and choose
the correct statement:
1. The RAMP scheme is a Central Sector Scheme aimed at improving access
to market and credit for MSMEs.
2. The scheme is assisted by the World Bank with a total financial outlay of
₹6,062.45 crore.
3. One of the key features of RAMP is to support various COVID-19 resilience
and recovery interventions.
4. The National MSME Council, which is headed by the Minister for MSME, is
responsible for the overall monitoring of the scheme.
[A] 1, 2, and 3 only
[B] 1, 3, and 4 only
[C] 2, 3, and 4 only
[D] 1, 2, and 4 only Answer: Option B
[E] All of the above
Star Performers
Topic Tag
PHASE I + II
NITI Aayog Discussion Paper on ESI – REPORTS AND INDICES
Multidimension Povery
PM JANAM ESI – Government Schemes

PHASE I
Indian Metrological Department GA – Miscellaneous
One Vehicle One FasTag GA – Government Initiative
Indian Post Payment Bank GA – Banking and Finance
RELEVANT FOR
PHASE 1 & PHASE 2
• Headcount ratio was much faster between 2015-16 to 2019-21
(10.66% annual rate of decline) compared to period 2005-06 to 2015-
16 (7.69% annual rate of decline)
• According to World Bank’s Poverty and Inequality Platform poverty
headcount ratio decreased from 18.73% in 2015 to 11.9% in 2021 in
India.
❑ India is expected to reach single digit level of poverty by the year
2024-25

• The World Bank defines the international poverty line at USD 2.15 (in 2017 purchasing
power parity [PPP] terms) a day to measure monetary poverty.
Largest Decline Registered in:
1. Uttar Pradesh (5.94 crore people)
2. Bihar (3.77 crore)
3. Madhya Pradesh (2.30 crore)
• SDG Target 1.2 -Reduce at least 50% of men, women and children of all ages living in poverty
in all its dimensions by 2030
About Multidimensional Poverty Index
• Implementing Agency: NITI Aayog, with technical inputs from
Oxford Policy and Human Development Initiative (OPHI) and
United Nations Development Programme (UNDP).
• Developed by: The National MPI uses the Alkire Foster
methodology.
• It covers 12 indicators, in contrast to the global MPI's 10
indicators.
• Poverty Goals: NITI Aayog aims to reduce multidimensional
poverty to below 1%.
RELEVANT FOR
PHASE 1
Key Initiative launched
• Indigenously developed Decision Support System (DSS): Known as the Weather
Analysis and Forecast Enabling System (WAFES)
o To aid in decision-making for severe weather phenomena and their socio-
economic impact.
o It provides real-time information for various sectors such as Urban, Power,
Hydrology, Health, Energy, Agriculture, Transport and Tourism under the
"UPHHEATT" initiative.
• National Framework of Climate Services (NFCS): To enhance the utilization of
climate information in various sectors.
o IMD prepares the climatology of basic meteorological parameters and
updates in every 10 years.
• Panchayat Mausam Seva for Farmers: A collaborative effort between IMD, the
Ministry of Earth Sciences, the Ministry of Panchayati Raj, and Green Alert
Mausam Sewa.
o Provides weather forecasts in English, Hindi, and 12 regional languages to
every panchayat, aiding farmers in planning agricultural activities.
About India Meteorological Department
• Established in: 1875
• Ministry: Ministry of Earth Sciences.
• Headquarters: New Delhi
• Aim/Objective: To provide meteorological services for weather forecasting,
climate research, and protection of life and property against natural disasters.
• Objective of this Initiative: To discourage the use of a single FASTag for multiple vehicles
or linking multiple FASTags to one vehicle
About FASTag
• Launch Year: Introduced in 2014 and was made mandatory for every vehicle in 2019.
• Ministry: Ministry of Road Transport & Highways.
• Developed by: National Payments Corporation of India (NPCI).
• Implementing Agency: National Highways Authority of India (NHAI).
• Aim/Objective: To enhance the efficiency of the Electronic Toll Collection system and
ensure seamless movement at Toll Plazas.
• Technology Utilization: Radio Frequency Identification (RFID) technology, facilitating
direct toll payments from linked prepaid or savings accounts.
• Validity Period: 5 years.
About India Post Payments Bank
• Pilot Project Launch: 2017, in Ranchi and Raipur.
• Nationwide Launch: September, 2018.
• Ministry: Department of Posts, Ministry of Communication.
• Vision: To build the most accessible, affordable, and trusted bank for the common man.
• Mission: To spearhead financial inclusion by removing barriers and reducing costs for
accessing banking services.
• Features:
• Leverages India Stack for Paperless, Cashless, and Presence-less banking
• Offers banking solutions through intuitive interfaces in 13 languages
• Committed to promote a less cash economy and contributing to Digital India.
• Network: Comprises 155,000 Post Offices (135,000 in rural areas) and 300,000 Postal
employees.
• Headquarters: Located in New Delhi.
Let’s Practice
GA GOVERNMENT INITIATIVE

Q1. Consider the following statements about FASTag and identify the correct
ones as of January 2024.
1. FASTag was first introduced in 2014 and became mandatory for all vehicles
in 2017.
2. The technology used in FASTag is based on Radio Frequency Identification
(RFID).
3. The validity period of a FASTag is 3 years.
4. FASTag is implemented by the Reserve Bank of India (RBI).
[A] 1 and 2 only
[B] 2 and 3 only
[C] 2 only
[D] 1, 3, and 4 only
[E] All of the above Answer: Option C
GA PERSONS IN NEWS

Q2. Which of the following statements is/are true regarding India Post
Payments Bank (IPPB) as of January 2024?
[A] IPPB was launched nationwide in January 2017.
[B] The bank operates under the Ministry of Finance.
[C] IPPB's network includes 155,000 Post Offices, with 135,000 in rural
areas.
[D] Its headquarters is located in Mumbai.
[E] IPPB supports banking in 24 regional languages.

Answer: Option C
GA MISCELLANEOUS

Q3. Which of the following statements is true about the India


Meteorological Department (IMD) as of January 2024?
[A] IMD was established under the Ministry of Environment, Forest and
Climate Change.
[B] The MAUSAM App provides weather-related services only in English
and Hindi.
[C] IMD's Weather Analysis and Forecast Enabling System (WAFES) is part
of the "UPHHEATT" initiative.
[D] IMD was established in 1845 to provide meteorological services.
[E] The National Framework of Climate Services (NFCS) by IMD updates
its climatology data every 5 years.

Answer: Option C
ESI Homework Question REPORTS AND INDICES

Q4. Evaluate the following statements related to NITI Aayog's Discussion Paper on
Multidimensional Poverty in India and identify the correct ones as of January 2024.
1. The Multidimensional Poverty Index (MPI) in India covers 10 indicators of poverty.
2. Uttar Pradesh, Bihar, Madhya Pradesh, and Rajasthan reported the largest declines in
multidimensional poverty.
3. NITI Aayog aims to reduce multidimensional poverty in India to below 5%.
4. The national MPI uses the Alkire Foster methodology.
5. The poverty headcount ratio saw an annual decline of 10.66% between 2015-16 and
2019-21.
6. Sustainable Development Goal Target 1.2 aims to halve poverty in all its dimensions by
2030.
Options:
[A] 2, 4, 5 and 6 only
[B] 1, 3, and 6 only
[C] 2, 4, and 6 only
[D] 1, 2, and 4 only
[E] All of the above
ESI Homework Question GOVERNMENT SCHEMES

Q5. Evaluate the following statements related to the Pradhan Mantri Janjati Adivasi
Nyaya Maha Abhiyan (PM-JANMAN) as of January 2024 and identify the correct ones.
1. PM-JANMAN was launched on 15th November 2023, coinciding with Janjatiya
Gaurav Diwas.
2. The initiative is under the supervision of the Ministry of Rural Development.
3. The primary aim is to provide socio-economic welfare to all tribal communities in
India.
4. It includes 11 critical interventions covering areas like housing, education, and road
connectivity.
5. A budget of Rs 24,000 crores is allocated for the period from FY 2023-24 to 2025-
26 under the Development Action Plan for Scheduled Tribes (DAPST).
Options:
[A] 1, 3, and 5 only
[B] 1, 4, and 5 only
[C] 2, 4, and 5 only
[D] 1, 3, and 4 only
[E] All of the above.
Homework Question from the
previous session
ESI Homework Question ESI - Employment Generation

Q5. Consider the following statements about the 'Startup Shala' initiative by
Startup India and identify the incorrect ones.
1. 'Startup Shala' is a one-year long incubation program for mid-stage
startups.
2. The program is designed to support startups at the scaleup stage with a
variety of resources, including access to knowledge and funds.
3. The first cohort of the program is focused on the clean-tech sector.

[A] Only 1
[B] Only 2
[C] Only 3
[D] 2 and 3 Answer: Option D
[E] 1 and 3
Homework Question ESI - Sustainable Development and
ESI
Environment Issues

Q6. Review the following statements about the Swachh Survekshan 2023
and identify the correct ones.

1.The Mhow Cantonment Board in Madhya Pradesh was declared the


Cleanest Cantonment Board.
2.Chhattisgarh was recognized as the 'Best Performing State' in Swachh
Survekshan 2023.
3.The components of Swachh Survekshan 2023 include Service Level
Progress, Certification, and Citizen Voice, contributing to the final score.

[A] Only 1
[B] Only 2
[C] Only 3
[D] 1 and 3
[E] 2 and 3 Answer: Option D
Topic Tag
PHASE I + II
MPLADS ESI – Government Schmes

State Start Up Ranking 2022 ESI – Reports and Indices

PHASE I
Munawar Rana GA – Persons in News
R Praggnanandhaa GA - Sports
RELEVANT FOR
PHASE 1 & PHASE 2
With respect to work
• Flexibility in choosing works which can lead to
creation of durable public assets for larger
public good of society
• Not required to wait for actual fund to be released
by Ministry before recommending new projects
• Entire process of fund flow under revised
guidelines to operate through web portal
Financial Assistance
• Annual MPLADS fund entitlement per MP
constituency is ₹5 crore, released in 2-installments
of ₹2.5 crore each
• Funds released in grants in-aid directly to district
authorities
• Funds are non-lapsable

• MPs to recommend every year, works costing at least 15% of


MPLADS entitlement for year for areas inhabited by SC and
7.5% for areas inhabited by ST population
• If insufficient Scheduled Tribes population, fund may be utilized in
areas predominantly inhabited by Scheduled Castes and vice-
versa
• Funds not released by Government of India in a year carried
forward for making releases in subsequent years
Recommendation of Work
• Nominated Members of Lok Sabha and Rajya Sabha: Any 1 or more
Districts from any 1 State in Country
• Elected Lok Sabha Members: Within their Constituencies and Elected
Members of Rajya Sabha: Within State of Election
• Elected MP can also recommend works anywhere in country outside
usual region in which he/ she can recommend work
❑ Ceiling of ₹25 lakhs in a financial year per MP for all such
recommendations except in case of calamity
❖ Declared by Central Government: ₹1 Crore per annum for
rehabilitation and reconstruction works in areas affected by natural
in any part of country
❖ Declared by a State Government: (Lok Sabha Members from any
constituency in that State and any Rajya Sabha Members of State)
can consent up to maximum of ₹25 lakh per annum in affected
area(s) of State
• Minimum amount sanctioned for any individual work shall,
normally, be not less than ₹2.5 lakh
❑ If Implementing District Authority is of view that work of lesser amount
is beneficial to public, it may sanction same
• MPLADS funds can be used for repair and renovation of
immovable assets subject to condition that a MP can recommend
funds only upto ₹50 lakhs per year for all such works
• Can be pooled with individual/ stand-alone projects of other
Central/State schemes.

Works once recommended by MP cannot be changed by successor


About State Startup Ranking 2022
• Released by: Department for Promotion of Industry and
Internal Trade
• 1st edition: February 2018.
RELEVANT FOR
PHASE 1
• In 2014, he was honoured with the Sahitya Akademi
Award for his poem, 'Shahdaba’.
Recipient of Awards like:
• Ameer Khusro Award
• Mir Taqi Mir Award
• Ghalib Award
• Dr Zakir Hussain Award
• Saraswati Samaj Award.
• India Chess Wizard Rameshbabu Praggnanandhaa
defeated World Chess Champion Ding Liren in the
Tata Steel Masters in Wijk aan Zee, in the
Netherlands.
• As a result of this victory, Praggnanandhaa became
only the second Indian after Vishwanathan Anand to
defeat a reigning champion.
• He also overtook Anand to become India's new
number one Chess player in terms of live ratings.
Let’s Practice
ESI GOVERNMENT SCHEMES

Q1. Which of the following statements are correct regarding the MPLADS
scheme
1. The MPLADS was initially under the Ministry of Rural Development before
1994.
2. An MP can recommend works within their constituency and elected
Members of Rajya Sabha can recommend works within their State of
Election except in certain cases.
3. The minimum amount sanctioned under MPLADS for any individual work is
₹1.5 lakh.
[A] 1 and 2 only
[B] 2 and 3 only
[C] 1 and 3 only
[D] 1 only
Answer: Option A
[E] 1, 2, and 3
ESI GOVERNMENT SCHEMES

Q.2 Regarding the funding of MPLADS, identify the correct


statements:
1. The annual MPLADS fund entitlement per MP constituency is ₹25
crore.
2. Funds not released under the scheme in a year lapse and cannot
be carried forward.
3. MPLADS funds can be used for repair and renovation of
immovable assets.
[A] 1 only
[B] 2 only
[C] 3 only
[D] 1 and 3 only Answer: Option C
[E] 2 and 3 only
ESI REPORTS AND INDICES

Q3. Consider the following statements with respect to the State Start Up
Ranking 2022:
1. Himachal Pradesh is the best performing state under Category B.
2. Access to Market is one of the five reform areas.
3. First edition of the ranking was released in the year 2018.
Choose the correct one:
[A] 1 and 2 only
[B] 3 only
[C] 2 and 3 only
[D] 1 and 3 only
[E] 1, 2, and 3

Answer: Option D
ESI Homework Question REPORTS AND INDICES

Q4. Consider the following statements regarding the MPLADS:


1. State Nodal Authorities are required to inspect a minimum of 5% of
works by value in every district each year.
2. MPs must recommend works costing at least 15% of MPLADS
entitlement for areas inhabited by SC and ST population.
3. The maximum amount allowed for a constituency to consent to
works beyond their region is ₹50 lakhs per annum.
Choose the correct one:
[A] 1 and 2 only
[B] 3 only
[C] 2 and 3 only
[D] 1 and 3 only
[E] 1, 2, and 3
Topic Tag
PHASE I + II
Alliance for Global Good- Gender Equity and ESI – Gender Issues
Equality
'Pradhan Mantri Suryodaya Yojana ESI – Government Schemes

PHASE I
Japan’s SLIM GA – Science & Technology
New DG of SSB GA - Appointments

Parakram Diwas GA – Important Days

Exercise Khanjar GA – Defence


RELEVANT FOR
PHASE 1 & PHASE 2
➢ World Economic Forum’s 54th Annual Meet: Davos,
Switzerland

➢ WEF theme for the 2024: 'Rebuilding Trust’.

Key takeaway-
• The alliance has been launched with the support and
endorsement by WEF and Government of India.

• The idea of this alliance emerged from the G20 Leaders’


Declaration and India’s abiding commitment to the cause of
women-led development.

• Objective: to bring together global best practices, knowledge


sharing and investments in the identified areas of women’s
health, education, and enterprise.
• Supported by the Bill and Melinda Gates Foundation,
the alliance will be housed and anchored by the CII Centre
for Women Leadership.

• The World Economic Forum has come on board as a


‘Network Partner’ and Invest India as an ‘Institutional
Partner’.

• ‘Alliance for Global Good- Gender Equity and Equality’ is


poised to be a force to reckon with on all gender related
issues.

• This year’s unique initiative at WEF, led by Union


Minister of Women & Child Development was a first ever
women leadership lounge (We Lead lounge) by CII and
BMGF.
❑ Aim: to provide electricity to low and middle-income individuals
through solar rooftop installations, along with offering additional
income for surplus electricity generation.

❑ Target: To install rooftop solar on 1 crore houses

❑ This scheme will provide rooftop solar panels for consumers.

❑ It will allow every household with a roof to reduce their electricity bills
and to make them truly aatmanirbhar for their electricity needs.
RELEVANT FOR
PHASE 1
• The 1st pilot project of making aviation turbine fuel
from Alcohol, has been inaugurated.
• Set up at: Praj Industries Ltd Research and Development
unit in Pirangut near Pune, Maharashtra
• Sustainable Aviation Fuel (SAF) would be produced in
this project.

• Air Asia India, successfully operated the country’s first commercial flight on
the Pune-Mumbai sectors using an indigenously produced sustainable
aviation fuel (SAF) blend.
• Japan became the fifth country in history to reach the moon when its
spacecraft landed on the lunar surface.
❑ Japan follows the United States, the Soviet Union, China and India in
reaching the moon.
• SLIM, nicknamed “the Moon Sniper,” was aiming to hit a very small target,
is a lightweight spacecraft about the size of a passenger vehicle.
❑ It was using “pinpoint landing” technology that promises far greater
control than any previous moon landing.

• As per JAXA (Japan Aerospace Exploration Agency), the mission’s main


goal is to test new landing technology that would allow moon missions
to land at any place, rather than where it is easy to land.
• Senior IPS officer Daljit Singh Chaudhary has been
appointed as the Director General of the Sashastra
Seema Bal (SSB) for a period up to November 30,
2025.

• The SSB guards India's frontiers with Nepal and Bhutan.


• Rohit Sharma has been named captain of the ICC ODI
Team of the Year, which featured five other Indians

• Pat Cummins (Australia) has been named as the captain of


the ICC Men’s Test Team of the Year.
• Ravindra Jadeja and R Ashwin are the only Indian players
to be included in the test team.

• The SSB guards India's frontiers with Nepal and Bhutan.


• Parakram Diwas marks the birth anniversary of Netaji
Subhas Chandra Bose.
• The government announced in 2021 that January 23
will be observed as Parakram Diwas (or the Day of
Valour) in order to pay tribute to Netaji's relentless
pursuit of India's freedom.
• Bharat Parv, a nine-day event (January 23 and 31) has
been launched by PM to showcase the country's rich
diversity and exhibit different cultures.
• The 11th edition of India-Kyrgyzstan Joint Special
Forces Exercise “KHANJAR” has commenced.
• Venue: Bakloh, Himachal Pradesh.
• It is an annual event conducted alternatively in both
the countries.
Let’s Practice
ESI GOVERNMENT SCHEMES

Q1. Consider the following statements about the 'Pradhan Mantri Suryodaya
Yojana':
1. The scheme's aim is to provide electricity to high-income individuals
through solar rooftop installations.
2. The target of the scheme is to install rooftop solar on 1 crore houses.
3. Households can earn additional income for surplus electricity generation
under this scheme.
Which of the above statements is/are correct?
[A] 1 and 2 only
[B] 2 and 3 only
[C] 1 and 3 only
[D] 2 only
[E] 1, 2, and 3 Answer: Option B
GA Defence

Q.2) Consider the following statements regarding the India-


Kyrgyzstan Joint Special Forces Exercise "KHANJAR":
1. The exercise "KHANJAR" is held biennially between India and
Kyrgyzstan.
2. The 11th edition of the exercise is being conducted in Bakloh,
Himachal Pradesh, India.
3. This exercise is held in the same location every year.
Which of the above statements is/are correct?
[A] 1 and 2 only
[B] 2 only
[C] 2 and 3 only
[D] 1 and 3 only
Answer: Option B

[E] 1, 2, and 3
ESI Gender Issues

Q3. Consider the following statements related to the World Economic


Forum’s 54th Annual Meeting:
1. The theme for the WEF Annual Meeting 2024 was 'Strengthening
Economies'.
2. The "Alliance for Global Good- Gender Equity and Equality" was launched
with the support of the WEF but not the Government of India.
3. The alliance aims to pool global best practices and resources for
improving women’s health, education, and entrepreneurship.
Which of the above statements is/are correct?
[A] 1 only
[B] 1 and 2 only
[C] 3 only
[D] 2 and 3 only Answer: Option C
[E] None of the above
GA Important Days

Q4. Which day is observed as Parakram Diwas in India to honor the birth
anniversary of Netaji Subhas Chandra Bose?
[A] January 23
[B] January 26
[C] August 15
[D] October 2
[E] December 25

Answer: Option A
ESI Homework Question Gender Issues

Q5. Consider the following statements concerning the "Alliance for Global
Good- Gender Equity and Equality":
1. The alliance aims to bring together global best practices, knowledge
sharing and investments in the identified areas of women’s health,
education, and enterprise.
2. The concept of the alliance was inspired by the Paris Agreement on
climate change.
3. The World Economic Forum is the 'Institutional Partner' for the alliance.
Which of the above statements is/are correct?
[A] 1 only
[B] 1 and 2 only
[C] 1 and 3 only
[D] 2 and 3 only
[E] All of the above
ESI Homework Question Previous Government Schemes
Session

Q4. Consider the following statements regarding the MPLADS:


1. State Nodal Authorities are required to inspect a minimum of
5% of works by value in every district each year.
2. MPs must recommend works costing at least 15% of MPLADS
entitlement for areas inhabited by SC and ST population.
3. The maximum amount allowed for a constituency to consent
to works beyond their region is ₹50 lakhs per annum.
Choose the correct one:
[A] 1 and 2 only
[B] 3 only
[C] 2 and 3 only
[D] 1 and 3 only
[E] 1, 2, and 3
Topic Tag
PHASE I + II
Scheme for promotion of Coal/Lignite ESI – Government Schemes
Gasification Projects
PHASE I
BCCI Awards GA – Awards
EX Desert Knight & Cyclone GA - Defence

Jan Nayak Karpoori Thakur GA – Person in News

National Voter Day & other important days GA – Days in News


RELEVANT FOR
PHASE 1 & PHASE 2
➢ Aim: To produce synthetic natural gas, dimethyl ether, and
ammonium nitrate from coal. Gasification of coal is a process in which coal is
partially oxidated by air, oxygen, steam or carbon
➢ Allocation: Rs.8,500 crore dioxide under controlled conditions to produce a
fuel gas.
➢ The amount will be provided as financial assistance for coal
gasification projects under three categories
Category – 1

• Rs.4,050 crore provisioned for Government PSUs in which upto


3 projects will be supported by providing lump-sum grant of
Rs.1,350 crore or 15% of capex, whichever is lower.

Category – 2

• Rs.3,850 crore provisioned for private sector as well as


Government PSUs in which lump-sum grant of Rs.1,000 crore
or 15% of capex, whichever is lower provided for each project.

• At least one project will be bid out on tariff-based bidding


process and its criteria will be designed in consultation with
NITI Aayog.
Category – 3

• Rs.600 crore provisioned for demonstration Projects (indigenous


technology) and/or small-scale product-based Gasification Plants under
which lump-sum grant of Rs.100 crore or 15% of capex, whichever is Nm3/h (normal cubic meters per
lower. hour) is a measure of gas flow
rate
• Grant will be given to the selected entity who will have a minimum Capex
of Rs. 100 Crore and minimum production of 1500 Nm3/hr Syn gas.

The grant will be paid to


the selected entity in two
equal instalments.
RELEVANT FOR
PHASE 1
• The Board of Control for Cricket in India (BCCI) announced its first
post-Covid awards, honouring winners for the last four seasons in
BCCI Awards
domestic and international cricket.
• Conducted between: Indian Air Force (IAF), French Air and
Space Force (FASF), United Arab Emirates (UAE) Air Force
• The exercise in Indian FIR was conducted over the Arabian
Sea

• Defence exercise between India & France:


❑ Varuna: Naval
❑ Garuda: Air
❑ Shakti: Army
• 2nd edition of India-Egypt Joint Special Forces
Exercise CYCLONE
• Conducted at Anshas, Egypt from 22nd January to 1st
February 2024
• Karpoori Thakur, a prominent figure in Bihar's OBC
politics and two-time chief minister, has been
posthumously honored with the Bharat Ratna, India's
highest civilian award, on the eve of his birth centenary.
• He was also known as 'Jan Nayak’.
• Today, 250 million children and adolescents are out of school and
763 million adults are illiterate, according to UNESCO.
• The right to education is enshrined in article 26 of the Universal
Declaration of Human Rights

• National Girl Child Day is observed countrywide with the


aim of engaging the nation in creating a positive
environment that values the girl child.

• The day promotes equal opportunities in education,


healthcare, and nutrition while raising awareness of the
injustices that girls face.
• On January 25, 2024, the Election Commission of India (ECI) celebrates
its 75th year of service to the Nation.
• A short voter awareness film - ‘My Vote My Duty’ produced by ECI
with renowned filmmaker Raj Kumar Hirani
• Commemorative postage stamp themed "Inclusive Elections"

• Best Electoral Practices Awards for the year 2023 will be presented to State
and District level officers for their outstanding performance in the conduct
of elections during 2023
Let’s Practice
ESI GOVERNMENT SCHEMES
Q1. Consider the following statements about the scheme approved by the Cabinet chaired by
Prime Minister Shri Narendra Modi for the promotion of Coal/Lignite Gasification Projects:
1. The total outlay of the scheme is Rs. 8,500 crore, divided into three categories for financial
assistance.
2. In Category I, up to three Government PSU projects will receive a lump-sum grant of Rs.
1,350 crore each.
3. Category II includes private sector as well as Government PSUs projects with a provision of
Rs. 3,850 crore, where each project will receive a grant of Rs. 1,000 crore or 15% of capex.
4. For Category III, a minimum Capex of Rs. 200 Crore is required for eligibility.
5. The grant will be paid in three equal instalments to the selected entity.
Choose the correct combination of statements:
[A] 1, 2, and 3
[B] 2, 3, and 5
[C] 1, 3, and 4
Answer: Option A
[D] 1, 2, and 5
[E] All of the above
GA Important Days

Q.2) Which of the following statements is/are correct regarding the


National Voters' Day (NVD) 2024 celebrations by the Election
Commission of India (ECI)?
[A] The ECI celebrated its 50th year of service to the Nation on NVD
2024.
[B] The voter awareness film produced by the ECI was titled ‘Our Vote,
Our Future’.
[C] A commemorative postage stamp released on NVD 2024 was
themed "Inclusive Elections".
[D] The theme for NVD 2024 was ‘Vote for a Better Tomorrow’.
[E] None of the above

Answer: Option C
GA Person in News

Q3. Who was posthumously honored with the Bharat Ratna, India's highest
civilian award, on the eve of their birth centenary? He was a significant
figure in Bihar's OBC politics and a two-time chief minister.
[A] Dr. Rajendra Prasad
[B] Karpoori Thakur
[C] Lok Nayak Jayaprakash Narayan
[D] Bhagwan Birsa Munda
[E] Ramdhari Singh Dinkar

Answer: Option B
GA Defence

Q4. Which countries participated in the 2nd edition of the Joint Special Forces
Exercise CYCLONE, and where was it conducted?

[A] India and Russia at Anshas, Egypt


[B] India and Egypt at Anshas, Egypt
[C] India and the United States at Cairo, Egypt
[D] India and Egypt at Cairo, Egypt
[E] India and Russia at Cairo, Egypt

Answer: Option B
ESI Homework Question Gender Issues
Previous Session

Q. Consider the following statements concerning the "Alliance for Global


Good- Gender Equity and Equality":
1. The alliance aims to bring together global best practices, knowledge
sharing and investments in the identified areas of women’s health,
education, and enterprise.
2. The concept of the alliance was inspired by the Paris Agreement on
climate change.
3. The World Economic Forum is the 'Institutional Partner' for the alliance.
Which of the above statements is/are correct?
[A] 1 only
[B] 1 and 2 only
[C] 1 and 3 only Answer: Option A
[D] 2 and 3 only
[E] All of the above
Performers
ESI Homework Question Government Schemes

Q. Consider the following statements regarding the scheme for the promotion of
Coal/Lignite Gasification Projects approved by the Cabinet chaired by Prime Minister Shri
Narendra Modi:
1. The scheme provides a total financial assistance of Rs. 8,500 crore, specifically
targeting coal gasification projects.
2. In Category II of the scheme, private sector projects are exclusively eligible for a lump-
sum grant of Rs. 1,000 crore or 15% of capex, whichever is lower.
3. For Category III, the projects need to have a minimum production capacity of 1500
Nm3/hr Syn gas to be eligible for a grant of Rs. 100 crore or 15% of capex, whichever is
lower.
Choose the correct combination of statements:
[A] 1 and 2 only
[B] 2 and 3 only
[C] 1 and 3 only
[D] All of the above
[E] None of the above
Topic Tag
PHASE I + II
AISHE Report ESI – Reports & Indices
RELEVANT FOR
PHASE 1 & PHASE 2
For the purpose of this Survey, Higher Education is
defined as the education, which is obtained after
completing 12 years of schooling or equivalent and is of
the duration of at least nine months (full time) or after
completing 10 years of schooling and is of the duration
of at least 3 years.

The education may be of the nature of General,


Vocational, Professional or Technical education.
All India Survey on Higher Education (AISHE) 2021-2022

Ø Released annually by: Ministry of Education

Ø Ministry has been conducting AISHE since 2011, covering all


Higher Educational Institutions (HEIs) in the country registered
with AISHE

Ø The survey covers all the Institutions in the country engaged


in imparting of higher education.

Ø Data is being collected on several parameters such as

v teachers, student enrolment, programmes, examination


results, education finance, infrastructure.
Number of Institutions
• A total of 1,168 Universities/University level
Institutions, 45,473 Colleges and 12,002
Stand Alone Institutions were registered in
AISHE 2021-22.

• In all, 341 Universities/University level


institutions have been established since 2014-
15.

• 17 Universities (of which 14 are State Public


Universities) and 4,470 Colleges are exclusively
for women.
Student Enrolment
• The total enrolment in higher education has increased to nearly 4.33 crore in 2021-22 from 4.14
crore in 2020-21.
• There has been an increase of around 91 lakh in the enrolment from 3.42 crore (26.5%) in 2014-15.
Female Enrolment
• Female enrolment has increased to 2.07 crore in 2021-22 from 2.01 crore in 2020-21.
• There has been an increase of around 50 lakh in the female enrolment from 1.57 crore (32%) in 2014-
15.
Social Category-wise Enrolment
Category Enrolment in 2021-22 Enrolment in 2014-15 % increase since
2014-15
SC students 66.23 lakh 46.07 lakh 44%

ST students 27.1 lakh 16.41 lakh 65.2%

OBC students 1.63 crore 1.13crore 45%

Category Enrolment in 2021-22 Enrolment in 2020-21 Enrolment in 2014-15 % increase since


2014-15
SC Female students 31.71 lakh 29.01 lakh 21.02 lakh 51%

ST Female students 13.46 lakh 12.21 lakh 7.47 lakh 80%

OBC Female students 78.19 lakh 72.87 lakh 52.36 lakh 49.3%

Minority Student Enrolment


• It has increased to 30.1 lakh in 2021-22 from 21.8 lakh in 2014-15 (an increase of 38%).
• Female Minority Student enrolment has increased to 15.2 lakh in 2021-22 from 10.7 lakh in 2014-15
(42.3% increase).
Gross Enrolment Ratio
• Overall GER has increased from 23.7 in 2014-15 to 28.4 in 2021-22.
• Female GER has increased to 28.5 in 2021-22 from 22.9 in 2014-15.
• GER of Female SC students has increased to 26 in 2021-22 from 18.1 in 2014-15.
• GER of Female ST students increased to 20.9 in 2021-22 from 12.2 in 2014-15
Gender Parity Index
• It is the ratio of female GER to male GER.
• GPI is 1.01 in 2021-22.
• GPI has continued to be above 1 since 2017-18 i.e., female GER continues to be more
than male GER for fifth consecutive year

Undergraduate Level Courses Postgraduate Level Ph.D. Courses In 2021-22, out of the total
Courses enrolment in UG, PG, Ph.D.
• 78.9% of the total students • 12.1% of the total • Enrolment has increased by 81.2% and M.Phil. levels, 57.2 lakh
students to 2.12 lakh in 2021-22 compared to
students are enrolled in Science
• Enrolment is highest in- 1.17 lakh in 2014-15
v Arts (34.2%), followed by • maximum students Stream, with female students
science (14.8%), Commerce are enrolled in • Female Ph.D. enrolment has (29.8 lakh) outnumbering male
(13.3%) and Engineering & v Social Science doubled to 0.99 lakh in 2021-22
Technology (11.8%). (21.1%) followed by from 0.48 lakh in 2014-15. students (27.4 lakh)
science (14.7%) v The annual increase for the period
2014-15 to 2021-22 in Female Ph.D.
enrolment is 10.4%.
Universities
• Government Universities constituting 58.6% of total Universities.
• Government Universities contribute 73.7% of total enrolment, Private Universities account
for 26.3% of total enrolment.

• The total number of Pass-Outs has increased to 1.07 Crore in 2021-22 as against 95.4 lakh in 2020-21.
• The total number of foreign nationals/students enrolled in India is 46,878.

Faculties
• The total number of faculty/teachers in 2021-22 are 15.98 lakh,
of which about 56.6% are male and 43.4% are female.

• Female faculty/teachers have increased to 6.94 lakh in 2021-22


from 5.69 lakh in 2014-15 (an increase of 22% since 2014-15)

• There is a marginal betterment of female per 100 male faculty


from 75 in 2020-21 to 77 in 2021-22.
Let’s Practice
ESI Reports & Indices

Q1. Consider the following statements regarding the All India Survey on Higher
Education (AISHE) for the year 2021-2022 and choose the correct ones:

[A] The survey was conducted by the Ministry of Commerce.


[B] This was the first year that the Ministry of Education conducted AISHE.
[C] The survey observed a decline in student enrolment in higher education from
the year 2014-15 to 2021-22.
[D] There was an increase of 91 lakh students in higher education enrolment,
marking a 26.5% rise since 2014-15.
[E] Female Gross Enrollment Ratio (GER) saw a decrease in 2021-22 as compared
to 2014-15.

Answer: Option D
ESI Reports & Indices

Q.2) Consider the following statements regarding the All India Survey on
Higher Education (AISHE) for the year 2021-2022 and identify the incorrect
statements:
1. The GER of SC students has increased to 25.9 in 2021-22 from 18.9 in
2014-15.
2. The GER of Female ST students has shown even more spectacular
increase to 20.9 in 2021-22 from 12.2 in 2014-15.
3. Among Disciplines at undergraduate level in AISHE 2021-22, enrolment is
highest in Science (34.2%), followed by arts (14.8%).
[A] Only 1
[B] Only 2
[C] 1 and 3
Answer: Option E
[D] 2 and 3
[E] Only 3
ESI Homework Question Government Schemes
Previous Session

Q. Consider the following statements regarding the scheme for the promotion of
Coal/Lignite Gasification Projects approved by the Cabinet chaired by Prime Minister Shri
Narendra Modi:
1. The scheme provides a total financial assistance of Rs. 8,500 crore, specifically
targeting coal gasification projects.
2. In Category II of the scheme, private sector projects are exclusively eligible for a lump-
sum grant of Rs. 1,000 crore or 15% of capex, whichever is lower.
3. For Category III, the projects need to have a minimum production capacity of 1500
Nm3/hr Syn gas to be eligible for a grant of Rs. 100 crore or 15% of capex, whichever is
lower.
Choose the correct combination of statements:
[A] 1 and 2 only
[B] 2 and 3 only
[C] 1 and 3 only Answer: Option C
[D] All of the above
[E] None of the above
Performers
ESI Homework Question Reports & Indices

Q) Consider the following statements regarding the All India Survey on Higher
Education (AISHE) for the year 2021-2022 and identify the correct
statements:
1. The total number of Pass-Outs has increased to 1.07 Crore in 2021-22 as
against 95.4 lakh in 2020-21.
2. The total number of faculty/teachers in 2021-22 are 15.98 lakh, of which
about 56.6% are female and 43.4% are male.
3. The total number of foreign nationals/students enrolled in India is 1,46,878.

[A] Only 1
[B] Only 2
[C] 1 and 3
[D] 2 and 3
[E] Only 3
Topic Tag
PHASE I + II
The Indian Economy a Review: January 2024 ESI – Reports and Indices

PHASE I
India Mother of Democracy GA – Miscellaneous
UNESCO World Heritage List GA – Places in News
SADA TANSEEQ GA - Defence
Rohan Bopanna GA – Persons in News
Dr. Nitya Anand GA – Persons in News
RELEVANT FOR
PHASE 1 & PHASE 2
• Report Name: The Indian Economy, A Review January 2024.
• Released by: Department of Economic Affairs
Three Main Challenges highlighted
• Reliance on Global Supply as global economy is struggling to
maintain recovery post Covid 19 as successive shocks have
buffed it.
• Advent of Artificial Intelligence poses growth for service sector,
technology might remove the advantage of cost
competitiveness.
• Energy transition challenge concerns over rising a temperature
have led to a single minded focus of reducing carbon emission.
1. India's gross domestic product (GDP) would grow faster than 7.2%
in FY24 with third straight year when the Indian economy would
grow by more than 7%.
2. India has taken the overall public sector capital investment from
₹5.6 lakh crore in FY15 to ₹18.6 lakh crore in FY24.
Financial Sector
1. Non Food credit excluding personal loan is growing at double digit.
2. Under PM Jan Dhan Yojana, women bank account holders have
increased from 53% in 2015-16 to 78.6% in 2019-21.
3. Fiftyone crore bank accounts under Jan Dhan Yojana now have
total deposits of over ₹2.1 lakh crore.
4. In Dec. 2019, household financial assets were 86.2 per cent of GDP;
liabilities were 33.4 per cent of GDP which has enhanced to 103.1
per cent and 37.6 per cent in March 2023.
5. Net Financial Assets of households were 52.8 per cent of GDP in
Dec. 2019, and by March 2023, it had improved to 65.5 per cent of
GDP.
Women Participation
1. The female labour force participation rate (LFPR) hiked from 23.3% in 2017-
18 to 37% in 2022–23.
2. The female Gross Enrolment Ratio (GER) in higher education has quadrupled
from 6.7% in FY01 to 27.9% in FY21.
3. GER has more than doubled from 24.5% to 58.2% between FY05 and FY22. .
Key Achievements
1. India has moved from the 10th largest economy of the world to the 5th
largest economy of the world.
2. India is the third-largest fintech economy in the world, after the USA and the
UK.
3. India has become the fourth-largest stock market overtaking Hong Kong to
in the world.
Lessons from the growth experience till 2014
• Indian economy transitioned from a closed economy to an open economy.
• Indian Economy transitioned from the dominance of public investment to
the co-existence of public and private investment.
• Technology began to be identified as a key growth driver
RELEVANT FOR
PHASE 1
• Announced by: Indira Gandhi National Centre for the
Arts (IGNCA).
List of UNESCO World Heritage
• It includes 12 component parts, representing forts and Sites in Maharashtra:
✓ Ellora Caves (1983)
military structures (11 in Maharashtra and Gingee Fort in
✓ Ajanta Caves (1983)
Tamil Nadu). ✓ Elephanta Caves (1987)
✓ Chhatrapati Shivaji
Maharaj Terminus (1997)
✓ Victorian Gothic and Art
Deco Ensembles of
Mumbai (2018)
✓ Western Ghats (2012) –
Natural Category
• Rohan Bopanna along with his partner Matthew Ebden
won the men’s doubles.
• It was his maiden Grand Slam men's doubles title
victory.
• Recently achieved the World No. 1 ranking in ATP
men's doubles
• Other Achievements
• Recently awarded Padam Shri by the government of
India.
• Mixed doubles title at the 2017 French Open with his
partner Gabriela Dabrowski.

Oldest ever female grand slam winner to date is Serena Williams.


About Saheli:
• Introduced in 1986
• First and only once-weekly, non-steroidal, non-
hormonal oral contraceptive pill.
• Incorporated into the National Family Programme
of India in 2016
Let’s Practice
ESI REPORTS AND INDICES

Q1. Consider the following statement with respect to "The Indian Economy, A
Review January 2024" and identify the correct statements:
1. India's GDP is projected to grow faster than 7.2% in FY24, marking the third
consecutive year of growth above 7%.
2. Public sector capital investment in India increased from ₹5.6 lakh crore in
FY15 to ₹18.6 lakh crore in FY24.
3. The female labor force participation rate (LFPR) increased from 23.3% in
2017-18 to 27.9% in 2022–23.
A. 1 and 2 only
B. 2 and 3 only
C. 1 and 3 only
D. 1, 2, and 3
E. None of the above Answer: Option A
GA GOVERNMENT SCHEMES

Q.2 Dr. Nitya Anand, known for developing India's first oral
contraceptive pill 'Saheli,' was recognized for his contributions with
which prestigious award?
A. Bharat Ratna
B. Padma Bhushan
C. Padma Shri
D. Padma Vibhushan
E. None of the above

Answer: Option C
GA PLACES IN NEWS

Q3. Which theme did the Ministry of Culture's Tableau, which won the first
spot at the 75th Republic Day Parade 2024, showcase?
A. Unity in Diversity
B. India: Land of Heritage
C. India: Mother of Democracy
D. Digital India
E. Freedom Struggle

Answer: Option C
GA Homework Question PLACES IN NEWS

Q4. Which of the following statements are true regarding the "Maratha
Military Landscapes of India" nominated for UNESCO World Heritage
recognition?
A. The nomination includes 15 forts and military structures across
Maharashtra and Tamil Nadu.
B. Gingee Fort, a part of the nomination, is located in Maharashtra.
C. All 12 component parts of the nomination are protected by the
Archaeological Survey of India.
D. This is the first cultural property from Maharashtra to be nominated.
E. The Maratha Military Landscapes were developed between the 17th
and 19th centuries by the Maratha rulers.
ESI Homework Question REPORTS AND INDICES

Q5. Evaluate the following statements from "The Indian Economy, A


Review January 2024“ and choose the incorrect one:
1. India has overtaken Hong Kong to become the fourth-largest stock
market in the world.
2. The female Gross Enrolment Ratio (GER) in higher education
increased from 6.7% in FY01 to 27.9% in FY21.
3. India is currently the second-largest fintech economy globally.
A. 1 and 2 only
B. Only 3
C. 1 and 3 only
D. Only 2
E. 1, 2, and 3
Topic Tag
PHASE I + II

World Wetland Day ESI – Sustainable Development and


Environmental Issues
Animal Husbandry Infrastructure ESI – Government Schemes
Development Fund (AHIDF)
PHASE I
UPI in France GA – Banking and Finance
Bharat Ratna Awards GA – Awards
Vyomitra GA – Science and Technology
Zakir Hussain GA – Persons in News
Triple A Award for Sustainable Finance GA – Awards in News
RELEVANT FOR
PHASE 1 & PHASE 2
About World Wetlands Day
• Date – 2nd February 2024
• Theme: ‘Wetlands and Human Wellbeing’

Wetlands in India:
• India now has 80 Ramsar wetlands.
• Five new addition: Ankasamudra Bird Conservation Reserve,
Aghanashini Estuary and Magadi Kere Conservation Reserve
(Karnataka), Karaivetti Bird Sanctuary and Longwood Shola
Reserve Forest (Tamil Nadu)
• Tamil Nadu has maximum number of Ramsar Sites (16 sites)
followed by Uttar Pradesh (10 sites).
• Territorial Coverage under Ramsar sites in India is now 1.33 million
hectares.
• Wetland City Accreditation (WCA):
• India has nominated Indore, Bhopal, and Udaipur for the Wetland
City Accreditation scheme under the Ramsar Convention.
✓ Wetlands in Indore: Sirpur Wetland and Yashwant Sagar .
✓ Wetlands in Bhopal: Bhoj Wetland.
✓ Wetlands in Udaipur: Pichola, Fateh Sagar, Rang Sagar,
Swaroop Sagar, and Doodh Talai.
• Accreditation helps cities gain international recognition for their
efforts to protect and wisely use their natural or human-made
wetlands.
• These nominations mark the first Indian cities proposed for this
accreditation.
About Ramsar Wetlands
• Ramsar sites are wetlands of international importance designated
under the Ramsar Convention.
• Ramsar Convention was adopted in 1971.
• Partnering Agency: Convention on Biological Diversity (CBD) in
conjunction with the Ramsar Convention.
• Objective: Conservation and wise use of wetlands through local and
national actions and international cooperation.
• Tenure/Timeline: Extended by three years up to 2025-26.
• Financial Allocation: Rs.29,610.25 crore.

About Animal Husbandry Infrastructure Development Fund (AHIDF)


• Launch Year – 2020 as a part of Atmanirbhar Bharat Package
• Type – Central Sector Scheme
• Ministry: Ministry of Fisheries, Animal Husbandry & Dairying.
• Implementing Agency: Department of Animal Husbandry and Dairying.
• Territorial Coverage: Pan India.
• Aim/Objective: To promote infrastructure development in the animal
husbandry sector, including dairy, meat processing, and animal feed, to
enhance production capacity and encourage private sector investment.
• Eligible Entities: Individuals, private companies, FPOs, MSMEs, Section 8
companies and Dairy cooperatives for modernization efforts.
• Features:
• A credit guarantee up to 25% of the borrowed credit from a Rs.750
crore Credit Guarantee Fund will be provided to MSMEs and Dairy
Cooperatives, facilitated by NABARD.
• 3% interest subvention for 8 years, including a 2-year moratorium on
loans.
❑ Up to 90% from scheduled banks, National Cooperative Development
Corporation (NCDC), National Bank for Agriculture and Rural
Development (NABARD) and National Dairy Development Board
(NDDB).
• Maximum re-payment period - 10 years inclusive of moratorium of 2
years on repayment of principal.
• Loan Details:
• Loans up to 90% of the project cost for eligible projects.
• Beneficiary contribution varies by enterprise size, with 10% for
Micro and Small units, 15% for medium units and up to 25% for
others.
• Benefits:
• Scheme will be a channel towards employment generation directly
and indirectly to 35 lakh people through entrepreneurship
development.
• It aims for wealth creation in the livestock sector.
• At present, AHIDF has benefitted directly /indirectly approximately
to 15 lakh farmers
RELEVANT FOR
PHASE 1
• Entities Involved in launching the initiative:
• National Payments Corporation of India (NPCI) from
India
• Lyra (a French leader in securing e-commerce and
proximity payments).
• Purpose: To enable seamless financial transactions for
Indian tourists at the Eiffel Tower, as part of the vision to
globalize UPI.

About UPI:
• An Indian instant payment system developed by the
National Payments Corporation of India (NPCI) in 2016.
• Designed to consolidate multiple bank accounts into a
single mobile application.
• Shri Lal Krishna Advani has become the 50th Recipient after Karpoori Thakur
(Posthumous - politician and former Bihar chief minister who became 49th Recipient).

• About Bharat Ratna:


• Highest civilian award of the nation.
• To recognize exceptional service/performance of the highest order in any field of
human endeavor.
• Started in 1954 with first recipients being Chakravarti Rajagopalachari (1954),
Sarvapalli Radhakrishnan (1954) and Chandrasekhara Venkata Raman (1954).
• In terms of Article 18 (1) of the Constitution, the award cannot be used as a prefix
or suffix to the recipient's name. An award winner can only use the following:
• ‘Awarded Bharat Ratna by the President’ or ‘Recipient of Bharat Ratna
Award’
• Maximum Bharat Ratna in a Year: 3.
About Vyommitra: About Gaganyaan Project:
• A female robot astronaut, capable of monitoring • India's first human spaceflight which will carry
module parameters, executing life support Indian astronauts into a 400-kilometer orbit and
operations, and responding to queries safely returning them to Earth, landing in India's sea
• “Vyommitra" mission, an uncrewed robot flight water.
will precede the Gaganyaan mission in 3rd • Scheduled for launch in 2025.
quarter of 2024. • First Test Vehicle Flight “TV D1”, qualifying the crew
• "Vyommitra" is derived from two Sanskrit words, escape system and parachute system for
"Vyoma" (meaning Space) and "Mitra" (meaning "Gaganyaan," was accomplished in October 2023
Friend).
List of Awards Won by Zakir Hussain
• BEST CONTEMPORARY INSTRUMENTAL ALBUM - “As We
Speak,” - Béla Fleck, Zakir Hussain, Edgar Meyer, featuring
Rakesh Chaurasia
• BEST GLOBAL MUSIC PERFORMANCE - “Pashto,” - Béla Fleck,
Edgar Meyer and Zakir Hussain featuring Rakesh Chaurasia
• BEST GLOBAL MUSIC ALBUM - “This Moment,” - by Shakti
(John McLaughlin, Zakir Hussain, Shankar Mahadevan,
V.Selvaganesh, Ganesh Rajagopalan)
Ravi Shankar was the first Indian to
win Grammys in 1968 under Best
Other Awardees Chamber Music Performance
• Album of the year - Taylor Swift (1st performer to win this category.
award for 4 times), Midnights
• Record of the year - Miley Cyrus, Flowers
• Song of the year - Billie Eilish, What Was I Made For?
• Purpose of the Award: For the issuance of $750
million USD Green Bonds in 2023.
• This issuance was the first USD Green Bond from India
and the largest ever Senior Green Bond Tranche by a
South or South-East Asian issuer.

About REC (Rural Electrification Corporation) Limited:


• Established in 1969. REC also priced its inaugural Euro-Yen
• A 'Maharatna' CPSE under the Ministry of Power. Green bonds of JPY 61.1 billion in
January 2024, marking the largest Yen
• Registered with RBI (Reserve Bank of India) as Non-
Bond issuance by an Indian corporate
Banking Finance Company (NBFC), and Infrastructure
Financing Company (IFC).
Let’s Practice
ESI BANKING AND FINANCE

Q1. Which of the following statements about the launch of Unified Payments
Interface (UPI) at the Eiffel Tower, Paris, France, is/are incorrect?
[A] The National Payments Corporation of India (NPCI) partnered with Lyra to
enable UPI payments at the Eiffel Tower.
[B] UPI was developed by the NPCI in 2016 to allow users to consolidate
multiple bank accounts into a single mobile application.
[C] The initiative marks the first instance of UPI payments being available in
Europe, starting with the Eiffel Tower.
[D] Indian tourists are the largest group of international visitors to the Eiffel
Tower.
[E] A Memorandum of Understanding was signed with Al Etihad Payments of
Abu Dhabi in 2023 to enable cross-border transactions.

Answer: Option D
GA AWARDS

Q.2 Identify the correct statement(s) about the Bharat Ratna award
recently (February 2024) seen in news:
[A] The Bharat Ratna is India's second-highest civilian award,
recognizing contributions in any field of human endeavor.
[B] Recipients of the Bharat Ratna are allowed to use the award as a
prefix to their names as per the Constitution of India.
[C] The award was first conferred in 1953, with Chakravarti
Rajagopalachari, Sarvapalli Radhakrishnan, and Chandrasekhara
Venkata Raman being the inaugural awardees.
[D] A maximum of four Bharat Ratna awards can be granted in a single
year.
[E] As of 2024 the Bharat Ratna has been awarded to 50 individuals,
including 17 posthumous recognitions.

Answer: Option E
GA SCIENCE AND TECHNOLOGY

Q3. Which of the following statements accurately describes the role and
significance of Vyommitra in the context of India's space exploration
efforts?
[A] Vyommitra is intended to accompany human astronauts on the
Gaganyaan mission, serving as a co-pilot.
[B] Vyommitra, whose name means "Space Friend" in Sanskrit, will lead
the first human spaceflight mission by ISRO in 2024.
[C] Designed to monitor module parameters, execute life support
operations, and respond to queries, Vyommitra will embark on an
uncrewed robot flight in the 3rd quarter of 2024.
[D] Vyommitra is a communication satellite launched by ISRO to enhance
India's satellite communication capabilities.
[E] Vyommitra was developed as a part of ISRO's Chandrayaan-2 mission
to explore the lunar surface.
Answer: Option C
ESI REPORTS AND INDICES

Q4. Consider the following statements regarding the World Wetlands Day 2024
celebrations and the Ramsar sites in India. Identify the correct statement(s):
1. World Wetlands Day 2024 was celebrated at Sirpur Lake in Indore, which is
recognized as a Ramsar site.
2. The celebration marked the increase of India's Ramsar sites tally to 80, including
notable sites like Odisha’s Chilka Lake and West Bengal’s Sundarbans.
3. The theme for World Wetlands Day 2024 was ‘Wetlands and Climate Change’.
4. The Ramsar Convention, aiming at the conservation and wise use of wetlands,
was adopted in 1982.
[A] 1 and 2
[B] 1, 2, and 4
[C] 1, 3, and 4
[D] 3 and 4
[E] All of the above Answer: Option A
GA Homework Question AWARDS

Q5. Select the correct statement(s) based on the 66th Grammy Awards 2024:
1. Zakir Hussain won three Grammy awards, making him the Indian artist with the most
wins at this edition of the Grammys.
2. "As We Speak" won the award for Best Contemporary Instrumental Album and
featured artists such as Béla Fleck, Zakir Hussain, and Edgar Meyer.
3. Rakesh Chaurasia received his first Grammy award in 2024.
4. Taylor Swift's album "Midnights" won the Album of the Year, marking her fourth win in
this category.
5. "Pashto" secured the Best Global Music Performance, a collaboration by Béla Fleck,
Edgar Meyer, and Zakir Hussain featuring Rakesh Chaurasia.
[A] 1, 2, and 4
[B] 2, 3, and 5
[C] 1, 2, and 5
[D] 2, 4, and 5
[E] All of the above
ESI Homework Question REPORTS AND INDICES
Q6. Evaluate the statements below regarding the Credit Enhancement Guarantee Scheme for
Scheduled Castes (CEGSSC) and select the correct statement(s):
1. The CEGSSC was launched as part of the 2014-15 Union Budget to support
entrepreneurship among Scheduled Castes by offering a credit enhancement guarantee
for loans.
2. Eligibility for the scheme extends to Individuals, Sole Proprietorships, Registered
Companies, Partnerships, and Societies with a minimum of 75% ownership and
management control by SC members for at least 12 months.
3. The scheme provides a guarantee cover for loans ranging from Rs. 15 lakhs to Rs. 5 crores,
with a guarantee period of up to 7 years or the loan repayment period, whichever is earlier.
4. Standalone Working Capital facilities are eligible under the scheme, and a third-party
guarantee is required for loans availed.
5. Loans under CEGSSC are subject to an interest rate cap that the maximum interest rate
charged cannot exceed 3% over the MLI's base rate.
[A] 1, 3, and 5
[B] 1, 2, and 4
[C] 2, 3, and 4
[D] All of the above
[E] 1, 2, and 5
Topic Tag
PHASE I + II

Credit Enhancement Guarantee Scheme for ESI – Government Schemes


Schedule Caste
Edcil Vidyanjali Scholarship Program ESI – Social Sectors – Education

Public Examination (Prevention of Unfair ESI – Social Sectors – Education


Means) Bill 2024
National Social Assistance Program ESI – Government Schemes

PHASE I
National Science Day GA – Days in News
RELEVANT FOR
PHASE 1 & PHASE 2
As on 31.1.2024, on cumulative basis, 66 entities promoted by SC entrepreneurs have been
sanctioned guarantee cover of approx. Rs.72 crore against bank loan of approx.

• Ministry - Ministry of Social Justice & Empowerment (MoSJE)


• Implementing Agency - IFCI Ltd
• Objective - To promote entrepreneurship amongst the Scheduled Castes
entrepreneurs by providing Credit Enhancement Guarantee to Member Lending
Institutions (MLIs) who shall be providing financial assistance to SC entrepreneurs.
• Eligibility (belonging to Scheduled Castes) –
❑ Registered companies (75% shareholding by Scheduled Caste
entrepreneurs/promoters/members with management control for the past 12 months)
❑ Partnerships & Societies (75% shareholding with Scheduled Caste Partners for the past
12 months; none of the partners should be below the age of 18 years)
❑ Small and Medium Enterprises promoted and run by Scheduled Caste entrepreneur,
which are not covered under any other State/Central Government Subsidy/ Guarantee
Schemes
• Credit Structure
Key Features
• No collateral required.
• No, Third party Guarante is required.
• Loans to be provided by – MLIS who have signed the agreement for the
implementation of the scheme with IFCI ltd.
• Maximum Tenure – 7 years.
• Loan cover - Term Loan / Composite Term Loan granted to SC
Enterprises by MLIs.
• Security for MLIs - Asset created from Loan, and pledge of promoters’
shareholdings in the assisted Company/firm/society.
• Supported by: Corporate CSR (Corporate social responsibility) sponsors, About Vidyanjali Initiative:
including Fiat India, HDFC Bank Limited, ClearMedi Healthcare, Bharat
Forge, and Bill & Melinda Gates Foundation. • An initiative by the Department of
School Education and Literacy,
• Purpose:
Ministry of Education.
To provide scholarships to meritorious students of Navodaya Vidyalayas

• "Vidya" (correct knowledge or
(NVS) who lack financial means to pursue education, in line with National
clarity) and "Anjali" (an offering
Education Policy 2020.
with both hands) in Sanskrit, means
• Program symbolises whole-of-society approach to empowerment through contributions through knowledge
access and opportunities of education.
and resources.
• Key Facts and Initiatives: • Vidyanjali portal allows volunteers
• Nearly 14,000 NVS students have secured admission to premier institutes to contribute their services, assets,
like IITs and NITs without coaching. or materials to schools across India.
• Launch of the Vidyanjali Fintech Platform/portal to facilitate this
initiative.
• Introduced by Ministry of Personnel, Grievances, and Pensions.
Purpose of the initiative or programme:
• To penalize malpractices in public examinations.
• Applicable to central recruitment and entrance exams conducted by
UPSC, SSC, RRBs, IBPS, NTA and such other authorities as may be
notified by the Centre.
• It specifically addresses issues related to unfair means in the conduct
of exams that are not covered under the purview of the Bharatiya
Nyaya Sanhita Act.
• Key Facts and Initiatives:
• Defines "unfair means" broadly, including leakage of question papers, collusion, and
unauthorized access to examination materials.
• It does not include examination at school level.
• All offences are cognizable, non-bailable, and non-compoundable.
• Punishment:
• A minimum of 3 to 5 years of imprisonment to curb cheating.
• 5 to 10 years of imprisonment and a minimum fine of Rs 1 crore for organised crimes of
cheating.
• Bonafied candidate or student are left out of the purview of bill.

• Some like Gujarat and Assam already have their own legislation.
• For the first time, all exams held by the UPSC and SSC are now being conducted in 13 regional
languages and efforts are on to conduct exams in all regional 22 Scheduled Languages.
• Pension amounts have been periodically revised since its inception on 15th August 1995.
• 2007: Eligibility criterion was changed from "destitution" to Below Poverty Line (BPL).
• 15th Finance Commission Cycle (2021-26):
• Government approved the continuation of the NSAP scheme in its present form due to
financial constraints.
• Encouraged States/UTs to provide top-up amounts at least equal to the central assistance.
• Guidelines mandate monthly disbursement of pensions by States/UTs.
• Currently, 27 States/UTs disburse pensions monthly.
• Himachal Pradesh, Uttar Pradesh and Uttarakhand disburse pension on quarterly basis.
• Arunachal Pradesh and Nagaland disburse pension on non-periodic basis.
• To ensure timely pension payments, almost all States have adopted the Direct Benefit Transfer
(DBT) and Aadhaar Payment Bridge (APB) systems, using the Public Financial Management
System (PFMS) for disbursements.
RELEVANT FOR
PHASE 1
• Day commemorates the discovery of the 'Raman Effect' by
Sir C.V. Raman.
• He was later awarded the Nobel Prize in 1930 for the
same (first Nobel Prize in science for India). .

India’s Achievements in Science and Innovation:


• Globally among the top 5 countries in scientific
research publications.
• 40th in Global Innovation Index (GII).
• Our patent filing has crossed 90,000 which is highest
in two decades.
Let’s Practice
ESI SOCIAL SECTORS - EDUCATION
Q1. Recently in February 2024, consider the following statements about the EdCIL
Vidyanjali Scholarship Programme:
1. The programme was launched by the Ministry of Corporate Affairs.
2. It is supported by Corporate CSR sponsors such as Fiat India, HDFC Bank Limited,
and the Bill & Melinda Gates Foundation.
3. The primary aim is to provide scholarships to all Indian students for international
education.
4. Nearly 14,000 NVS students have been admitted to premier institutes like IITs and
NITs without coaching assistance through this programme.
5. A Vidyanjali Fintech Platform/portal was developed to facilitate the scholarship
process.
Which of the above statements are correct?
[A] 1, 3, and 5
[B] 2 and 4
[C] 2, 4, and 5 Answer: Option C
[D] 3 and 5
[E] 1 and 3
ESI INDIAN POLITICAL SYSTEM

Recently in February 2024, consider the following statements about The Public Examinations
(Prevention of Unfair Means) Bill, 2024:
1. It was introduced by the Ministry of Personnel, Grievances, and Pensions and passed by the
Lok Sabha.
2. The bill is designed to address and penalize the use of unfair means in central and state
recruitment and entrance exams.
3. "Unfair means" under the bill include actions such as the leakage of question papers, collusion,
and unauthorized access to examination materials.
4. All offences under the bill are cognizable, non-bailable, and compoundable.
5. For cheating, the bill prescribes a minimum of 3 to 5 years of imprisonment, and for organized
crimes of cheating, it mandates 5 to 10 years of imprisonment and a minimum fine of Rs 1 crore.
6. It mandates that all exams conducted by UPSC and SSC must be available in all 22 Scheduled
Languages of India.
Which of the above statements are correct?
[A] 1, 2, 3, and 4
[B] 2, 3, 4, and 5
[C] 1, 2, 4, and 5
[D] All except 6
Answer: Option E
[E] 1, 3 and 5
ESI GOVERNMENT SCHEMES

Q3. Recently, regarding the National Social Assistance Programme (NSAP), consider the following
statements:
1. The NSAP pension amounts have never been revised since the program's inception in 1995.
2. In 2007, the eligibility criterion for the NSAP was changed from "destitution" to Below Poverty
Line (BPL).
3. During the 15th Finance Commission Cycle, the government decided to discontinue the NSAP
due to financial constraints.
4. States and Union Territories are encouraged to provide top-up amounts at least equal to the
central assistance under NSAP.
5. Currently, all States and UTs disburse pensions on a monthly basis.
6. The Direct Benefit Transfer (DBT) and Aadhaar Payment Bridge (APB) systems have been
widely adopted for the disbursement of pensions through the Public Financial Management
System (PFMS).
Which of the above statements are correct?
[A] 2, 4, and 6
[B] 1, 3, and 5
[C] 2, 3, and 4
[D] 4, 5, and 6 Answer: Option A
[E] 1, 3, and 6
GA DAYS IN NEWS

Q4. Recently in February 2024, which of the following statements about National
Science Day is true?
[A] The theme for National Science Day 2024 was "Future Technology for
Sustainable Development".
[B] National Science Day was first celebrated in 1995 to encourage scientific temper
among the youth.
[C] The day marks the anniversary of the discovery of the 'Raman Effect' by Sir C.V.
Raman, commemorating India's first Nobel Prize in science.
[D] All of the above
[E] None of the above

Answer: Option A
ESI Homework Question GOVERNMENT SCHEMES

Q5. Evaluate the statements below regarding the Credit Enhancement Guarantee
Scheme for Scheduled Castes (CEGSSC) and select the correct statement(s):
1. The CEGSSC was launched as part of the 2014-15 Union Budget to support
entrepreneurship among Scheduled Castes by offering a credit enhancement
guarantee for loans.
2. Eligibility for the scheme extends to Individuals, Sole Proprietorships, Registered
Companies, Partnerships, and Societies with a minimum of 75% ownership and
management control by SC members for at least 18 months.
3. Loans to be provided by MLIS who have signed the agreement for the
implementation of the scheme with IFCI ltd.
4. Standalone Working Capital facilities are eligible under the scheme, and a third-party
guarantee is required for loans availed.
[A] 1 and 2
[B] 2 and 4
[C] 3, and 4
[D] All of the above
[E] 1, 2, and 4
HOMEWORK QUESTIONS
FROM PREVIOUS SESSION
GA Homework Question AWARDS

Q5. Select the correct statement(s) based on the 66th Grammy Awards 2024:
1. Zakir Hussain won three Grammy awards, making him the Indian artist with the most
wins at this edition of the Grammys.
2. "As We Speak" won the award for Best Contemporary Instrumental Album and
featured artists such as Béla Fleck, Zakir Hussain, and Edgar Meyer.
3. Rakesh Chaurasia received his first Grammy award in 2024.
4. Taylor Swift's album "Midnights" won the Album of the Year, marking her fourth win in
this category.
5. "Pashto" secured the Best Global Music Performance, a collaboration by Béla Fleck,
Edgar Meyer, and Zakir Hussain featuring Rakesh Chaurasia.
[A] 1, 2, and 4
[B] 2, 3, and 5
[C] 1, 2, and 5
[D] 2, 4, and 5 Answer: Option E
[E] All of the above
ESI Homework Question GOVERNMENT SCHEME
Q6. Evaluate the following statements about the Animal Husbandry Infrastructure
Development Fund (AHIDF) and identify the correct statement(s):
1. The AHIDF aims to stimulate infrastructure development in the animal husbandry sector,
offering a credit guarantee up to 25% of the borrowed credit from a dedicated Credit
Guarantee Fund facilitated by NABARD.
2. Eligible entities for AHIDF include individuals, private companies, FPOs, MSMEs, Section 8
companies, and now dairy cooperatives, aiming for modernization and capacity
enhancement in dairy, meat processing, and animal feed sectors.
3. The fund provides a 5% interest subvention for loans up to 90% of the project cost from
scheduled banks, with a maximum repayment period of 12 years, including a 3-year
moratorium on the repayment of principal.
4. AHIDF has been extended for three years up to 2025-26, with a financial allocation of
Rs.29,610.25 crore to support infrastructure development across India in the animal
husbandry sector.
[A] 1 and 2
[B] 2 and 4
[C] 1, 2, and 4
Answer: Option C
[D] All of the above
[E] 1, 2, and 3
Topic Tag
PHASE I + II

Pradhan Mantri Matsya Kisan Samridhi Sah- ESI – Government Schemes


Yojana (PM-MKSSY)
PHASE I
Bharat Ratna Awards GA – Awards
Wildlife Photographer of the Year People's GA – Awards
Choice Award
Narendra Kumar Yadav GA – First in News
RELEVANT FOR
PHASE 1 & PHASE 2
• Ministry - Ministry of Fisheries, Animal Husbandry & Dairying
• Sub Scheme of – Pradhan Mantri Matsya Sampada Yojana
• Tenure - FY 2023-24 to FY 2026-27
• Coverage - All States/Union Territories
Beneficiaries

• Fishers, Fish (Aquaculture) Farmers, Fish workers, Fish Vendors


or such other person directly engaged in fisheries value chain.
• Micro and Small enterprises in the form of Proprietary Firms,
Partnership Firms and Companies registered in India, Societies,
Limited Liability Partnerships (LLPs), Cooperatives, Federations,
Village Level Organizations like Self Help Groups (SHGs), Fish
Farmers Producer Organizations (FFPOs) and Startups engaged
in fisheries and aquaculture value chains.
• FFPOs also include Farmers Producer Organizations (FPOs).
• Any other beneficiaries that may be included by the Department
of Fisheries, Gol as targeted beneficiaries.
Aim and Objective
• Gradual Formalization of the unorganized fisheries sector (support 6.4 lakh micro-enterprises
and 5,500 fisheries cooperatives) through self registration of fishers, fish farmers.

• Providing one-time incentive to beneficiaries for purchasing aquaculture insurance

• Facilitating access to institutional financing fisheries sector micro and small enterprises .

• Incentivizing fisheries and aquaculture microenterprises through performance grants for

• Improving fisheries sector value-chain efficiencies to reengage in production, creation


and maintenance of jobs with priority for women and to enhance value chain efficiencies
within selected value chains under a set of measurable parameters.

• Adoption and expansion of fish and fishery product safety and quality assurance systems
including creation and maintenance of jobs.
Component 1
1A: Formalization of fisheries sector and facilitating access of fisheries microenterprises to
Government of India programs for working capital financing:

• National Fisheries Digital Platform (NFDP) will be created and all the
stakeholders will be mobilized to register on it (provide 40 lakh small
and micro-enterprises).
• Activities of NFDF:
o Serve as multiple functions including disbursement of financial
incentives.
o Proposed to undertake activities such as training and extension
support, improving financial literacy, facilitating project
preparation and documentation through financial support,
reimbursing the processing fee and such other charges, if any
and strengthening of existing fisheries cooperative societies.
1B: Facilitating adoption of aquaculture insurance:
Incentive for Aquaculture Farm Aquaculture other than farms such as cage
• Proposed to facilitate creation of appropriate culture, Re-circulatory Aquaculture
insurance product and to cover at least 1 lakh System (RAS), bio-floc, raceways, etc.
hectare of aquaculture farms during project • Incentive payable for Intensive form - 40%
period to provide the scale of operation. of premium.
• Incentive will be at the rate of 40% of the cost of • Maximum incentive payable: 1 lakh and
premium subject to a limit of Rs.25000 per the maximum unit size eligible will be of
hectare of water spread area of the aquaculture 1800 m3.
farm.
• Maximum farm size eligible for incentive: 4 The aforesaid benefit of 'onetime
hectares of water spread area. incentive' will be provided for aquaculture
insurance purchased for one crop only i.e.
• Maximum incentive payable to single farmer: one crop cycle.
Rs.1,00,000
Component 2: Supporting microenterprises to improve fisheries sector value
chain efficiencies:

Scale of performance grant and the criteria for providing performance grants are
indicated below:
i. For a Microenterprise
i. General Category - Shall not exceed 25% of the total investment or Rs.35 lakhs,
whichever is lower.
ii. SC, ST and Women owned microenterprises- Shall not exceed 35% of total
investment or Rs.45 lakhs.
ii. For Village Level Organizations and Federations of SHGs, FFPOs and Cooperatives -
35% of total investment or Rs.200 lakhs, whichever is lower.
Component 3: Adoption and expansion of fish and fishery product safety and
quality assurance systems

The scale of performance grants the criteria for providing performance grants are
indicated below:
i. For a Microenterprise
i. General Category - Shall not exceed 25% of the total investment or Rs.35 lakhs,
whichever is lower.
ii. SC, ST and Women owned microenterprises- 35% of total investment or Rs.45 lakhs.
ii. Village Level Organizations and Federations of SHGs, FFPOs and Cooperatives -shall not
exceed 35% of total investment or Rs.200 lakhs, whichever is lower.
iii. Small enterprise
i. General Category - Shall not exceed 25% of total investment or Rs.75 lakhs,
whichever is lower.
ii. SC, ST and Women owned small enterprises - 35% of total investment or Rs.100
lakhs, whichever is lower
Performance Grant disbursement criteria for components 2 and 3

a. Number of jobs created and maintained; including jobs created and


maintained for women.
• For each job created and maintained for a
• Woman - Rs.15,000 per year will be paid
• Man - Rs.10,000 per year will be paid
• Subject to the limit of 50% of total eligible grant
b. Performance grant for investments made will be disbursed after completion
of the investment subject to the limit of 50% of the eligible grant.
Component 4

Component 4: Project management, monitoring and reporting:


Proposed to set up Project Management Units (PMUs) to manage, implement,
monitor and evaluate project activities.
Financial Allocation – Rs 6000 crores

50% (Rs.3,000 crore) 50% i.e.Rs.3,000 crore


Public finance including Anticipated investment from
the World Bank and the the beneficiaries/private
AFD external financing sector leverage
RELEVANT FOR
PHASE 1
CT 360 – 1st to 5th Feb 2024
• Award Name - Wildlife Photographer of the Year People's
Choice Award.
• Hosted by – London's Natural History Museum
• Winner - Sarikhani's "Ice Bed"
About Fit India Movement
• Launched in 2019
• Coverage – Whole India
• Aim of encouraging Indians to incorporate
physical activity and sports into their daily lives.
• It envisions a healthier and fitter India, promoting
the importance of physical fitness, mental well-
being, and holistic wellness for all citizens.
Let’s Practice
ESI GOVERNMENT SCHEMES

Q1. Regarding the financial aspects and beneficiary support under the Pradhan Mantri
Matsya Kisan Samridhi Sah-Yojana (PM-MKSSY), consider the following statements:
1. The financial allocation for the scheme is equally divided between public finance
and anticipated private sector investment, each contributing Rs.3,000 crore.
2. Performance grants for microenterprises can go up to 25% of the total investment
or Rs.35 lakhs, whichever is lower, for the general category in component 2 and 3.
3. Jobs maintained for women under the scheme will receive Rs.25,000 per year as
part of the performance grant disbursement criteria.
4. The scheme covers a wide range of beneficiaries including fishers, fish farmers,
and startups engaged in the fisheries and aquaculture value chains.
Which of the statements given above are correct?
[A] 1, 2, and 3 only
[B] 2, 3, and 4 only
[C] 1, 3, and 4 only
[D] 1, 2, and 4 only
Answer: Option D
[E] All of the above
GA AWARDS

Which among the following persons have not received the Bharat Ratna Award?
1. Indira Gandhi
2. JRD Tata
3. Lal Bahadur Shastri
4. PV Narsimha Rao
5. Kapoori Thakur
6. Mulayam Singh Yadav
7. Manmohan Singh
Which of the above statements are correct?
[A] All except 6 and 7
[B] All except 7
[C] All except 6
[D] 1, 3, 4 and 5 Answer: Option A
[E] All of the above
GA AWARDS

Q3. In the prestigious landscape of nature and wildlife photography,


the "Wildlife Photographer of the Year People's Choice Award" stands
out as a beacon of artistic excellence and environmental storytelling.
Hosted by London's Natural History Museum, this award garners
entries from across the globe, each capturing unique facets of the
natural world through the photographer's lens. Which of the following
photo has won the award given in February 2024?
A. The Happy Turtle
B. Starling Murmuration
C. Ice Bed
D. Shared Parenting
E. Aurora Jellies

Answer: Option C
ESI Homework Question GOVERNMENT SCHEMES

Q4. Consider the following statements about the Pradhan Mantri Matsya Kisan Samridhi
Sah-Yojana (PM-MKSSY):
1. The scheme aims to support 6.4 lakh micro-enterprises and 5,500 fisheries cooperatives
by providing access to institutional credit.
2. A one-time incentive for purchasing aquaculture insurance is provided under the
scheme.
3. The maximum incentive payable for aquaculture insurance is Rs.1,00,000 for farms with
up to 4 hectares of water spread area.
4. The National Fisheries Sector Digital Platform (NFDP) is designed for the self-
registration of fishers, fish farmers, and support workers.
Which of the statements given above are correct?
[A] 1, 2, and 3 only
[B] 2, 3, and 4 only
[C] 1, 2, and 4 only
[D] 1, 3, and 4 only
[E] All of the above
HOMEWORK QUESTIONS
FROM PREVIOUS SESSION
ESI Homework Question GOVERNMENT SCHEMES

Q5. Evaluate the statements below regarding the Credit Enhancement Guarantee
Scheme for Scheduled Castes (CEGSSC) and select the correct statement(s):
1. The CEGSSC was launched as part of the 2014-15 Union Budget to support
entrepreneurship among Scheduled Castes by offering a credit enhancement
guarantee for loans.
2. Eligibility for the scheme extends to Individuals, Sole Proprietorships, Registered
Companies, Partnerships, and Societies with a minimum of 75% ownership and
management control by SC members for at least 18 months.
3. Loans to be provided by MLIS who have signed the agreement for the
implementation of the scheme with IFCI ltd.
4. Standalone Working Capital facilities are eligible under the scheme, and a third-party
guarantee is required for loans availed.
[A] 1 and 2
[B] 2 and 4
[C] 3, and 4
[D] All of the above
Answer: 1 and 3
[E] 1, 2, and 4
Topic Tag
PHASE I + II

India Energy Week 2024 ESI – Sustainable Development and


Environmental Issues
SWATI ESI – Social Sectors – Education/Social
Justice
PARAKH ESI – Social Sectors – Education/Social
Justice
ONDC ESI – Employment Generation

PHASE I
UPI Payment Services in Sri Lanka GA – Banking and Finance
RELEVANT FOR
PHASE 1 & PHASE 2
About SWATI Portal
• Launched on the International Day of Women and Girls in Science.
• Developed, hosted, and maintained by: National Institute of Plant Genome
Research (NIPGR), New Delhi.
• Funded by: Department of Biotechnology.
• Aim: To serve in policymaking by addressing the challenges of the gender gap in
STEMM.
• The portal is interactive, first-of-its-kind in the world.
• The initiative is part of the Inter Academy Panel (IAP) on Women in Science,
Technology, Engineering, Mathematics & Medicine (STEMM).
• Detailed discussion took place about the ‘Academic
Bank of Credit’ strengthening the ‘Competency-
Based Framework’ through the use of ‘Holistic
Progress Cards (HPCs)’.
• HPCs are visualized as teacher training tools and
make the teacher aware of the grade-specific
competencies and their related assessments.
1st State Educational Achievement
Survey conducted in November, 2023,
assessed learners from Grades 3, 6, and 9
across 30 States/UTs, covering
approximately 8 million learners.
• The survey aims to gauge
competencies in foundational
literacy, numeracy, language, and
mathematics.
• Barring Chhattisgarh, Delhi, Odisha,
Punjab, Rajasthan and West Bengal,
all other States and UTs participated
in the implementation of the survey.
• Pai Platforms has acquired Bitsila,
a seller platform on the Open
Network for Digital Commerce
(ONDC), enhancing its presence in
the online retail sector.
• Bitsila, launched in 2020, is
recognized among the top 3 seller
platforms on ONDC, managing over
600 million product categories
across more than 10,000 stores in
over 30 cities.
RELEVANT FOR
PHASE 1
About UPI
• An Indian instant payment system
developed by the National Payments
Corporation of India (NPCI) in 2016.
• Designed to consolidate multiple bank
accounts into a single mobile
application.
Let’s Practice
ESI SUSTAINABLE DEVELOPMENT AND ENVIRONMENTAL ISSUES

Q1. Regarding India's energy and environmental initiatives, identify the correct
combination of statements based on the provided information:
1. The Global BioFuel Alliance has economic opportunities estimated at $500 billion,
involving 22 nations and 12 international organizations.
2. India's commitment to establishing 5,000 compressed biogas (CBG) plants is part
of its 'waste to wealth' initiatives.
3. The "Indian Oil Market Outlook to 2030" report by the International Energy Agency
(IEA) predicts India will be the largest source of global oil demand growth by 2030.
4. India's LPG imports have tripled over the last decade, reflecting the country's
growing energy needs.
5. The plan to increase India's oil refining capacity to 450 MMTPA by 2030 is in
response to the anticipated doubling of India's energy demand.
[A] 1, 2, and 3
[B] 2, 3, and 4
[C] 1, 3, and 5
Answer: Option E
[D] 1, 2, 4, and 5
[E] All of the above
ESI SOCIAL SECTORS - EDUCATION

Q2. Consider the following statements about the SWATI Portal and identify which one is
correct:
[A] The portal was launched on National Science Day to support women in various
scientific disciplines.
[B] It is developed, hosted, and maintained by the Indian Institute of Science Education
and Research (IISER), funded by the Ministry of Science and Technology to enhance
women's participation in science.
[C] It aims to bridge the gender gap in STEMM (Science, Technology, Engineering,
Mathematics & Masters in Business Administraton).
[D] It aimed at informing policymaking to address the gender gap in STEMM fields, and
is supported by the Department of Biotechnology.
[E] The SWATI Portal, funded by UNESCO, is designed to facilitate the global exchange
of information and resources among women scientists and researchers, launched during
the World Science Forum.

Answer: Option D
ESI SOCIAL SECTORS - EDUCATION

Q3. Regarding PARAKH's establishment and its primary objectives, evaluate the following
statements and select the correct combination:
1. PARAKH was established in March 2023, following the National Education Policy 2020, to act as
an independent body under the Ministry of Education.
2. The primary aim of PARAKH is to implement competency-based assessments across all
educational stages, facilitated by Project Vidyasagar's initiative to update educators on
pedagogical reforms.
3. The Educational Testing Service (ETS) is designated as the implementing agency for PARAKH,
working under the direct supervision of Central Board of Secondary Education.
4. A significant function of PARAKH includes conducting large-scale achievement surveys, with
the inaugural state survey assessing foundational literacy and numeracy among students in
grades 3, 6, and 9.
5. PARAKH's efforts to ensure the equivalence of school boards involve creating a unified
assessment framework across states and UTs.
[A] 1, 2, and 4
[B] 1, 3, and 5
[C] 2, 4, and 5
[D] All except 3 Answer: Option D
[E] 1, 2, 3, and 5
ESI Homework Question EMPLOYMENT GENERATION

Q4. Examine the following statements regarding the ONDC and select the correct
combination:
1. ONDC, a brainchild of DPIIT, was established in 2021 and launched its pilot in 2022,
aiming to transform the landscape of digital commerce through inclusivity and
decentralization.
2. It is a Section 10 company that ensures increased choices for consumers by
promoting interoperability through the ONDC Protocol.
3. The ONDC Registry, integral to the network's infrastructure, functions to connect
various compliant platforms, enhancing the discoverability akin to a digital
phonebook.
4. The pilot phase of ONDC was initiated across a Delhi and Bengaluru only.
[A] 1 and 3
[B] 1, 2, and 3
[C] 2, 3, and 4
[D] 1, 3, and 4
[E] All of the above
HOMEWORK QUESTIONS
FROM PREVIOUS SESSION
ESI Homework Question GOVERNMENT SCHEMES

Q4. Consider the following statements about the Pradhan Mantri Matsya Kisan Samridhi
Sah-Yojana (PM-MKSSY):
1. The scheme aims to support 6.4 lakh micro-enterprises and 5,500 fisheries cooperatives
by providing access to institutional credit.
2. A one-time incentive for purchasing aquaculture insurance is provided under the
scheme.
3. The maximum incentive payable for aquaculture insurance is Rs.1,00,000 for farms with
up to 4 hectares of water spread area.
4. The National Fisheries Sector Digital Platform (NFDP) is designed for the self-
registration of fishers, fish farmers, and support workers.
Which of the statements given above are correct?
[A] 1, 2, and 3 only
[B] 2, 3, and 4 only
[C] 1, 2, and 4 only
[D] 1, 3, and 4 only Answer: Option E
[E] All of the above
Topic Tag
PHASE I + II

Electoral Bond Scheme ESI – Government Schemes

Surya Ghar Muft Bijli Yojana ESI – Government Schemes

ULPIN ESI – Human Development

APAAR ESI – Social Sectors - Education

PHASE I
SUFALAM Campaign GA – First In News
India – UAE Agreements GA – MOUs and Agreements
RELEVANT FOR
PHASE 1 & PHASE 2
Key Highlights
• The scheme violates Article 19(1)(a) of
constitution, freedom of speech and expression.
• SBI ordered to stop issuing Electoral bonds immediately.
• SBI to furnish details of the political parties that received
Electoral Bonds since April 12, 2019, and all the
particulars received and submit them to the Election
Commission of India
About Electoral Bond Scheme
• Launch: 2018

• Authorized Agency: State Bank of India

• Valid for 15 calendar days from date of issue and no payment shall be made to any payee
Political Party if Bond is deposited after expiry of validity period.

• Electoral Bond deposited by an eligible Political Party in its account credited on same day.

• May be purchased by a person who is a citizen of India or incorporated or established in


India.

• One can buy Bonds, either singly or jointly with other individuals.

• Only Political Parties registered under Section 29A of Representation of the People Act,
1951

o which secured not less than 1% of votes polled in last General Election to House of
People or Legislative Assembly of State, shall be eligible to receive Bond.
An interest free bearer instrument, do not carry name of payee and can be bought for any
value, in multiples of ₹1,000, ₹10,000, ₹1 lakh, ₹10 lakh or ₹1 crore.
• Launch – 2024
• Nodal Ministry - Ministry of New & Renewable Energy
• Aim - To light up 1 crore households by providing up to 300 units of free
electricity every month.
• Investment - over ₹75,000 crores
Key Features:
• Subsidies - will be given directly to bank accounts.
• Heavily concessional bank loans to ensure that there is no cost burden on people.
• To popularize scheme at grassroots:
o Urban Local Bodies and Panchayats shall be incentivised to promote rooftop
solar systems in their jurisdiction.
• All stakeholders will be integrated to a National Online Portal
• Benefits:
o more income, lesser power bills and employment generation for people
Information about scheme as stated by Union Minister for Power and New and Renewable Energy:

• Those who availed low-income houses under Pradhan Mantri Awas Yojna will be considered in
this scheme.
• Government has appointed 8 central public sector undertakings (CPSUs) to implement scheme
across all states and union territories of country.
o including NTPC, NHPC, EESL, PowerGrid, Grid-India, THDC, SJVN and NEEPCO
• Government will increase existing subsidy for rooftop solar to 60% from current 40% which will
be given to CPSUs.
o Remaining 40% of funding through loans which CPSUs will take from institutions like REC
Ltd, which is also nodal agency for programme.
• Loan will be taken by special purposed vehicles (SPV) set by CPSUs assigned for a particular
state.
• 300 units generated through rooftop solar will offset monthly electricity bill of that household:
o excess unit of solar power that will be generated, will go to pay off loan taken by CPSUs.
• Such households will not have to pay for solar installations on their rooftops.
• Loan tenure is expected to be up to 10 years:
o Post 10-year period when loan is repaid, rooftop solar infrastructure would be transferred
to household,
▪ which would then themselves be able to sell their excess power to discoms.
• Households with power consumption more than 300 units per month will have to install rooftop
solar on their own, even as they will be eligible for enhanced subsidies.
• National Generic Document Registration
System (NGDRS) part of the One Nation
One Software initiative, is working
towards digitalization and standardization
of land records.
• Developed by: National Informatics
Centre (NIC), Pune.
About Automated Permanent Academic Account
Registry (APAAR)
• Nodal Ministry: Ministry of Education
• Implemented by: National Educational Technology Forum
(NETF)
• Features
o Dubbed as the 'One Nation, One student ID' is an
Education Ecosystem Registry or an 'EduLocker'.
o It is registry of entire educational ecosystem, comprising
students, teachers, and schools and colleges.
o It will give each student enrolled from pre-primary to
higher education, a unique identification number (UIN)
of 12 digit in addition to the already existing Aadhaar
ID that every individual has.
o It will be a lifelong ID number for students for seamless
tracking of their academic journey and achievements.
o It will be able to digitally store their exam results,
learning outcomes, co-curricular achievements such
as ranking in Olympiads or receiving specialized skill
training and much more.
o Students transferring from one school to another will
face less hassle in obtaining admission to a new
institution in any part of the country.
o Target – All student records to be mapped on to
APAAR by 2026-27.
o Enrolment process
✓ Carried out by the school, with the consent of their
parents, who will also be able to withdraw their
consent at any given point of time.
✓ Data will be shared only with concerned government
agencies if and when required.
✓ The data collected on each student by the schools
will be stored in centrally functioning District
Information for Education portal.
RELEVANT FOR
PHASE 1
• Organizing Body: Ministry of Food Processing Industries
(MoFPI)
• Key Facts:
• Aimed at enhancing networking, facilitating knowledge
sharing, and enabling startups to leverage various
government schemes.
Important Announcements
• India's UPI will be linked with UAE's AANI, and India's RuPay cards with UAE's JAYWAN card.
• Inauguration of BAPS (Bochasanwasi Shri Akshar Purushottam Swaminarayan Sanstha)
Swaminarayan Temple in Abu Dhabi.
o BAPS temple is the first Hindu temple in the UAE.
• Agreements signed by Rites Ltd and Gujarat Maritime Board with Abu Dhabi Ports Co. to
develop port infrastructure.
• MoU for cooperation in the field of heritage and museums to support the Maritime Heritage
Complex at Lothal, Gujarat.
• Bilateral Investment Treaty was signed.
• An Intergovernmental Framework Agreement was signed on the India-Middle East Economic
Corridor.
• Sovereign Wealth Fund Investment: Abu Dhabi Investment Authority’s (ADIA) significant
investment in India’s National Infrastructure Investment Fund (NIIF).
UAE’s relation with India
• 4th-largest source of FDI in India at $3.5 billion.
• Second-largest export destination
• Third-largest trading partner.
Let’s Practice
GA MOUs and Agreements

Q1. Select the correct combination of statements based on the recent developments in
India-UAE ties.
1. India's UPI will be linked with UAE's AANI, facilitating seamless financial
transactions between the two countries.
2. The first Hindu temple inaugurated in the UAE is the BAPS Swaminarayan Temple
in Abu Dhabi.
3. A Bilateral Investment Treaty was signed to boost economic cooperation but does
not include provisions for infrastructure development.
4. An Intergovernmental Framework Agreement on the India-Middle East Economic
Corridor aims at enhancing trade routes and connectivity.
5. The Ministry of Electronics and Information Technology announced the opening of
a new CBSE office in Dubai.
[A] 1, 2, and 4
[B] 1, 3, and 5
[C] 2, 3, and 4
Answer: Option A
[D] 1, 2, and 5
[E] All of the above
ESI SOCIAL SECTORS - EDUCATION

Q2. Which of the following statement accurately describe the APAAR: One Nation One
Student ID Card initiative?
[A] APAAR was launched in 2020 by the Ministry of Electronics and Information
Technology.
[B] APAAR IDs are meant to be temporary and need to be renewed every academic year.
[C] The APAAR initiative aims to provide a unique and permanent 12-digit ID to every
student in India, integrating with digital platforms like DigiLocker and Academic Bank of
Credit.
[D] Only students in higher education are eligible for an APAAR ID.
[E] All of the above

Answer: Option C
ESI SOCIAL SECTORS - EDUCATION

Q3. Recently (February 2024), Smt. Nidhi Khare rolled out National Generic
Document Registration System (NDGRS) throughout Assam along with the launch
of Unique Land Parcel Identification Number (ULPIN) seeding of geo referenced
Cadastral Maps. Special Chief Secretary, Government of Assam Shri Syedain
Abbasi launched rollout of Blockchain in land records in Darrang district as a pilot
today. The event was organized by the Revenue and Disaster Management
Department, Government of Assam.
Identify the correct statement(s) regarding the Unique Land Parcel Identification
Number (ULPIN)?
[A] ULPIN was rolled out in 2020 to ensure the digitalization of land records across
India.
[B] The implementation agency for ULPIN is the Ministry of Electronics and
Information Technology.
[C] ULPIN is a 14-digit alphanumeric number used to uniquely identify every land
parcel in India.
[D] ULPIN aims to decrease transparency and efficiency in land transactions and
management.
[E] None of the above Answer: Option C
GA FIRST IN NEWS

Q4. SUFALAM “Start-Up Forum for Aspiring Leaders and Mentors Startup Conclave
2024” campaign in February 2024 has highlighted the significance of emphasizing
their capacity to enhance networking, facilitate knowledge exchange, and enable
startups to capitalize on various government schemes for the food processing
sector.
Which ministry is responsible for organizing the Start-Up Forum for Aspiring
Leaders and Mentors (SUFALAM) Startup Conclave 2024?
[A] Ministry of Commerce and Industry
[B] Ministry of Skill Development and Entrepreneurship
[C] Ministry of Electronics and Information Technology
[D] Ministry of Food Processing Industries
[E] Ministry of Small and Medium Enterprises

Answer: Option D
ESI GOVERNMENT SCHEMES

Q5. Supreme Court has recently (February 2024), held the changes made in the
law to introduce the __X__ scheme as unconstitutional, in a unanimous verdict on a
batch of pleas challenging the legal validity of the Centre’s scheme which allowed
for anonymous funding to political parties.
Identify the correct statement with respect to the scheme discussed in the above
passage?
[A] It is Valid for 30 calendar days from date of issue.
[B] Electoral Bond deposited by an eligible Political Party in its account credited on
same day.
[C] One cannot buy Bonds jointly with other individuals.
[D] Reserve Bank of India is authorized agency to deal with these bonds.
[E] All of the above

Answer: Option B
ESI Homework Question EMPLOYMENT GENERATION

Q6. Reflecting on the statement made by the Union Minister for Power and New and
Renewable Energy regarding the "PM Surya Ghar Muft Bijli Yojana," identify the
correctness of the following statements:
1. 10 central public sector undertakings (CPSUs) including NTPC, NHPC, and
PowerGrid are tasked with the scheme's implementation.
2. The excess solar power generated by households will be used to offset the loan
taken by CPSUs for solar installations.
3. Government will increase existing subsidy for rooftop solar to 60% from current 40%
which will be given to CPSUs.
[A] 1 and 2
[B] 2 and 3
[C] Only 2
[D] Only 3
[E] All of the above
HOMEWORK QUESTIONS
FROM PREVIOUS SESSION
ESI Homework Question EMPLOYMENT GENERATION

Q4. Examine the following statements regarding the ONDC and select the correct
combination:
1. ONDC, a brainchild of DPIIT, was established in 2021 and launched its pilot in 2022,
aiming to transform the landscape of digital commerce through inclusivity and
decentralization.
2. It is a Section 10 company that ensures increased choices for consumers by
promoting interoperability through the ONDC Protocol.
3. The ONDC Registry, integral to the network's infrastructure, functions to connect
various compliant platforms, enhancing the discoverability akin to a digital
phonebook.
4. The pilot phase of ONDC was initiated across a Delhi and Bengaluru only.
[A] 1 and 3
[B] 1, 2, and 3
[C] 2, 3, and 4 Answer: Option A
[D] 1, 3, and 4
[E] All of the above
Topic Tag
PHASE I + II

FCI ESI – Indian Agriculture

Jal Jeevan Mission ESI – Government Schemes

PHASE I
Kaji Nembu ARD Current Affairs
IREDA-PNB Agreement GA – MOUs and Agreements
Innovative Technology Award GA - Awards
R. Ashwin GA – Persons in News
RELEVANT FOR
PHASE 1 & PHASE 2
• Aim: to enhance FCI's operational capabilities in procuring food
grains at Minimum Support Price (MSP), maintaining strategic
food grain stocks, and stabilizing market prices.
• It plans to modernize storage facilities, improve transportation, and
adopt advanced technologies to reduce post-harvest losses and
ensure efficient distribution.
About Food Corporation of India
• Established under the Food Corporation Act 1964.
• Objectives: effective price support operations to protect the
interests of farmers, distribution of food grains throughout
the country for the public distribution system and national to
maintain satisfactory level of circulation and buffer stock of
food grains to ensure food security.
• The top priority of FCI is to ensure food security for the
nation by effectively managing the procurement, storage and
distribution of essential commodities, thereby providing reliable
and adequate food supply to meet the needs of the population.
▪ Launch – 2019
▪ Ministry of Jal Shakti
▪ Objective –
o To provide Functional Household Tap Connection
(FHTC) to every rural household
o To provide functional tap connection to Schools,
Anganwadi centres, GP buildings, Health centres,
wellness centres and community buildings
▪ Type: Centrally Sponsored Scheme
▪ 90:10 - Himalayan (Uttarakhand,Himachal Pradesh), N-E
States, Union Territories with Legislature
▪ 100:0 - Centre: Union Territories without Legislature
▪ 50:50 - Other States
▪ Estimated Budget - ₹3.60 Lakh Crore
▪ Target Year – 2024
Components
▪ Development of in-village piped water supply infrastructure to provide
tap water connection to every rural household
▪ Development of reliable drinking water sources and/ or augmentation of
existing sources to provide long-term sustainability of water supply
system
▪ Wherever necessary, bulk water transfer, treatment plants and
distribution network to cater to every rural household
▪ Technological interventions for removal of contaminants where water
quality is an issue
▪ Retrofitting of completed and ongoing schemes to provide FHTCs at
minimum service level of 55 lpcd
▪ Greywater management-treatment and reuse
▪ Support activities, training, development of utilities, water quality
laboratories, water quality testing & surveillance, R&D etc
▪ Any other unforeseen challenges/ issues emerging due to natural
disasters/ calamities which affect goal of FHTC
Key Features
• National Rural Drinking Water Programme (NRDWP) restructured and subsumed
under JJM.
• All households, schools and anganwadis in village should have tap water before
village is certified as ‘Har Ghar Jal’
• Institutional Framework: 4-tier at National, State, District and Village level
• Gram Panchayat and/or its subcommittee, i.e. Village Water and Sanitation
Committee (VWSC)/ Paani Samiti/ User Group
• Consist of 10-15 members comprising elected members of Panchayat up to
25% of composition
• 50% women members ; and remaining 25% may consist of representatives of
weaker sections of village (SC/ST) proportional to their population
• To mobilize and motivate community to contribute 5% or 10% of in-village
infrastructure capital expenditure
• Contribution may be in form of cash and/ or kind and/ or labour
• To ensure preparation of Village Action Plan (VAP) - having 3 components
Water source & its maintenance, Water supply, Greywater (domestic
wastewater) management
Criteria for allocation of fund

• Rural Population (as per last Census) - 30%

• Rural SC and ST populaon (as per last Census) - 10%

• States under DDP, DPAP, HADP and special category Hill States
in terms of rural areas - 30%

• Population residing in habitaons affected by chemical


contaminants including heavy metals (as on 31 March of
preceding FY) - 10%

• Weightage for balance individual household connections - 20%


RELEVANT FOR
PHASE 1
• Kaji nemu is declared as state fruit of Assam.
• Botanical name - citrus limon.
• It got GI-certification in 2019.
• Purpose of the MoU: To advance renewable energy initiatives across
India through joint efforts in co-lending, loan syndication for renewable
energy projects, and investment in bonds issued by either organization.
• Aim: To provide robust financial support to a diverse range of renewable
energy projects, in alignment with India's commitment to achieving a 500
GW Non-Fossil-based electricity generation capacity by 2030.
About REC Limited: REC Limited has been
recognized for its CSR
• Status: Maharatna Central Public Sector Enterprise activities with several
• Ministry: Under the Ministry of Power, Government of India prestigious awards,
including the Global CSR
• Nature: Leading Non-Banking Finance Company (NBFC) and
Leadership Awards in
Infrastructure Financing Company (IFC). 2023 and the PSE
• REC has been instrumental in the implementation of flagship Excellence Award for CSR
in 2023.
government schemes for the power sector, such as: Pradhan Mantri
Sahaj Bijli Har Ghar Yojana (SAUBHAGAYA), Deen Dayal Upadhyaya
Gram Jyoti Yojana (DDUGJY), and the Revamped Distribution Sector
Scheme (RDSS), significantly contributing to village electrification,
household electrification, and strengthening of the last-mile
distribution system in the country.
• He became only the ninth man to reach the 500-
milestone.
• He is the only the seventh spinner in Test history
to take 500 or more wickets.
• He is the only the second Indian bowler to reach
500 wickets in Tests after Anil Kumble.
Let’s Practice
ESI INDIAN AGRICULTURE

Q1. Consider the following statement with respect to Food Corporation of India and
choose the incorrect statement:
1. Recently (February 2024), government has increased the authorized capital of FCI
from 10,000 to 21,000 crores.
2. It is established under the Food Corporation Act 1965.
3. It was made on the recommendation of MS Swaminathan Committee.
[A] Only 1
[B] Only 3
[C] Only 2
[D] 2 and 3
[E] 1 and 3

Answer: Option D
ARD ARD – Current Affairs

Q2. Recently (Feb 2024), Kaji nemu is declared as state fruit of Assam. It got
the GI tag from the government of Assam in which of the following year?
[A] 2016
[B] 2017
[C] 2018
[D] 2020
[E] 2019

Answer: Option E
GA AWARDS

Q3. Recently (February 2024), REC Limited has received 'Innovative


Technology Development Award' at IIT Madras CSR Summit for its 2 MW
Rooftop Solar Plant at IIT.Which among the following scheme is/are
implemented by REC Limited?
1. Pradhan Mantri Sahaj Bijli Har Ghar Yojana (SAUBHAGAYA)
2. Deen Dayal Upadhyaya Gram Jyoti Yojana (DDUGJY)
3. Revamped Distribution Sector Scheme (RDSS)
A. 1 and 2
B. 2 and 3
C. 1 and 3
D. 1, 2 and 3
E. None of the Above Answer: Option D
GA PERSONS IN NEWS

Q4. Which among the following Indians have achieved the milestone of achieving
500 wickets in the Test Match?
1. Anil Kumble
2. Kapil Dev
3. Harbhajan Singh
4. Ravichandran Ashwin
5. Zaheer Khan
A. 1 and 3
B. 1, 2, 3 and 4
C. 1 and 4
D. All of the Above
E. 1, 2 and 4

Answer: Option D
ESI Homework Question GOVERNMENT SCHEMES

Q5. Which of the following statement(s) is/are correct with respect to Jal Jeevan
Mission (JJM)?
1. Community needs to contribute 5% or 10% of capital cost for in-village
infrastructure as the case may be, in cash/ kind/ and/ or labour.
2. Every village to prepare a Village Action Plan (VAP) which will have two
components.
3. Technological interventions to be provided for removal of contaminants where
water quality is an issue.
4. Greywater (industrial wastewater) management is also covered under Village
Action Plan (VAP).
A. 1, 2 and 3
B. 2 and 3
C. 1 and 3
D. 1, 2 and 4
E. 2, 3 and 4
HOMEWORK QUESTIONS
FROM PREVIOUS SESSION
ESI Homework Question EMPLOYMENT GENERATION

Q6. Reflecting on the statement made by the Union Minister for Power and New and
Renewable Energy regarding the "PM Surya Ghar Muft Bijli Yojana," identify the
correctness of the following statements:
1. 10 central public sector undertakings (CPSUs) including NTPC, NHPC, and
PowerGrid are tasked with the scheme's implementation.
2. The excess solar power generated by households will be used to offset the loan
taken by CPSUs for solar installations.
3. Government will increase existing subsidy for rooftop solar to 60% from current 40%
which will be given to CPSUs.
[A] 1 and 2
[B] 2 and 3
[C] Only 2
[D] Only 3 Answer: Option B
[E] All of the above
Topic Tag
PHASE I + II

First Skill India Centre ESI – Employment Generation

Flood Management and Border Area Program ESI – Government Schemes

Scheme on Safety of Women ESI – Government Schemes

PHASE I
First Nort East State to provide Universal GA – First In News
Health Coverage
IBSA Fund GA – Miscellaneous
Khelo India Winter Games GA – Sports
RELEVANT FOR
PHASE 1 & PHASE 2
• Inaugurated in Sambalpur, Odisha.

About the First Skill India Centre of India


• Ministry: Ministry of Skill Development & Entrepreneurship.
• Implementing Agency: National Skill Development Corporation
(NSDC).
• Aim/Objective: To equip over 1200 students with employable
skills in demand-driven trades, thereby creating an industry-
ready workforce and promoting entrepreneurship.
• Features:
• It focuses on high-demand trades like Media &
Entertainment, Leather, Tourism & Hospitality, and IT-ITeS,
providing a mix of classroom and work-based learning.
• A Centre Manager, designated by the NSDC, will ensure the
seamless implementation of training programs and
adherence to quality standards.
About National Skill Development Corporation

• A not-for-profit public limited company, incorporated in


2008, under section 8 of the Companies Act, 2013.
• Public-Private Partnership Model:
o Initially set up by the Ministry of Finance and now
functioning under the Ministry of Skill Development &
Entrepreneurship (MSDE).

• Aims and Objectives: To promote skill development across


the country by catalyzing the creation of large, quality, for-
profit vocational institutions
Flood Management and Border Areas Programme (FMBAP)

• First Phase of FMBAP: 2017-18 to 2019-20


• Now Extended Tenure: 2021-26 (15th Finance Commission
period).

• Total outlay for the next 5 years: ₹4,100 crore.

• Implementing Agency: Department of Water Resources


(Ministry of Jal Shakti).

• The Scheme has the provision of incentivizing the States which


implement flood plain zoning, recognized as an effective non-
structural measure for flood management.
TWO COMPONENTS

Flood Management Programme (FMP)


River Management Activities and Works Related to
• Origins: Started during the XI Plan. Border Areas (RMBA)
• FMP focuses on critical works in flood control, • Origins: Started during the XI Plan.
anti-erosion, drainage development, and anti-sea
• Activities: Hydrological observation, flood
erosion, among others.
forecasting, international cooperation, and
• Funding pattern for FMP among Centre and structural measures, maintenance of flood
States: protection works and anti-erosion works on
• 90:10 ratio for Special Category States (8 international borders.
North-Easter States and Hilly States of • Funding: 100% central assistance.
Himachal Pradesh, Uttarakhand and UT of
Jammu & Kashmir)
• 60:40 for General/Non-Special Category
States.
• Criteria for Projects: Cost above ₹10 crore for
Special Category States and ₹40 crore for
General States with Benefit-Cost ratio >1.0.
• Approval for the period of 2021-22 to 2025-26 ( aligning with 15th Finance Commission
period).
Key Highlights
• Objective: To enhance the safety of women across India.
• Funding: Total (Rs.1179.72 crore), Ministry of Home Affairs (MHA) will contribute Rs.885.49
crore, with the remaining Rs.294.23 crore funded from Nirbhaya Fund.
• Key projects under the scheme include:
• 112 Emergency Response Support System (ERSS) 2.0;
• Upgradation of Central Forensic Sciences laboratories, including setting up of National
Forensic Data Centre;
• Strengthening of DNA Analysis, Cyber Forensic capacities in State Forensic Science
Laboratories (FSLs);
• Cyber Crime Prevention against Women and Children;
• Capacity building and training of investigators and prosecutors in handling sexual
assault cases against women and children; and
• Women Help Desk & Anti-human Trafficking Units.
RELEVANT FOR
PHASE 1
About CM Jan Arogya Yojana (CMJAY)
• Aim - To provide free healthcare services to indoor patients up
to primary health centre level and empanelled private hospitals
of the state and outside.
Key Features
• It will provide health insurance of Rs 5 Lakh per family
annually.
• The whole process would be cashless and patients under this
scheme would get free medicines from the hospital for 15 days
on being discharged after any critical illness
About IBSA Fund
• The IBSA facility for poverty and hunger alleviation (IBSA Fund) was
established jointly by India, Brazil and South Africa in March 2004.
• It became operational in 2006 to identify scalable projects that can be
disseminated to developing countries on a demand driven basis as
examples of best practices in combating poverty and hunger.
About IBSA
• The grouping was formalized and named the IBSA Dialogue Forum when the
Foreign Ministers of the three countries met in Brasilia in June 2003 and
issued the Brasilia Declaration.
• Mascot: Snow leopard, named ‘Sheen-e She’ or Shan.
• Logo: Features the Indian tricolor and a Dharmachakra on a hilltop in
Chanspa, Leh.
• Ministry: Union Ministry for Youth Affairs & Sports.
• Hosts: Union Territories of Ladakh (1st time) and Jammu & Kashmir.

As per Snow Leopard Population Assessment in India (SPAI) Program, the


population of snow leopard resides in Ladakh (477), Uttarakhand (124),
Himachal Pradesh (51), Arunachal Pradesh (36), Sikkim (21), and Jammu and
Kashmir (9).
National Co-Ordinator for the Program - Wildlife Institute of India (WII)
Let’s Practice
ESI EMPLOYMENT GENERATION IN INDIA

Q.1) Recently in February 2024, significant strides were made in skill development with
the inauguration of the country's first Skill India Centre. Considering the information
provided, which of the following statements are true?
1. The Skill India Centre was launched in Sambalpur, Odisha.
2. The Ministry of Education is responsible for the launch of the Skill India Centre.
3. The National Skill Development Corporation (NSDC) along with World Skill
Development Centre are involved in the implementation of the Skill India Centre.
4. Over 1200 students will be equipped with employable skills in sectors like
Agriculture and Engineering.
5. Media & Entertainment, Leather, Tourism & Hospitality, and IT-ITeS are the focus
trades of the Skill India Centre.
[A] 1 and 2
[B] 1, 2, and 5
[C] 2, 3, and 4 Answer: Option D
[D] 1 and 5
[E] None of the above
ESI GOVERNMENT SCHEMES

Q2. Recently in February 2024, the Union Cabinet approved the continuation
of the Umbrella Scheme on "Safety of Women" with significant funding to
enhance women's safety across India. Which of the following statements is
correct regarding the key aspects of this scheme?
[A] The entire funding for the scheme will be provided by the Ministry of
Women and Child Development.
[B] The scheme solely focuses on the capacity building and training of
investigators and prosecutors in handling cases against women, with no
allocation for the upgradation of forensic capacities or emergency response
systems.
[C] Extended tenure for the scheme is from 2021 to 2026, aligning with 16th
Finance Commission period.
[D] Both [B] and [C]
Answer: Option E
[E] None of the above
GA SPORTS

Q3. Recently in February 2024, the 4th Edition of the Khelo India Winter
Games was announced, showcasing a range of winter sports and cultural
heritage. With regards to this event, which of the following statements are
true?
1. The mascot for the games is a snow leopard named ‘Sheen-e She’ or Shan.
2. The logo of the games depicts the Indian tricolor and a Dharmachakra
positioned on a hilltop in Chanspa, Leh.
3. The event is exclusively hosted by the Union Territory of Ladakh for the
first time, without the involvement of Jammu & Kashmir.
[A] Only 1 and 2
[B] Only 2 and 3
[C] 1, 2, and 3
[D] Only 1 Answer: Option A
[E] None of the above
GA MISCELLANEOUS

Q4. Recently in February 2024, Government of India has contributed US Dollar


one million to the India, Brazil and South Africa Facility for Poverty and Hunger
Alleviation Fund (IBSA Fund). Which among the following statement is/are
correct with respect to IBSA?
[A] It was formed under Brasilia in June 2003.
[B] IBSA Fund was established in March 2004
[C] IBSA became operational in 2006
[D] A, B and C
[E] A and B

Answer: Option E
ESI Homework Question GOVERNMENT SCHEMES

Q5. Recently in February 2024, the Union Cabinet approved the continuation of the Flood
Management and Border Areas Programme (FMBAP) for the period 2021-2026. Regarding
the details of FMBAP and its related aspects, which of the following statements are true?
1. FMBAP was initiated by merging "Flood Management Programme (FMP)" and "River
Management Activities and Works related to Border Areas (RMBA)".
2. The scheme primarily places the responsibility of flood management on the Union
Government, supplementing state efforts.
3. FMP and RMBA were both started during the XII Plan.
4. The funding pattern for FMP among Centre and States is 75:25 for General/Non-Special
Category States.
5. The scheme includes a provision for incentivizing states that implement flood plain
zoning as a non-structural measure for flood management.
[A] 1 and 5
[B] 1, 3, and 5
[C] 2, 4, and 5
[D] All of the above
[E] None of the above
HOMEWORK QUESTIONS
FROM PREVIOUS SESSION
ESI Homework Question GOVERNMENT SCHEMES

Q5. Which of the following statement(s) is/are correct with respect to Jal Jeevan
Mission (JJM)?
1. Community needs to contribute 5% or 10% of capital cost for in-village
infrastructure as the case may be, in cash/ kind/ and/ or labour.
2. Every village to prepare a Village Action Plan (VAP) which will have two
components.
3. Technological interventions to be provided for removal of contaminants where
water quality is an issue.
4. Greywater (industrial wastewater) management is also covered under Village
Action Plan (VAP).
A. 1, 2 and 3
B. 2 and 3
C. 1 and 3 Answer: Option C
D. 1, 2 and 4
E. 2, 3 and 4
Topic Tag
PHASE I + II

Swachh Bharat Mission - Urban 2.0 (SBM-U ESI – Government Scheme


2.0)
PHASE I
1st Moon landing by private vehicle GA – Science & Technology
India’s 1st Gati Shakti Research Chair GA – Miscellaneous
India’s largest tribal festival GA – Fairs & Festivals
MILAN 2024 GA – Defence
Maharashtra Former CM GA – Person in News
International Mother language Day GA – Important Days
RELEVANT FOR
PHASE 1
• Odysseus, a spacecraft built by Intuitive Machines, used a
Falcon 9 rocket of SpaceX to take off from Earth.
• It carried six NASA payloads to the Moon.
• The lander module of Odysseus, called Nova-C, has become
the second one, after Chandrayaan-3 last year, to land in the
Moon’s south pole region.
• It marks the first successful landing of a commercial
spacecraft on the moon

• 1st moon landing, by Luna 9 of the Soviet Union,


happened in 1966
• India’s first ‘Gati Shakti Research Chair’ established at IIM Shillong.
• MoU has been signed between the Ministry of Ports, Shipping and Waterways and the
Indian Institute of Management (IIM) Shillong.
• It will spearhead high-quality academic research on multimodal logistics with a North-
East focus:
❑ building knowledge on multimodal logistics development strategies and practice in the
North-Eastern region
❑ facilitate knowledge and innovation for logistical capacity building in the region.
• It aims to foster collaboration with global experts in creating and developing knowledge
& expertise while strengthening the links between the multimodal logistics research
and capacity-building activities with key stakeholders.
About Medaram Jatara
• The biennial tribal festival is the second-largest fair of India,
after the Kumbh Mela.
• It is celebrated by the Koya tribe.
• Celebrated in collaboration with the Tribal Welfare
Department, Government of Telangana
• Jatara was declared a State Festival in 1996
• It helps the tribesmen communicate their tribal history
globally and preserve their distinctive tribal traditions, culture,
and heritage.
About Milan 2024
• Indian Navy’s largest-ever multilateral naval exercise - Milan
2024 Multilateral Naval exercise 'Milan', in Visakhapatnam.
• Navies from the US, Japan, Australia, France, Bangladesh,
South Korea, Vietnam, Indonesia and Malaysia, among others,
are participating in the 12th edition of the 'Milan' exercise
About Manohar Joshi
• Shri. Manohar Joshi was also Lok Sabha speaker from 2002 to
2004.
Theme: Multilingual education – a pillar of learning and
intergenerational learning

Note:

• Currently, 40% of the global population lacks access to


education in their native language, a figure that exceeds 90% in
certain regions.

• At least 45% of the estimated 7000 languages spoken in the


world are endangered.

Exhibiion: Akshar | Shabd | Bhasha


RELEVANT FOR
PHASE 1 & PHASE 2
Swachh Bharat Mission - Urban 2.0 (SBM-U 2.0)
• Launch : 2021
❑ Swachh Bharat Mission-Urban was launched in 2014
• Aim: To make cities completely free of garbage
• Objectives
➢ Sustainable Solid Waste Management: phased reduction in use
of single-use plastic
➢ Sustainable Sanitation and treatment of used water
➢ Awareness creation along with large scale citizen outreach to
create jan andolan and institutionalize swachh behavior
➢ Create institutional capacity to effectively implement
programmatic interventions
• Type: Centrally Sponsored Scheme
Swachh Bharat Mission - Urban 2.0 (SBM-U 2.0)
• Tenure: From 2021-22 till 2025-26
• Financial Outlay: ₹1,41,600 crores, including central share of
₹36,465 crores
Funding Pattern
• Centre: State distribution of project fund
➢ 90%:10% for ULBs in NE/Himalayan States
➢ 100% for UTs without legislature
➢ 80%: 20% for UTs with legislature
➢ 25%: 75% for 10 lakh plus ULBs
➢ 33%: 67% for ULBs with 1 lakh to 10 lakh population
➢ 50%: 50% for ULBs with less than 1 lakh population
• Coverage: All statutory towns in India
Mission Components:
• Sustainable Solid Waste Management
❑ To make all cities clean and garbage free, with 100% scientific processing
of Municipal Solid Waste
❑ Components of Municipal Solid Waste
➢ Source Segregation, Door to Door Collection, Separate transportation,
Waste Processing, Bulk Waste Generators, User Fee
❑ Setting up of construction & demolition (C&D) waste processing facilities
and deployment of mechanical sweepers in National Clean Air Programme
(NCAP) cities and in cities with more than 5 lakh population
❑ Remediation of all legacy dumpsites and converting them into green zones
• Sustainable Sanitation
❑ To sustain Open Defecation Free status in all Statutory towns
❑ Eligible components
➢ construction of Individual Household Latrines (IHHL)
➢ construction of Community and Public Toilet (CT and PT) seats
➢ construction of urinals, along with retrofitting of insanitary toilets
Mission Components:
• Used Water Management (UWM) - new component
❑ To ensure that no untreated fecal sludge or used water is
discharged into environment
❑ All used water (including sewerage and septage, grey water and
black water) is safely contained, transported and treated, along with
maximum reuse of treated used water, in all cities with less than 1
lakh population
• Information, Education and Communication (IEC) / Behaviour Change
Communication (BCC)
❑ To ensure awareness creation along with large scale citizen
outreach
• Capacity Building (CB)
❑ To create institutional capacity to effectively implement
programmatic interventions
Let’s Practice
GA Fairs & Festivals

Q.1) Recently, Union Tribal Affairs Minister Shri Arjun Munda visited Tribal festival, the
Sammakka-Saralamma Jatara at Medaram in Telangana. Which of the following
statements about Medaram Jatara is correct?

[A] It is an annual festival celebrated by the Koya tribe in Telangana.


[B] Medaram Jatara is considered the largest tribal festival in India.
[C] The festival is celebrated without any association with the Tribal Welfare
Department, Government of Telangana.
[D] Medaram Jatara was declared a State Festival in 2006.
[E] The festival was primarily initiated to boost the economic development of the region.

Answer: Option B
GA Science & Technology

Q2. Consider the following statements about recent developments in lunar exploration and identify
the correct ones.

1. The Odysseus spacecraft, constructed by Intuitive Machines, was launched using SpaceX's
Falcon 9 rocket.
2. Odysseus carried a total of ten NASA payloads to the Mars.
3. The Nova-C lander module of Odysseus is the first commercial spacecraft to land on the Moon's
south pole region.
4. This mission marks the first time a government space agency has successfully landed on the
lunar surface.
5. Chandrayaan-3 was the first to land in the Moon’s south pole region, followed by Nova-C this
year.
[A] 1 and 3 only
[B] 1, 3, and 5 only
[C] 2 and 4 only Answer: Option B
[D] 1, 2, and 4 only
[E] 1, 4, and 5 only
ESI Googly Question Government Schemes

Q3. The Swachh Bharat Mission - Urban 2.0 (SBM-U 2.0) has been a critical step in
India's journey towards urban sanitation and cleanliness. Consider the following
statements about SBM-U 2.0 and identify the correct statements:

1. SBM-U 2.0 was launched in 2021 with the aim of making cities completely free of
garbage, with Sustainable Solid Waste Management as one of its components.
2. The mission is a Centrally Sponsored Scheme with a financial outlay of ₹2,00,000
crores, having an objective of awareness creation along with large scale citizen
outreach to create jan andolan and institutionalize swachh behavior.
3. The tenure of SBM-U 2.0 is from 2021-22 till 2025-26, with funding pattern between
Centre & State distribution of project fund to be 100% for UTs with legislature.
[A] 1 only
[B] 1 and 2 only
[C] 2 and 3 only
Answer: Option A
[D] 1 and 3 only
[E] All of the above
GA Days in News

Q4. In the context of International Mother Language Day, which is observed to promote
linguistic and cultural diversity and multilingualism, consider the following statements:

1. The theme for International Mother Language Day was "Multilingual education – a
pillar of learning and intergenerational learning."
2. Approximately 40% of the global population does not have access to education in a
language they speak or understand.
3. Out of the estimated 7000 languages spoken worldwide, at least 45% are not at risk
of becoming endangered.
[A] 1 and 2 only
[B] 2 and 3 only
[C] 1 and 3 only
[D] 1 only Answer: Option A
[E] All of the above
ESI Homework Question GOVERNMENT SCHEMES
Q5. The Swachh Bharat Mission - Urban 2.0 (SBM-U 2.0) aims to address comprehensive
sanitation concerns in urban India through various mission components. Consider the
following statements about the components of SBM-U 2.0:

1. One of the mission's components is Sustainable Solid Waste Management, which


includes setting up construction & demolition waste processing facilities in all cities
regardless of population size.
2. Sustainable Sanitation is another component aimed at sustaining Open Defecation Free
status in all Statutory towns through the construction of Individual Household Latrines
(IHHL) and Community and Public Toilet (CT and PT) seats.
3. Used Water Management (UWM) is a new component introduced in SBM-U 2.0,
focusing on ensuring that no untreated fecal sludge or used water is discharged into
the environment in cities with more than 5 lakh population.
[A] 1 only
[B] 2 only
[C] 1 and 2 only
[D] 2 and 3 only
[E] 1, 2, and 3
HOMEWORK QUESTIONS
FROM PREVIOUS SESSION
ESI Homework Question GOVERNMENT SCHEMES

Q5. Recently in February 2024, the Union Cabinet approved the continuation of the Flood
Management and Border Areas Programme (FMBAP) for the period 2021-2026. Regarding the
details of FMBAP and its related aspects, which of the following statements are true?
1. FMBAP was initiated by merging "Flood Management Programme (FMP)" and "River
Management Activities and Works related to Border Areas (RMBA)".
2. The scheme primarily places the responsibility of flood management on the Union Government,
supplementing state efforts.
3. FMP and RMBA were both started during the XII Plan.
4. The funding pattern for FMP among Centre and States is 75:25 for General/Non-Special
Category States.
5. The scheme includes a provision for incentivizing states that implement flood plain zoning as a
non-structural measure for flood management.
[A] 1 and 5
[B] 1, 3, and 5
[C] 2, 4, and 5 Answer: Option A
[D] All of the above
[E] None of the above
Topic Tag
PHASE I + II

PM - USHA ESI – Government Schemes

G33 Meeting on Agriculture ESI – Regional Economic Co-operation

PHASE I
Vyomnauts GA – Science and Technology/Persons in
News
Hanooman AI GA – Science and Technology
Largest Solar Battery Project of India GA – Environnment
Shahpur Kandi Project GA – Places in News
Sudarshan Setu GA – Places in News
Yashasvi Jaiswal GA – Persons in News
RELEVANT FOR
PHASE 1 & PHASE 2
• Palamuru University in Telangana under the Multidisciplinary Research
University (MERU).
• Only university in Telangana to get this grant.
• Karnatak University Dharwad has been chosen for a grant of ₹20 crore.
• Uttar Pradesh has secured Rs 740 crore under the Pradhan Mantri
Uchchatar Shiksha Abhiyan (PM-USHA) programme, the highest amount in
the country
• Six universities receiving Rs 100 crore each for the development of
these institutions as multi-disciplinary education and research
universities (MERUs).
• Rs 140 crore has been distributed among eight other universities in the RUSA scheme has been
state for renovating dilapidated buildings. launched as PM-USHA in line
with NEP 2020.
• Dubrugarh University, Assam has granted an amount of Rs 100 crore to
develop the university as a Multi-Disciplinary Education & Research RUSA was launched in 2013 with
University (MERU) its 2nd phase launched in 2018
About Pradhan Mantri Ucchatar Shiksha Abhyaan
• Launch: 2023
• Nodal Ministry: Ministry of Education
• Type of Scheme: Centrally Sponsored Scheme
• Funding Pattern between Centre and State Governments
• 90:10 for NorthEastern States, J&K, Himachal Pradesh, and Uttarakhand
• 60:40 for Other States and UTs with Legislature
• UTs without Legislature - 100% centrally funded
• Covers government and government-aided institutions of the States and UTs
• Aims to cater to 5 pillars of NEP 2020 - accessibility, quality, equity, accountability,
and affordability
• Priority would be given to Focus Districts

Identified by concerned States/UTs subject to a maximum of 50% of their districts


considering lack of access to government colleges and proportion of
disadvantaged categories such as SC/STs/OBCs/Women and remote/aspirational
districts.
Objectives

• Focus on low GER, Left Wing Extremism (LWE), border area districts,
aspirational districts and districts with higher SC/ ST population

• To improve overall quality of existing state higher educational institutions

• Usher transformative reforms in State higher education system by creating


a facilitating institutional structure, promoting autonomy in State
Universities

• Providing better hostel facilities in remote areas

• Create enabling atmosphere in higher educational institutions to devote


themselves to research and innovations

• Correct regional imbalances in access to higher education by facilitating


access to high-quality institutions in urban & semi‐urban areas

• Establishing New Model Degree Colleges in districts where there are no


Government and Government-aided institutions.

• Enhancing employability through skilling and vocationalization


Focus on
• Equity, Access, and Inclusion
• Providing adequate opportunities to underprivileged groups, increase Gross
Enrolment Ratio (GER), increasing accessibility among different courses
• Developing Quality Teaching & Learning processes
• Inclusion of innovative and adaptable curricular structures, engaging innovative
pedagogy
• Would provide facilities to institution for upgrading physical and digital
infrastructure with conversion of single-stream higher education institutions
(HEIs) into multiple streams institutions
• Faculty training would be supported especially with help of digital infrastructure.
• Institutions would be facilitated to form alliances with Pandit Madan Mohan
Malviya National Mission on Teachers and Teaching (PMMMNMTT) and
University Grant Commission’s Human Resource Development Centres for
Resource Sharing
• Multiple Entry & Exit system, Choice-based Credit System (CBCS), and Academic
Bank of Credit (ABC) would be supported and prioritized
• Accreditation of Non-Accredited Institutions & Improving Accreditation
• ICT-based Digital Infrastructure like National Digital Library, DigiLocker, e-
Shodh Sindu, and virtual laboratories will use and give students a virtual
platform for their learning opportunities
• Enhancing Employability through Multidisciplinarity by focus on
establishment of employment cells to create linkages between Academia,
Industry, and Market
Components
• Multi-Disciplinary Education and Research Universities
(MERU) - 35
• Grants to Strengthen Universities (Accredited &
Unaccredited Universities) - 73
• Grants to Strengthen Colleges (Accredited & Unaccredited
Colleges) - 401
• New Model Degree Colleges - 40
• Gender Inclusion and Equity Initiatives - 50 Districts
• Management Monitoring Evaluation and Research (MMER
Key Highlights
• G-33 advocated for a permanent solution on public stockholding
(PSH) for food security.
o A peace clause agreed to at Bali Ministerial Conference of
2013 gives many developing countries, including India,
immunity against legal action from other members in case
the PSH subsidies exceed the limit of 10% of value of
production.
✓ But it is limited to programmes existing in 2013 and
riddled with many onerous conditions.
• G-33 also pushed for the adoption of Special Safeguard
Mechanism (SSM) by the next Ministerial Conference to protect
against import surges and price declines.
• Special and Differential Treatment (S&D) for developing
countries was strongly advocated to be preserved in WTO
agreements.
RELEVANT FOR
PHASE 1
About Gaganyaan
The project envisages
demonstration of
human spaceflight
capability by launching
a crew of four members
to an orbit of 400 km
for a three-day mission
and bring them back
safely to earth, by
landing in Indian water
About BharatGPT Hanooman
• Spearheaded by IIT Bombay.
• Backed by the Department of Science and Technology.
• Aim/Objective: To revolutionize various sectors including healthcare, governance,
financial services, and education with AI innovation, and to establish India's leadership
in the global AI arena.
• Features:
• Languages: Supports 22 Indian languages, aiming for linguistic inclusivity and
accessibility.
• Multimodal Capabilities: Capable of generating text, speech, and video content,
facilitating versatile applications.
• Size and Complexity: Parameters range from 1.5 billion to 40 billion, allowing for
nuanced and contextually relevant responses.
• Open Source: Ensures accessibility and encourages community development.
Additional Information

• Hanooman is a series of Indic large language models trained on 22


Indian languages.
• It is unveiled by Seetha Mahalaxmi Healthcare (SML) in partnership
with the IIT Bombay-led BharatGPT ecosystem.
• Commissioned by: Solar Energy Corporation of India
Limited (SECI), under the aegis of the Ministry of
New and Renewable Energy.
• Located at: Rajnandgaon, Chhattisgarh.
• Capacity: 40 megawatts (MW) / 120MWh BESS
• Project funded by: World Bank and Clean
Technology Fund under the Innovation in Solar
Power & Hybrid Technologies Project.
• The Shahpur Kandi (Pathankot district, Punjab)
Barrage on the Ravi River has been completed.

• It allows India to stop the flow of water to Pakistan


and utilize it for irrigation purposes in the Kathua and
Samba districts of Jammu and Kashmir.

• It represents a strategic shift in water management


and allocation between India and Pakistan,
particularly in the context of the 1960 Indus Water
Treaty.
Key Features

• Connects Beyt Dwarka island to mainland Okha in


Gujarat's Devbhumi Dwarka district.

• India’s longest cable-stayed bridge (2.32 Km).

• 4-lane cable-stayed bridge.

• Also known as Okha-Beyt Dwarka Signature Bridge.

• Features a footpath adorned with verses from the


Bhagavad Gita and images of Lord Krishna.
• Recently, Yashasvi Jaiswal equalled Virat Kohli’s haul
of 655 runs during the 4th Test between India and
England in Ranchi.

• He also became the joint-second-highest Indian run-


getter in a home Test series.

• Sunil Gavaskar holds the top spot in this list with 732
runs scored against the West Indies in the 1978/79
series.
Let’s Practice
ESI GLOBALIZATION

Q.1) Recently in February 2024, during the 13th WTO Ministerial Conference in Abu
Dhabi, the G-33 group emphasized several points concerning agriculture trade
negotiations. Which of the following statements is correct regarding their proposals?
[A] The G-33 rejected the concept of public stockholding (PSH) for food security
purposes.
[B] The G-33 proposed a temporary solution for public stockholding (PSH) that would
be applicable to new programs only.
[C] The G-33 advocated for the adoption of a Special Safeguard Mechanism (SSM) by
the next Ministerial Conference to protect against import surges and price declines.
[D] The G-33 emphasized the elimination of Special and Differential Treatment (S&D)
for developing countries in WTO agreements.
[E] The G-33 announced a new initiative to increase the limit of PSH subsidies to 20%
of the value of production.

Answer: Option C
GA SCIENCE AND TECH

Q.2) Which of the following statements is correct about BharatGPT Hanooman?


[A] It is primarily a commercial venture by Reliance Jio and 7 Indian Institutes of
Technology (IITs).
[B] The AI model supports only 10 Indian languages to ensure specialization in high-
demand languages.
[C] BharatGPT Hanooman is an open-source project with multimodal capabilities
supporting text, speech, and video content generation.
[D] The AI model is designed with parameters ranging only up to 1.5 billion to focus on
speed over complexity.
[E] None of the Above

Answer: Option C
GA PLACES IN NEWS
Q.3) Recently (February 2024), the Shahpur Kandi Barrage was completed, allowing
India to utilize the water for irrigation purposes in Jammu and Kashmir for the usage of
water under Indus Water Treaty. Which of the following statement is correct with
respect to Indus Water Treaty?
[A] It was signed in 1960.
[B] It was signed between India, Pakistan and IMF.
[C] India has the full right of usage of water over Indus, Jhelum and Ravi.
[D] Shahpur Kandi project is established on riven Jhelum.
[E] All of the above

Answer: Option A
GA PERSONS IN NEWS

Q.4) Recently (February 2024), which cricketer equalled Virat Kohli's haul of 655 runs in
a home Test series during the 4th Test between India and England in Ranchi?
[A] Rohit Sharma
[B] Cheteshwar Pujara
[C] Ajinkya Rahane
[D] Shubman Gill
[E] Yashasvi Jaiswal

Answer: Option E
GA PLACES IN NEWS

Q.4) Recently (February 2024), Prime Minister of India has inaugurated India's longest
cable-stayed bridge connecting Beyt Dwarka island to mainland Okha in Gujarat's
Devbhumi Dwarka district. Which of the following statement is correct with respect to
the project?
1. It was named as Dwarka Setu.
2. It features a footpath adorned with verses from the Bhagavad Gita and images of
Lord Krishna.
3. It was funded by World Bank and Gujarat Government.
[A] Only 1
[B] Only 2
[C] Only 3
[D] 2 and3 Answer: Option B
[E] 1 and 3
GA PLACES IN NEWS

Q.5) Recently (February 2024), Solar Energy Corporation of India Limited (SECI)
commissioned India's largest Battery Energy Storage System (BESS) integrated with a
solar photovoltaic (PV) plant. In which of the following locations is this plant situated?
[A] Bilaspur
[B] Durg
[C] Rajnandgaon
[D] Raipur
[E] Korba

Answer: Option C
GA Homework Question SCIENCE AND TECHNOLOGY

Q.6) Recently (February 2024), Prime Minister has visited the Vikram Sarabhai Space
Centre (VSSC) in Thiruvananthapuram to unveil the names of the four selected
astronauts for the historic Gaganyaan mission. Which among the following is not one of
the selected astronauts?
1. Captain Prasnath Balakrishnan Nair
2. Captain Ajit Kumar
3. Captain Angad Rawat
4. Wing Commander Shubhanshu Shukla
[A] 1 and 2
[B] 1, 2, 3 and 4
[C] 1 and 4
[D] 2, 3 and 4
[E] 1 and 3
ESI Homework Question GOVERNMENT SCHEMES

Q.7) Which the following statement/s is/are correct with respect to objectives of the
Pradhan Mantri Uchchatar Shiksha Abhiyan (PM-USHA)?
1. One of the objectives of the scheme is to improve overall quality of existing state
higher educational institutions by ensuring their conformity to prescribed norms and
standards and adoption of accreditation as a quality assurance framework.
2. New Model Degree Colleges in districts where there are no private institutions to be
established.
3. Employability enhancement through skilling and vocationalization to be done.
[A] 1 and 2
[B] 2 and 3
[C] Only 1
[D] 1, 2 and 3
[E] 1 and 3
HOMEWORK QUESTIONS
FROM PREVIOUS SESSION
ESI Homework Question GOVERNMENT SCHEMES

Q5. Which of the following statement(s) is/are correct with respect to Jal Jeevan
Mission (JJM)?
1. Community needs to contribute 5% or 10% of capital cost for in-village
infrastructure as the case may be, in cash/ kind/ and/ or labour.
2. Every village to prepare a Village Action Plan (VAP) which will have two
components.
3. Technological interventions to be provided for removal of contaminants where
water quality is an issue.
4. Greywater (industrial wastewater) management is also covered under Village
Action Plan (VAP).
A. 1, 2 and 3
B. 2 and 3
C. 1 and 3 Answer: Option C
D. 1, 2 and 4
E. 2, 3 and 4
Topic Tag
PHASE I + II

Alliance for Global Good Gender Equity and ESI – Gender Issues
Equality
Garbhini ESI – Gender Issues

PHASE I
Gene Therapy for Haemophilia GA – First In News
Pushpak GA – Science and Technology
Reliance Disney Merger GA – MOUs and Agreements
RELEVANT FOR
PHASE 1 & PHASE 2
Logo
• Alphabet “e” represents Equality and Equity
• Parallel strokes reflect collective action
• Tricolours symbolize strength, courage (Saffron), truth
(White), and growth (Green).
• Red dot denotes prosperity
About Alliance for Global Good- Gender Equity and Equality
• Launched at 54th World Economic Forum's (WEF) Annual Meeting 2024 in
Davos.
• Ministry: Ministry of Women and Child Development
• Implementing Agency: CII Centre for Women Leadership
• Developed by:
o Supported by the Bill and Melinda Gates Foundation;
o World Economic Forum as a ‘Network Partner’
o Invest India as an 'Institutional Partner'
• Objective: To bring together global best practices and investments in
women’s health, education, and enterprise.
• Part of the G20 (Group of 20) Leaders' Declaration (New Delhi Leaders’
Declaration).
• Territorial Coverage: Global outreach.
Why in News
• Researchers from BRIC-THSTI Faridabad and IIT Madras have developed " Garbhini-
GA2," first India-specific Artificial Intelligence (AI) model for accurately determining the
age of a fetus in the second and third trimesters.
Key Highlights
• Developed by: Biotechnology Research and Innovation Council - Translational Health
Science and Technology Institute (THSTI), Faridabad, and Indian Institute of Technology
Madras (IIT Madras), utilizing the GARBH-Ini cohort data from Gurugram Civil Hospital,
Haryana.
• Garbhini-GA2 model significantly reduces the gestational age (GA) estimation median
error by more than 3 times compared to commonly used Hadlock model.
• Part of Interdisciplinary Group for Advanced Research on Birth Outcomes – DBT India
Initiative’ (GARBH-Ini) program.
• Funded by: Grand Challenges India programme of the Biotechnology Industry Research
Assistance Council, DBT.
About GARBH-Ini
• Launch Year: 2014, with a cohort started at the Gurugram Civil Hospital in 2015.
• Objective: To elucidate biological and non-biological risks of preterm birth (PTB) and
create interventions and technologies for sustainable implementation in clinical practice
and the community.
• Flagship programme of Department of Biotechnology (DBT).
• Included as one of the Atal Jai-Anusandhan Mission Programmes by the Ministry of
Science and Technology.
• Features:
o An inter-institutional interdisciplinary research program.
o Multi-Omics Approach: Identification of molecular determinants across normal
pregnancy for the discovery of biomarkers for PTB prediction.
Key Terms
• Gestational Age (GA): The age of the fetus calculated from the first day of the woman's
last menstrual period.
RELEVANT FOR
PHASE 1
• Supported by Department of Biotechnology and Centre for Stem Cell
Research, in collaboration with Emory University (USA).
Significant S&T Achievements
• India's bio-economy has grown from $10 billion in 2014 to over $130 billion
in 2024. Hemophilia A is characterized by
• India is globally among the top 5 countries in scientific research publications. deficiency in factor VIII clotting
activity that results in prolonged
• Global Innovation Index: 40th Rank. bleeding after injuries, tooth
• Patent filings have crossed 90,000, the highest in two decades. extractions, or surgery, and delayed
or recurrent bleeding prior to
• A compendium of Sophisticated Analytical & Technical Help Institutes (SATHI) complete wound healing.
Clusters was also unveiled by felicitating the heads of IIT – Hyderabad, BITS –
Pilani, ICT – Mumbai
• A white paper on "Evolution: Catalysing Technology-led Ecosystem for Bharat
e-Mobility" was released.
• Developed by: Indian Space Research Organisation (ISRO)
• Design and Capabilities: An all-rocket, fully reusable single-
stage-to-orbit (SSTO) vehicle.
• Objective: To significantly reduce the costs associated with
space launches by nearly 80%.
• Design received approval from the National Review Committee
in 2012.
• A successful autonomous landing mission was conducted in
2023.

Additional Information
• Currently, ISRO has 3 active launch vehicles: the Polar Satellite Launch Vehicle (PSLV), the Geosynchronous
Satellite Launch Vehicle (GSLV), and the Launch Vehicle Mark-III (LVM3).
• Other RLVs: SpaceX’s Falcon 9 (a reusable, two-stage rocket).
Entities Involved in the MoU:
• Reliance Industries Limited (RIL)
• Viacom 18 Media Private Ltd (Owned by Reliance)
• Walt Disney Company
Purpose of the MoU:
• To merge Viacom18's media operations into Star India through
a court-approved scheme of arrangement, forming a joint
venture (JV) valued at Rs 70,352 crore ($8.5 billion).
Key Facts
• JV will become one of India's leading media companies,
offering entertainment and sports content to over 750 million
viewers, including exclusive distribution rights for Disney films
and productions in India.
Ownership Split
• RIL will own 16.34%, Viacom18 will own 46.82%, and Disney will
own 36.84% of the JV.
• RIL and its affiliates will collectively hold a 63.16% stake,
controlling the JV.
Leadership
• Nita Ambani – Chairperson
• Uday Shankar - Vice Chairperson.
Let’s Practice
GA FIRST IN NEWS

Q1. Which of the following statements accurately describes an initiative or achievement related to
science and technology in India, as reported in recent updates?
A) India conducted its first animal clinical trials for gene therapy targeting cardiovascular diseases
at the All India Institute of Medical Sciences (AIIMS), New Delhi, supported by the Indian Council of
Medical Research (ICMR).
B) The Department of Biotechnology and the Centre for Stem Cell Research, in collaboration with
Harvard University, have conducted India's first human clinical trial of gene therapy for haemophilia
B (FIX deficiency) at Christian Medical College (CMC), Vellore.
C) India's bio-economy has escalated from $10 billion in 2014 to over $130 billion in 2024,
positioning the country globally among the top 5 in scientific research publications and witnessing
a significant rise in the Global Innovation Index from 81st in 2015 to 40th.
D) The Department of Science and Technology launched the Sophisticated Analytical & Technical
Help Institutes (SATHI) initiative, aiming to establish shared, professionally managed Science and
Technology infrastructure facilities to support academia, startups, manufacturing units, industries,
and R&D labs, with the primary focus on enhancing chemical engineering research.
E) A white paper titled "Evolution: Catalysing Technology-led Ecosystem for Bharat e-Mobility" was
released alongside the unveiling of a compendium of Sophisticated Analytical & Technical Help
Institutes (SATHI) Clusters
Answer: Option C
ESI WOMEN

Q2. Which of the following statements accurately describes the Alliance for
Global Good-Gender Equity and Equality?
[A] The Alliance was launched with the primary goal of enhancing women
participation in military.
[B] The initiative is solely funded and operated by the World Economic Forum.
[C] Alliance focuses on domestic policies within India.
[D] Alliance aims to consolidate global best practices and investments in
women’s health, education, and enterprise, with a global outreach.
[E] None of the Above

Answer: Option D
ESI HEALTH
Q3. Recently in February 2024, researchers from BRIC-THSTI Faridabad and IIT Madras developed
"Garbhini-GA2," the first India-specific Artificial Intelligence (AI) model for accurately determining the age of
a fetus in the second and third trimesters. Consider the following statements about the GARBH-Ini initiative
and the Garbhini-GA2 model:
1. The Garbhini-GA2 model was developed as part of the "Interdisciplinary Group for Advanced Research on
Birth Outcomes – DBT India Initiative" (GARBH-Ini) program.
2. Garbhini-GA2 model utilizes cohort data from AIIMS, New Delhi.
3. The model significantly reduces the gestational age estimation median error by more than three times
compared to the commonly used Hadlock model.
4. The GARBH-Ini program was launched in 2014, with its cohort starting at the Gurugram Civil Hospital in
2015.
5. Funding for the GARBH-Ini program and the development of Garbhini-GA2 comes from the National
Health Mission.
Which of the statements given above is/are correct?
[A] 1, 3, and 4 only
[B] 2 and 5 only
[C] 1, 2, and 3 only Answer: Option A
[D] All of the above
[E] None of the above
GA MoUs and Agreements

Q4. Which of the following statements accurately reflects details of the joint
venture (JV) formed between Reliance Industries Limited, Viacom 18 Media
Private Ltd, and Walt Disney Company?
[A] The JV is exclusively focused on producing sports content in India.
[B] The ownership of the JV is majorly held by Disney, with an 80% stake,
showcasing its dominance in the partnership.
[C] Nita Ambani will serve as the Chairperson, and Uday Shankar will be the
Vice Chairperson of the newly formed JV.
[D] The JV aims to merge Viacom18's media operations into Star India,
creating a venture valued at approximately Rs 10,000 crore ($1.2 billion).
[E] Reliance Industries Limited will have the smallest ownership stake in the
JV, with less than 10% control.

Answer: Option C
GA Homework Question SCIENCE AND TECH

Q5. Which of the following statements about India's Reusable Launch Vehicle - Pushpak,
developed by ISRO, is correct?
[A] Pushpak's design is aimed at decreasing the space launch costs by about 99%.
[B] The design of Pushpak was approved by the National Review Committee in 2015.
[C] Pushpak was first successfully launched and had its autonomous landing in 2022.
[D] ISRO's Pushpak is an all-rocket, fully reusable single-stage-to-orbit (SSTO) vehicle.
[E] Pushpak is the only reusable launch vehicle in the World.
HOMEWORK QUESTIONS
FROM PREVIOUS SESSION
GA Homework Question SCIENCE AND TECHNOLOGY

Q.6) Recently (February 2024), Prime Minister has visited the Vikram Sarabhai Space
Centre (VSSC) in Thiruvananthapuram to unveil the names of the four selected
astronauts for the historic Gaganyaan mission. Which among the following is not one of
the selected astronauts?
1. Captain Prasnath Balakrishnan Nair
2. Captain Ajit Kumar
3. Captain Angad Rawat
4. Wing Commander Shubhanshu Shukla
[A] 1 and 2
[B] 1, 2, 3 and 4
[C] 1 and 4
[D] 2, 3 and 4
[E] 2 and 3

Answer: Option E
ESI Homework Question GOVERNMENT SCHEMES

Q.7) Which the following statement/s is/are correct with respect to objectives of the
Pradhan Mantri Uchchatar Shiksha Abhiyan (PM-USHA)?
1. One of the objectives of the scheme is to improve overall quality of existing state
higher educational institutions by ensuring their conformity to prescribed norms and
standards and adoption of accreditation as a quality assurance framework.
2. New Model Degree Colleges in districts where there are no private institutions to be
established.
3. Employability enhancement through skilling and vocationalization to be done.
[A] 1 and 2
[B] 2 and 3
[C] Only 1
[D] 1, 2 and 3
[E] 1 and 3

Answer: Option E

You might also like